PEDs 2

Pataasin ang iyong marka sa homework at exams ngayon gamit ang Quizwiz!

"A child is admitted to the hospital with a diagnosis of Wilm's tumor, Stage II. Which of the following statements most accurately describes this stage? "A) The tumor is less than 3 cm. in size and requires no chemotherapy. B) The tumor did not extend beyond the kidney and was completely resected. C) The tumor extended beyond the kidney but was completely resected. D) The tumor has spread into the abdominal cavity and cannot be resected."

"1. Answer: C The staging of Wilm's tumor is confirmed at surgery as follows: Stage I, the tumor is limited to the kidney and completely resected; stage II, the tumor extends beyond the kidney but is completely resected; stage III, residual nonhematogenous tumor is confined to the abdomen; stage IV, hematogenous metastasis has occurred with spread beyond the abdomen; and stage V, bilateral renal involvement is present at diagnosis."

"The female client recently diagnosed with Hodgkin's lymphoma asks the nurse about her prognosis. Which is the nurse's best response? "1. Survival for Hodgkin's disease is relatively good with standard therapy. 2. Survival depends on becoming involved in an investigational therapy program. 3. Survival is poor, with more than 50% of clients dying within six (6) months. 4. Survival is fine for primary Hodgkin's, but secondary cancers occur within a year."

"1. Up to 90% of clients respond well to standard treatment with chemotherapy and radiation therapy, and those who relapse usually respond to a change of chemotherapy medications. Survival depends on the individual client and the stage of disease at diagnosis (correct). 2. Investigational therapy regimens would not be recommended for clients initially diagnosed with Hodgkin's disease because of the expected prognosis with standard therapy 3. Clients usually achieve a significantlylonger survival rate than six (6) months.Many clients survive to develop long-termsecondary complications. 4. Secondary cancers can occur as long as 20 years after a remission of the Hodgkin'sdisease has occurred."

Which condition assessed by the nurse would be an early warning sign of childhood cancer? 1. difficulty swallowing \ 2. nagging cough or hoarseness 3. slight changes in bowel and bladder function 4. swelling, lumps, masses on body

"Correct: 4. Swelling or lumps or masses anywhere on the body are early warning signs whereas difficulty swallowing or cough or hoarseness are signs of cancer in adults. there may be a marked sign in changes to bowel or bladder function, not a slight change"

A pediatric nurse specialist provides a teaching session to the nursing staff regarding osteogenic sarcoma. Which statement by a member of the nursing staff indicates a need for clarification of the information presented? "1.) ""The femur is the most common site of this sarcoma."" 2.) ""The child does not experience pain at the primary tumor site."" 3.) ""Limping, if a weight-bearing limb is affected, is a clinical manifestation."" 4.) ""The symptoms of the disease in the early stage are almost always attributed to normal growing pains."""

"2.) ""The child does not experience pain at the primary tumor site."" (CORRECT ANSWER--Need for further clarification). Osteogenic sarcoma is the most common bone cancer in children. Cancer usually is found in the metaphysis of long bones, especially in the lower extremities, with most tumors occurring in the femur (omit #1). Osteogenic sarcoma is manifested clinically by progressive, insidious, and intermittent pain at the tumor site (correct answer: #2). By the time these children receive medical attention, they may be in considerable pain from the tumor. All options: 1, 3, 4 are accurate regarding osteogenic sarcoma. "

A woman is to undergo chorionic villus sampling as part of a risk assessment for genetic disorders. Which of the following would the nurse include when describing this test to the woman? a) "A needle will be inserted directly into your fetus's umbilical vessel to collect blood for testing." b) "An intravaginal ultrasound measures fluid in the space between the skin and spine." c) "A small piece of tissue from the fetal placenta will be removed and analyzed." d) "A small amount of amniotic fluid will be withdrawn and collected for analysis."

"A small piece of tissue from the fetal placenta will be removed and analyzed." Correct Explanation: Percutaneous umbilical cord sampling involves the insertion of a needle into the umbilical vessel. An amniocentesis involves the collection of amniotic fluid from the amniotic sac. Fetal nuchal translucency involves the use of intravaginal ultrasound to measure fluid collected in the subcutaneous space between the skin and cervical spine of the fetus. Chorionic villus sampling involves the removal of a small tissue specimen from the fetal portion of the placenta

The nurse understands a primary step toward achievement of a long range goal associated with the rehabilitation of a client with a new colostomy is: "A. Mastery of techniques of colostomy care B. Readiness to accept an altered body function C. Awareness of available community resources D.Knowledge of the neccessary dietary modifications.

"ANSWER: B The client must be ready to accept changes in body image and function; this acceptance will facilitate mastery of the techniques of colosotomy care and optimal use of community resources."

A client is admitted to the hospital for a colon resection and in preparation for surgery the physician orders neomycin. The nurse understands the main reason why this antibiotic is especially useful before colon surgery is because it: "A. Will not affect the kidneys B. Acts systemically without delay C.Has limited absorption from the GI tract. D.Is effective against many different organisms "

"ANSWER:C Because neomycin is limited absorption form the GI tract, it exerts it antibiotic effect on the intestinal mucosa. In preparation of GI surgery, the level of microbial organisms will be reduced."

A child with lymphoma is receiving extensive radiotherapy. Which of the following is the most common side effect of this treatment? "A. Malaise B. Seizures C. Neuropathy D. Lymphadenopathy"

"Answer A is Correct Malaise is the most common side effect of radiotherapy. For children, the fatigue may be especially distressing because it means they cannot keep up with their peers."

An adolescent with a history of surgical repair for undescended testes comes to the clinic for a sport physical. Anticipatory guidance for the parents and adolescent would focus on which of the following as most important? "a) the adolescent sterility b) the adolescent future plans c) technique for monthly testicular self-examinations d) need for a lot of psychosocial support"

"Answer C Because the incidence of testicular cancer is increased in adulthood among children who have undescended testes. It is extremely important to teach the adolescent how to perform the testicular self-examination monthly."

"The mother of a 4 year old child brings the child to the clinic and tells the pediatric nurse specialist that the child's abdomen seems to be swollen. During further assessment of the subjective data, the mother tells the nurse that the child has been eating well and that the activity level of the child is unchanged. The nurse, suspecting the possibility of a Wilm's tumor, would avoid which of the following during the physical assessment? "1. Palpating the abdomen for a mass. 2. Assessing the urine for hematuria 3. Monitoring the temperature for presence of fever 4. Monitoring the blood pressure for presence of hypertension"

"Answer: 1 Rationale: Wilm's tumor is the most common intra-abdominal and kidney tumor of childhood. If Wilm's tumor is suspected, the tumor mass should not be palpated by the nurse. Excessive manipulation can cause the seeding of the tumor and spread of cancerous cells. Fever, hematuria, and hypertension are all clinical manifestations of Wilm's tumor."

"Chemotherapy dosage is frequently based on total body surFace area (BSA), so it is important for the nurse to do which of the following before administering chemotherapy? "1. Measure abdominal girth 2. Claculate BMI 3. Ask the client about his/her height and weight 4. Weigh and measure the client on the day of medication administration"

"Answer: 4 To ensure that the client receives optimal doses of chemotherapy, dosing is usually based on the total Body surface area(BSA) which requires accurate height and weight before each med administration. Simply asking the client about height/weight may lead to inaccuracies in determining BSA. Calculating BMI and measuring abdominal girth does not provide the data needed."

A preschool-aged child is to undergo several painful procedures. Which of the following techniques is most-appropriate for the nurse to use in preparing the child? "A. Allow the child to practice injections on a favorite doll. B. Explain the procedure in simple terms. C. Allow a family member to explain the procedure to the child. D. Allow the child to watch an educational video."

"Answer: B Preschoolers have the cognitive ability to understand simple terms. Use of a favorite doll is contraindicated because it is ""part"" of that child and he/she might perceive the doll is experiencing pain."

"When assessing a child with Wilm's tumor, the nurse should keep in mind that it is most important to avoid which of the following? "A. Measuring the child's chest circumference B. Palpating the child's abdomen C. Placing the child in an upright position D. Measuring the child's occipitofrontal circumference"

"Answer: B. The abdomen of the child with Wilm's tumor should not be palpated because of the danger of disseminating tumor cells. Children with Wilm's tumor should always be handled gently and carefully. Other answers. The child's head and chest measuring will not affect Wilm's tumor. Repositioning a child in the upright position may cause more pain to the child, but priority this is not worse than disseminating tumor cells."

A school-age child is being seen in the oncology clinic for possible Hodgkin's disease. During the course of the nursing assessment, which findings would be expected? Select all that apply. "a) fever. b) painless cervical nodes. c) painful cervical nodes. d) poor appetite. e) complaints of night sweats"

"Answers: b and d (complaints of night sweats, painless cervical nodes.) Painless cervical nodes are a hallmark sign of HD. In addition to this, night sweats also are characteristic. Fever, poor appetite, and painful cervical nodes are more characteristic of infection."

"Nursing considerations related to the administration of chemotherapeutic drugs include which of the following? "a) Anaphylaxis cannot occur, since the drugs are considered toxic to normal cells. b) Infiltration will not occur unless superficial veins are used for the intravenous infusion. c) Many chemotherapeutic agents are vesicants that can cause severe cellular damage if drug infiltrates. d) Good hand washing is essential when handling chemotherapeutic drugs, but gloves are not necessary."

"CORRECT c. Chemotherapeutic agents can be extremely damaging to cells. Nurses experienced with the administration of vesicant drugs should be responsible for giving these drugs and be prepared to treat extravasations if necessary. a. Anaphylaxis is a possibility with some chemotherapeutic and immunologic agents. b. Infiltration and extravasations are always a risk, especially with peripheral veins. d. Gloves are worn to protect the nurse when handling the drugs, and the hands should be thoroughly washed afterward."

"A child is undergoing remission induction therapy to treat leukemia. Allopurinol is included in the regimen. The main reason for administering allopurinol as part of the client's chemotherapy regimen is to: "a. Prevent metabolic breakdown of xanthine to uric acid b. Prevent uric acid from precipitating in the ureters c. Enhance the production of uric acid to ensure adequate excretion of urine d. Ensure that the chemotherapy doesn't adversely affect the bone marrow"

"CORRECT: Answer A. The massive cell destruction resulting from chemotherapy may place the client at risk for developing renal calculi; adding allopurinol decreases this risk by preventing the breakdown of xanthine to uric acid. Allopurinol doesn't act in the manner described in the other options."

"After teaching the parents of a child newly diagnosed with leukemia about the disease, which of the following descriptions given by the mother best indicates that she understands the nature of leukemia?" "A) ""The disease is an infection resulting in increased white blood cell production."" B) ""The disease is a type of cancer characterized by an increase in immature white blood cells."" C) ""The disease is an inflammation associated with enlargement of the lymph nodes."" D) ""The disease is an allergic disorder involving increased circulating antibodies in the blood."""

"CORRECT: B. Leukemia is a neoplastic, or cancerous, disorder of blood-forming tissues that is characterized by a proliferation of immature white blood cells."

"A 10 year old child with hemophilia A has slipped on the ice and bumped his knee. The nurse should prepare to administer an: "A. injection of factor X B. intravenous infusion of iron C. intravenous infusion of factor VIII D. intramuscular injection of iron using the Z track method"

"CORRECT: C Hemophila refers to a group of bleeding disorders resulting from a deficiency of specific coagulation proteins. the primary meds used are to replace missing clotting factor. Factor VIII will be prescribed intravenously to replace the missing clotting factor and minimize the bleeding,"

"A child is diagnosed with Wilms' tumor. During assessment, the nurse in charge expects to detect: "a. Gross hematuria b. Dysuria c. Nausea and vomiting d. An abdominal mass"

"CORRECT: D The most common sign of Wilms' tumor is a painless, palpable abdominal mass, sometimes accompanied by an increase in abdominal girth. Gross hematuria is uncommon, although microscopic hematuria may be present. Dysuria is not associated with Wilms' tumor. Nausea and vomiting are rare in children with Wilms' tumor."

"Which nursing diagnosis is highest-priority for a child undergoing chemotherapy and experiencing nausea and vomiting? "A. Fluid and Electrolyte Imbalance B. Alterations in Nutrition C. Alterations in Skin Integrity D. Body Image Disturbances"

"Correct Answer: A While all of the nursing diagnoses listed here are important, dehydration and fluid and electrolyte loss secondary to vomiting is the priority for this client."

"The mother of a child diagnosed with a potentially life-threatening form of cancer says to the nurse, ""I don't understand how this could happen to us. We have been so careful to make sure our child is healthy."" Which response by the nurse is most appropriate? "A. ""This must be a difficult time for you and your family. Would you like to talk about how you are feeling?"" B. ""Why do you say that? Do you think that you could have prevented this?"" C. ""You shouldn't feel that you could have prevented the cancer. It is not your fault."" D. ""Many children are diagnosed with cancer. It is not always life-threatening."""

"Correct Answer: A Parents of children diagnosed with cancer require major emotional support, and should be allowed to express their feelings. Prevention and blaming oneself is not supportive, nor is telling the parents that there are many other children with cancer."

"After a client is admitted to the pediatric unit with a diagnosis of acute lymphocytic leukemia, the laboratory test indicates that the client is neutropenic. The nurse should perform which of the following? "a. Advise the client to rest and avoid exertion b. Prevent client exposure to infections c. Monitor the blood pressure frequently d. Observe for increased bruising"

"Correct Answer: B. Prevent client exposure to infections Rationale: Neutropenia is a decreased number of neutrophil cells in the blood which are responsible for the body's defense against infection. Rest and avoid exertion would be related to erythrocytes and oxygen carrying properties. Monitoring the blood pressure, and observing for bruising would be related to platelets and sign and symptoms of bleeding."

A patient who has been told by the health care provider that the cells in a bowel tumor are poorly differentiated asks the nurse what is meant by "poorly differentiated." Which response should the nurse make? "a. ""The cells in your tumor do not look very different from normal bowel cells."" b. ""The tumor cells have DNA that is different from your normal bowel cells."" c. ""Your tumor cells look more like immature fetal cells than normal bowel cells."" d. ""The cells in your tumor have mutated from the normal bowel cells."""

"Correct Answer: C Rationale: An undifferentiated cell has an appearance more like a stem cell or fetal cell and less like the normal cells of the organ or tissue. The DNA in cancer cells is always different from normal cells, whether the cancer cells are well differentiated or not. All tumor cells are mutations form the normal cells of the tissue."

"The most common signs and symptoms of leukemia related to bone marrow involvement are which of the following? "a. Petechiae, fever, fatigue b. Headache, papilledema, irritability c. Muscle wasting, weight loss, fatigue d. Decreased intracranial pressure, psychosis, confusion"

"Correct answer: A Signs of infiltration of the bone marrow are petechiae from lowered platelet count, fever related to infection from the depressed number of effective leukocytes, and fatigue from the anemia. The other options are not signs of bone marrow involvement."

"A child with cancer has the following lab result: WBC 10,000, RBC 5, and plts of 20,000. When planning this child's care, which risk should the nurse consider most significant? "A. Hemorrage B. Anemia C. Infection D. Pain"

"Correct answer: A Hemorrhage The lab values presented all are normal except for the platelet count. Decreases in platelet counts place the child at greatest risk for hemorrhage."

"The goals of cancer treatment are based on the principle that "a. surgery is the single most effective treatment for cancer. b. initial treatment is always directed toward cure of the cancer. c. a combination of treatment modalities is effective for controlling many cancers. d. although cancer cure is rare, quality of life can be increased with treatment modalities. "

"Correct answer: C Rationale: The goals of cancer treatment are cure, control, and palliation. When cure is the goal, treatment is offered that is expected to have the greatest chance of disease eradication. Curative cancer therapy depends on the particular cancer being treated and may involve local therapies (i.e., surgery or irradiation) alone or in combination, with or without periods of adjunctive systemic therapy (i.e., chemotherapy)."

"A preschool-age child undergoing chemotherapy experiences nausea and vomiting. Which of the following would be the best intervention to include in the child's plan of care? "a. Administer tube feedings. b. Offer small, frequent meals. c. Offer fluids only between meals. d. Allow the child to choose what to eat for meals."

"Correct answer: D While all options can be done to encourage nutrition, allowing the preschooler choices meets two issues: nutrition and developmental tasks."

"A 9-year old child with leukemia is in remission and has returned to school. The school nurse calls the mother of the child and tells the mother that a classmate has just been diagnosed with chickenpox. The mother immediately calls the clinic nurse because the leukemic child has never had chickenpox. The appropriate response by the clinic nurse to the mother is: "1. There is no need to be concerned. 2. Bring the child into the clinic for a vaccine. 3. Keep the child out of school for 2 week period. 4. Monitor the child for an elevated temperature, and call the clinic if this happens."

"Correct anwser: 2. Rationale: immunocompromised children are unable to fight varicella adequately. Chickenpox can be deadly to the them. If the child who has not had chickenpox is exposed to someone with varicella, the child should receive varicella zoster immune globulin within 96hrs of exposure. Options 1,3,4, are incorrect because they do nothing to minimize the chances of developing the disease."

"The nurse is admitting a patient who is jaundiced due to pancreatic cancer. The nurse should give the highest priority to which of the following needs? "1. Nutrition 2. Self-image 3. Skin integrity 4. Urinary elimination"

"Correct: 1. 1. profound weight loss and anorexia occur with pancreatic cancer. Correct. 2. jaundiced patients are concerned about how they look, but physiological needs take priority 3. jaundice causes dry skin and pruritis, scratching can lead to skin breakdown 4. urine is dark due to obstructive process, kidney function is not affected"

A nurse is teaching a client about the risk factors associated with colorectal cancer. The nurse determines that further teaching related to the colo-rectal cancer is necessary if the client identifies which of the following as an associated risk factor? "1. Age younger than 50 years 2. History of coloractal polyps 3. Family history of colorectal cancer 4. Chronic inflammatory bowel disease"

"Correct: 1. Colorectal cancer risk factors include age older than 50 years, a family history of the disease, colorectal polyps, and chronic inflammatory bowel disease."

"The pediatric nurse understands that the most common cancer found in children is: "1. Non-hodgkin's lymphoma 2. Acute lymphocytic leukemia 3. Chronic lymphocytic leukemia 4. Ewing's sarcoma"

"Correct: 2. 1. No - this is not a common cancer in children 2. YES! this is the most common form of cancer found in children is acute lymphocytic leukemia. 3. No - this is not a common cancer in children 4. No - this is not a common cancer in children"

The nurse is completing a care plan for a client diagnosed with leukemia. Which independent problem should be addressed? "1.Infection. 2.Anemia. 3.Nutrition. 4.Grieving."

"Correct: 4. Grieving is an independent problem, and the nurse can assess and treat this problem with or without collaboration."

A nurse is discussing childhood cancer with the parents of a child in an oncology unit. Which statement by the nurse would be the most accurate? "A. ""The most common site for children's cancer is the bone marrow."" B. ""All childhood cancers have a high mortality rate."" C. ""Children with leukemia have a higher survival rate if they are older than 11 when diagnosed."" D. ""The prognosis for children with cancer isn't affected by treatment strategies."""

"Correct: A. Childhood cancers occur most commonly in rapidly growing tissue, especially in the bone marrow. Mortality depends on the time of diagnosis, the type of cancer, and the age at which the child was diagnosed. Children who are diagnosed between the ages of 2 and 9 consistently demonstrate a better prognosis. Treatment strategies are tailored to produce the most favarable prognosis. (NCLEX-RN Questions & Answers, made Incredibly Easy)"

The mother of a 5-year-old child asks the nurse questions regarding the importance of vigilant use of sunscreen. Which information is most important for the nurse to convey to the mother? "a.) Appropriate use of sunscreen decreases the risk of skin cancer. b.) Repeated exposure to the sun causes premature aging of the skin. c.) A child's skin is delicate, and burns easily. d.) In addition to causing skin cancer, repeated sun exposure predisposes the child to other forms of cancer."

"Correct: A. While all of the answer choices are correct, recommending the use of sunscreen to decrease the incidence of skin cancer (a) is the best response."

"The parent of a child undergoing chemotherapy asks the nurse why the child must wear a mask in public places. Which of the following responses by the nurse would be most appropriate? "A) ""Chemotherapy causes dry mouth, and the mask will help contain moisture."" B) ""Chemotherapy decreases immune system function, increasing the risk of acquiring an infection."" C) ""Chemotherapy makes the oral mucous membranes deteriorate and makes them susceptible to infection."" D) ""Chemotherapy kills cancer cells, and your child might spread those cells to others."""

"Correct: B Chemotherapeutic agents decrease the immunity of the child. Proper use of the mask will decrease the chance of acquiring an infection. Cancer is not spread; a mask cannot contain moisture; and unsightly mouth sores are not a medical reason to wear a mask."

What are the needs of the patient with acute lymphocytic leukemia and thrombocytopenia? "(A) to a private room so she will not infect other patients and health care workers (B) to a private room so she will not be infected by other patients and health care workers (C) to a semiprivate room so she will have stimulation during her hospitalization (D to a semiprivate room so she will have the opportunity to express her feelings about her illness "

"Correct: B. a-— poses little or no threat B(CORRECT:)- protects patient from exogenous bacteria, risk for developing infection from others due to depressed WBC count, alters ability to fight infection c-— should be placed in a room alone d-ensure that patient is provided with opportunities to express feelings about illness"

"David, age 15 months, is recovering from surgery to remove Wilms' tumor. Which findings best indicates that the child is free from pain? "a. Decreased appetite b. Increased heart rate c. Decreased urine output d. Increased interest in play"

"Correct: D Answer D. One of the most valuable clues to pain is a behavior change: A child who's pain-free likes to play. A child in pain is less likely to consume food or fluids. An increased heart rate may indicate increased pain; decreased urine output may signify dehydration."

"The postoperative care of a preschool child who has had a brain tumor removed should include which of the following? "a. colorless drainage is to be expected b. analgesics are contraindicated because of altered consciousness c. positioning is on the operative side in the Trendelenberg position d. carefully monitor fluids due to cerebral edema"

"D CORRECT: Because of cerebral edema and the danger of increased intracranial pressure postoperatively, fluids are carefully monitored. A. Colorless drainage may be leakage of cerebrospinal fluid from the incision site. This needs to be reported as soon as possible. B. Analgesics can be used for postoperative pain. C. Child should not be positioned in Trendelenburg position postoperatively."

"A nurse is performing an assessment on a 10-year old child suspected having Hodgkin's disease. The nurse understands that which of the following assessment findings is characteristic of this disease? "a) fever and malaise b) anorexia and weight loss c) painful, enlarged inguinal lymph nodes d) painless, firm, and movable adenopathy in the cervical area"

"D. painless, firm, and movable adenopathy in the cervical area - Clinical manifestations specifically associated with Hodgkin's disease include painless, firm, and movable adenopathy in the cervical and supraclavicular areas. Hepatosplenomegaly also is noted. Although fever, malaise, anorexia, and weight loss are associated with Hodgkin's disease, these manifestations are seen in many disorders."

The parent of a child diagnosed with Duchenne muscular dystrophy asks why gene therapy is not being used to treat her child. What is the best response by the nurse? a) "Clinical trials are very successful, and you should find one immediately." b) "Gene therapy remains experimental and is used only in clinical trials." c) "Genetic testing is unethical." d) "Gene therapy does not work for muscular dystrophy."

"Gene therapy remains experimental and is used only in clinical trials." Correct Explanation: Gene therapy in the United States is currently experimental and is used only in clinical trials. Clinical trials have resulted in minimal success. No documentation supports the statement that gene therapy would not work for muscular dystrophy. Genetic testing is used to diagnose illness; therefore, it is widely accepted as ethical when used to diagnose disorders. Gene therapy may be viewed by some as unethical, but the nurse should provide information in a nonjudgmental manner.

A young couple who underwent preconceptual genetic counseling and testing have learned that they are at high risk for having a child with Down syndrome. They have decided not to have children. Which of the following would be the most appropriate response for the nurse to give? a) "I think you made the right decision. After all, I never had children, and I'm perfectly happy." b) "I appreciate your decision, but I urge you to think through this further. Having a child, even one with Down syndrome, is so rewarding." c) "I understand. In case you would like to discuss this further with a genetic counselor, here is the contact information for the genetic counseling center." d) "I understand and support your decision. The risk is just not worth it."

"I understand. In case you would like to discuss this further with a genetic counselor, here is the contact information for the genetic counseling center." Correct Explanation: Even if a couple decides not to have more children, be certain they know genetic counseling is available for them should their decision change. It is never appropriate for a health care provider to impose his or her own values or opinions on others. Individuals with known inherited diseases in their family must face difficult decisions, such as how much genetic testing to undergo or whether to terminate a pregnancy that will result in a child with a specific genetic disease. Be certain couples have been told all the options available to them, and then leave them to think about the options and make their decision by themselves. Help them to understand nobody is judging their decision because they are the ones who must live with the decision in the years to come.

A couple has just learned that their son will be born with Down's syndrome. The nurse shows a lack of understanding when making which of the following statements? a) "I will alert your entire family about this so you don't have to." b) "We have counseling services available, and I recommend them to everyone facing these circumstances." c) "I will support you in any decision that you make." d) "I will give you as much information as I can about this condition."

"I will alert your entire family about this so you don't have to." Correct Explanation: It is necessary to maintain confidentiality at all times, which prevents healthcare providers from alerting family members about any inherited characteristic unless the family member has given consent for the information to be revealed.

Which statement by the parent of a 12-month-old child diagnosed with Down syndrome shows the need for further education? a) "Thyroid testing is needed every year." b) "I will need to delay any further immunizations." c) "In a couple of years, my child will need an x-ray of the neck." d) "I will watch closely for development of respiratory infection."

"I will need to delay any further immunizations." Correct Explanation: Down syndrome children are at higher risk for infection because of a lowered immune system. Delaying immunizations may expose the child to illnesses that could have been prevented. Down syndrome children are at greater risk for developing thyroid disorders, 1st and 2nd vertebrae disorders, and respiratory infections.

Which statement by a parent regarding mitochrondrial disorders requires further education? a) "It is passed from female to female. That's why my son cannot be affected." b) "The cells most affected are the ones that require high levels of energy." c) "My child can exhibit signs and symptoms of the disorder at any point in his life." d) "Mitochondrial disorders usually worsen over time."

"It is passed from female to female. That's why my son cannot be affected." Explanation: Mitochondrial disorders usually are inherited from the mother and affect offspring regardless of sex. Mitochondrial disorders are progressive, and onset of signs and symptoms can occur from infancy to adulthood. The disorder affects cells that require high levels of energy

A community health nurse is visiting her 16-year-old patient, a new mother. The nurse explains to the patient and her mother the genetic screening that is required by the state's law. The patient asks why it is important to have the testing done on the infant. What is the nurse's best response? a) "This testing is required and you will not be able to refuse it. It usually is free so there is no reason to refuse it." b) "Genetic testing is a way to determine the rate of infectious disease." c) "It is important to test newborns for PKU, congenital hypothyroidism, and galactosemia." d) "PKU, congenital hypothyroidism, and galactosemia are conditions that could result in disability or death if untreated."

"PKU, congenital hypothyroidism, and galactosemia are conditions that could result in disability or death if untreated." Correct Explanation: The first aim is to improve management, that is, identify people with treatable genetic conditions that could prove dangerous to their health if left untreated. The other answers are incorrect because genetic testing does not determine the rate of infectious disease. Answer B does not adequately explain the rationale for newborn testing. Answer D fails to inform the patient of the rationale for newborn testing.

The parents of a child diagnosed with Tay-Sachs inquire about progression of the disorder. Which statement by the nurse is accurate? a) "Anticonvulstants will be given to prolong life and prevent further brain damage." b) "Lifetime steroid therapy will reverse the blindness." c) "The child will experince decreased muscular and neurologic functioning until death occurs." d) "Symptoms can be controlled by eliminating dairy products."

"The child will experince decreased muscular and neurologic functioning until death occurs." Correct Explanation: This is an irreversible progressive disorder that affects the functioning of muscles and the neurologic system. Symptoms cannot be controlled by changes in the diet, and medication therapy will not reverse symptoms nor prolong life. Medication will be used to treat symptoms and provide comfort measures.

The nurse is counseling a couple who are concerned that their children might inherit sickle cell disease. Which of the following responses from the couple indicate a need for further teaching? a) "The disorder can be passed on to the children only if both parents have the gene." b) "If both parents have the gene, there is a 25% chance of the children having the disorder." c) "The father cannot pass the disorder onto his son or the mother to her daughter." d) "Even if the children do not get the disease they can still be carriers of the gene."

"The father cannot pass the disorder onto his son or the mother to her daughter." Correct Explanation: The father can pass the gene to his sons and the mother can pass the gene to her daughters. Sickle cell disease is an autosomal recessive disease. This means that both parents must have the disease or be carriers of the gene in order to pass it onto their children. If the parents are both carriers, then there is a 25% chance that they will pass it onto a child. The children can be carriers even if they don't have the disease.

Which statement by the nurse accurately describes the term phenotype? a) "The genetic makeup of an individual" b) "The somatic cells of an individual" c) "The individual's outward appearance" d) "Only the homozygous genes outwardly expressed"

"The individual's outward appearance" Correct Explanation: Phenotype is the outward characteristic of an individual. The genetic makeup of an individual is a genotype. A somatic cell is an individual cell that combines with others to form an organism. Phenotype can be determined by both homozygous genes and heterozygous genes.

Which statement by the nurse is most accurate when counseling a couple about transmitting Huntington's disease from father to child? a) "A daughter cannot be a carrier of the disease because it is an X-linked recessive disorder." b) "There is a 50% chance of transmission of the disorder because it is an autosomal dominant disorder." c) "You will transmit the disorder to a son because it is an X-linked dominant disorder." d) "There is a 75% chance that your offspring will express the disorder because it is an autosomal recessive disorder."

"There is a 50% chance of transmission of the disorder because it is an autosomal dominant disorder." Correct Explanation: An offspring of an autosomal dominant disorder has a 50% chance of acquiring the gene to be affected by the disorder. Huntington's is an autosomal dominant disorder. Female offspring of an X-linked recessive disorder have the possibility of being a carrier or of being afflicted with the disorder. With autosomal recessive disorders, there is only a 25% chance that the offspring will express the disorder.

A parent asks why a physical therapist is needed for the 6-month-old child diagnosed with Down syndrome. What is the best response by the nurse? a) "To optimize the child's development and functioning" b) "The earlier the intervention, the more likely we are to cure the problem." c) "To prevent contractures" d) "To ensure that the child meets all developmental milestones on time"

"To optimize the child's development and functioning" Correct Explanation: Interventional therapy is started early to promote the child's development and optimize functioning. The Down syndrome child usually meets developmental milestones at a slower pace. There is no cure for genetic disorders. Range-of-motion activities can prevent contractures; Down syndrome does not require physical therapy.

A nurse is teaching about autosomal dominant and recessive genetics. Which statement by the nurse is accurate? a) "Two abnormal genes, one from each parent, are required to produce the phenotype in an autosomal recessive disorder." b) "An autosomal dominant disorder is classified as X-linked." c) "One abnormal autosomal recessive gene is needed for outward presentation of the disorder." d) "An autosomal dominant disorder has a lower risk of phenotyping than an autosomal recessive disorder."

"Two abnormal genes, one from each parent, are required to produce the phenotype in an autosomal recessive disorder." Correct Explanation: An autosomal recessive disorder requires two abnormal genes to outwardly express the disorder. Recessive disorders have a lower risk of phenotyping than dominant disorders. X-linked and autosomal disorders are two different classifications.

A 16-year-old recently diagnosed with Marfan syndrome states, "I feel fine. Why do I need to have this testing done?" What is the best response by the nurse? a) "You want to live a long time, right?" b) "You are at risk of rupturing your aorta, and the echocardiogram will let us know if there are any problems." c) "This is routine. Nothing to worry about." d) "The lab work will let us know if you are developing diabetes as a complication."

"You are at risk of rupturing your aorta, and the echocardiogram will let us know if there are any problems." Correct Explanation: Marfan sydrome is a disorder that affects connective tissue. The aorta is susceptible to weakening because of the connective tissue disorder, leading to sudden death from aortic dissection. Diabetes is not a complication of Marfan. The other two choices offer no information and dismiss the teen's concerns.

The nurse is describing some of the developmental milestones the mother of a 3-month-old boy with Down syndrome can expect to see in her child. Which statement describes the milestones that are expected in a child with Down syndrome? a) "He will be speaking in sentences at 21 months of age." b) "Bladder training can be expected by 2.5 to 3 years of age." c) "You can expect him to eat with his hands by age 12 months." d) "He'll be crawling all over the house by 9 months of age."

"You can expect him to eat with his hands by age 12 months." Correct Explanation: Children with Down syndrome will accomplish eating with their hands by about 12 months of age. They will develop the skills of typical children, but at an older age. The child with Down syndrome will speak in sentences at 24 months rather than 21 months. Bladder training would occur by 48 months rather than 32 months. A child with Down syndrome will crawl at 11 months rather than 9 months.

A woman carries a recessive gene for sickle cell anemia. If her sexual partner also has this recessive gene, the chance that her first child will develop sickle cell anemia is a) 0 in 4. b) 2 in 4. c) 1 in 4. d) 3 in 4.

1 in 4. Correct Explanation: Autosomal recessive inherited diseases occur at a 1-in-4 incidence in offspring.

A woman who has a recessive gene for sickle cell anemia marries a man who also has a recessive gene for sickle cell anemia. Their first child is born with sickle cell anemia. The chance that their second child will develop this disease is a) 0 in 4. b) 3 in 4. c) 1 in 4. d) 2 in 4.

1 in 4. Explanation: Autosomal recessive inherited diseases occur at a 1-in-4 incidence in offspring. The possibility of a chance happening does not change for a second pregnancy.

A nurse enters a patient's room who has a suspected diagnosis of spinal muscular atrophy (SMA). Which finding by the nurse is most concerning? 1) The child's chest is visibly sunken inward. 2) The child appears to have an S-shaped sign. 3) The child has absent tendon reflexes. 4) The child has trouble initiating spontaneous movement.

1) Pectus excavatum can occur when the child breathes using the diaphragm without intercostal muscle support (paradoxical breathing). This causes the chest to funnel and the xiphoid process retracts, which further restricts respiratory development. Scoliosis, absent or diminished tendon reflexes, and diminished ability to initiate spontaneous movement are all expected in this patient. Page 816.

The nurse is caring for a child diagnosed with pectus excavatum post-op day 1 from a steel bar placement. Which finding by the nurse is most concerning? 1) Breath sounds are absent on the right side 2) The child is crying and says his chest hurts 3) The child is lying supine. 4) The child's mother is asking the child to roll to his right side and finish his homework.

1) This indicates a pneumothorax may have occurred as a post-operative complication, and this needs to be addressed immediately. The child should not roll to EITHER side for 4 weeks after surgery. However, we can encourage aerobic activity. Pain is expected after surgery.

The nurse has provided teaching to a family regarding clean intermittent catheterization for their 4 year-old daughter with myelomeningocele. Which statement, if made by the family, indicates that further teaching is required by the nurse? Select all that apply: 1) "When the catheter becomes stiff, we will soak it in warm water to soften it." 2) "We will store our catheter in a zip-top plastic bag." 3) "We will insert the catheter about 4-6 inches, or until we see urine flow." 4) "We will lubricate the catheter with a petroleum-based jelly before usage." 5) "We will clean our daughter's genitalia with a washcloth before catheterization."

1, 3, 4 When catheters become stiff, cloudy, rough, cracked, or damaged in general, they need to be thrown away. Catheters should be inserted approximately 2-3 inches for females and 4-6 inches for males, or until urine flow is noted. A water-based lubricator needs to be used. See page 809.

A nurse is preparing for the admission of an infant with a diagnosis of bronchiolitis caused by respiratory syncytial virus (RSV). Which interventions would the nurse include in the plan of care? Select all that apply. 1. Place the infant in a private room. 2. Ensure that the infant's head is in a flexed position. 3. Wear a mask at all times when in contact with the infant. 4. Place the infant in a tent that delivers warm humidified air. 5. Position the infant side-lying, with the head lower than the chest. 6. Ensure that nurses caring for the infant with RSV do not care for other high-risk children.

1,6

Any individual taking phenobarbital for a seizure disorder should be taught a) never to discontinue the drug abruptly. b) never to go swimming. c) to avoid foods containing caffeine. d) to brush his or her teeth four times a day.

A Phenobarbital should always be tapered, not stopped abruptly, or seizures from the child's dependency on the drug can result.

The mother of a 3-year-old child arrives at a clinic and tells the nurse that the child has been scratching the skin continuously and has developed a rash. The nurse assesses the child and suspects the presence of scabies. The nurse bases this suspicion on which finding noted on assessment of the child's skin? 1.Fine grayish red lines 2.Purple-colored lesions 3.Thick, honey-colored crusts 4.Clusters of fluid-filled vesicles

1. Fine grayish red lines Scabies is a parasitic skin disorder caused by an infestation of Sarcoptes scabiei (itch mite). Scabies appears as burrows or fine, grayish red, threadlike lines. They may be difficult to see if they are obscured by excoriation and inflammation. Purple-colored lesions may indicate various disorders, including systemic conditions. Thick, honey-colored crusts are characteristic of impetigo or secondary infection in eczema. Clusters of fluid-filled vesicles are seen in herpesvirus infection.

The clinic nurse is reviewing the health care provider's prescription for a child who has been diagnosed with scabies. Lindane has been prescribed for the child. The nurse questions the prescription if which is noted in the child's record? 1.The child is 18 months old. 2.The child is being bottle-fed. 3.A sibling is using lindane for the treatment of scabies. 4.The child has a history of frequent respiratory infections.

1. The child is 18 months old Lindane is a pediculicide product that may be prescribed to treat scabies. It is contraindicated for children younger than 2 years because they have more permeable skin, and high systemic absorption may occur, placing the children at risk for central nervous system toxicity and seizures. Lindane also is used with caution in children between the ages of 2 and 10 years. Siblings and other household members should be treated simultaneously

Question: Put the following events of a generalized epileptic seizure in correct order: Postictal period Prodromal period Clonic stage Tonic stage

2 4 3 1 A tonic-clonic seizure is characterized by the following events: 1) prodromal period, 2) tonic stage, 3) clonic stage, and 4) postictal period.

To prevent rupture of the sac in a patient born with myelomeningocele, which of the following treatments does the nurse anticipate? 1) Ultrasound 2) Surgical closure 3) Banding to form necrotic tissue 4) Insertion of drain to collect CSF leakage

2) Page 807.

A nurse admits a child who has a history cerebral palsy. Which assessment finding by the nurse is most concerning? 1) The mother reports the child had a seizure 5 hours ago. 2) The child has a fever of 100.3. 3) The child is standing on his toes. 4) The mother reports the child's twisting movements seem to have worsened since arriving at the clinic.

2) This fever could indicate aspiration pneumonia, and this needs to be investigated further immediately with questioning of coughing, respiratory difficulty, or sputum production. Seizures are common with cerebral palsy. The child's symptoms have likely worsened because of the stress due to a clinic visit. Standing on the toes or scooting on the back (instead of crawling on the abdomen) are both commonly seen in a patient with cerebral palsy.

The nurse is caring for a patient diagnosed with a meningocele. The nurse should perform all of the following actions except: 1) Documenting the presence of a sac protruding from the lower spinal column. 2) Documenting the presence of clear fluid draining from the meningocele. 3) Encouraging fluids hourly. 4) Measuring head circumference every shift.

2) This may indicate a CSF leak and should be reported

When assessing a child for developmental dysplasia of the hip, the nurse feels a "clunk" when the child's hip is abducted and relocated. Which of the following did the nurse perform? 1) Braxton's maneuver 2) Ortolani's sign 3) Trendelenburg's sign 4) Barlow's test

2) Barlow's is when the hip is adducted and dislocated.

While caring for a patient with a myelomeningocele, which of the following actions, if made by the pediatric nurse, is incorrect? Select all that apply: 1) The nurse reports the presence of clear fluid from the lesion. 2) The nurse places the child in supine position to prevent skin breakdown. 3) The nurse encourages that light-weight blankets be used in place of heavy blankets or coverings. 4) The nurse places a piece of plastic wrap below the meningocele. 5) The nurse moistens the sac with a saline-soaked piece of gauze.

2, 3 A warmer or isolette should be used in place of blankets, which may place too much pressure on the sac. Supine positioning should be avoided in patients with myelomeningocele, as this places excessive pressure on the spinal cord sac. Prone positioning is preferred.

A nurse prepares to care for a patient diagnosed with athetoid, or dyskinetic, cerebral palsy. Which of the following does the nurse expect to see? Select all that apply: 1) Hypertonicity of affected extremities 2) Drooling 3) Worsening of symptoms when the child gets stressed 4) Worm-like writhing 5) Exaggerated deep tendon reflexes

2, 3, 4 Others are characteristic of spastic cerebral palsy. The infant may also appear limp or flaccid with the face, neck, and tongue possibly affected.

The pediatric nurse knows that which of the following medications are commonly used for patients with cerebral palsy? Select all that apply: 1) Docusate sodium 2) Diazepam 3) Dantrolene sodium 4) Baclofen 5) Atropine

2, 3, 4, 5 Can also use botulinum toxin, but this is administered by a nurse practitioner or physician.

A nurse has just finished providing discharge teaching to the family of a child going home with a cast that was applied 30 minutes prior. Which statement by the family indicates that further teaching is necessary? Select all that apply: 1) "For the next couple of days, we should keep the casted arm above the level of the heart and apply ice." 2) "If my child's arm starts to itch, we can apply lotion if we can reach into the cast." 3) "We will tape a plastic bag around his cast before he takes a bath." 4) "If my child gets his cast wet, we'll blow dry it with the lowest heat setting on our blow dryer." 5) "We will make sure we regularly press the skin back around the edge of the cast." 6) "We will make sure our child does not eat anything messy while he has this cast on."

2, 4, 6 Nothing should be inserted into the cast, and lotions and powders should not be used. If the cast gets wet, a blow dryer with COLD air should be used. There is no need to adjust the child's diet, but the cast should be covered while the child eats or drinks. See page 844

The nurse caring for a child who sustained a burn injury plans care based on which pediatric considerations associated with this injury? Select all that apply. 1. Scarring is less severe in a child than in an adult. 2. A delay in growth may occur after a burn injury. 3. An immature immune system presents an increased risk of infection for infants and young children. 4. The lower proportion of body fluid to mass in a child increases the risk of cardiovascular problems. 5. Fluid resuscitation is unnecessary unless the burned area is more than 25% of the total body surface area. 6. Infants and young children are at increased risk for protein and calorie deficiency because they have smaller muscle mass and less body fat than adults.

2. A delay in growth may occur after a burn injury 3. An immature immune system presents an increased risk of infection for infants and young children. 6. Infants and young children are at increased risk for protein and calorie deficiency because they have smaller muscle mass and less body fat than adults. Pediatric considerations in the care of a burn victim include the following: Scarring is more severe in a child than in an adult. A delay in growth may occur after a burn injury. An immature immune system presents an increased risk of infection for infants and young children. The higher proportion of body fluid to mass in a child increases the risk of cardiovascular problems. Burns involving more than 10% of total body surface area require some form of fluid resuscitation. Infants and young children are at increased risk for protein and calorie deficiencies because they have smaller muscle mass and less body fat than adults.

The nurse is monitoring a child with burns during treatment for burn shock. The nurse understands that which assessment provides the most accurate guide to determine the adequacy of fluid resuscitation? 1.Skin turgor 2.Neurological assessment 3.Level of edema at burn site 4.Quality of peripheral pulses

2. Neurological assessment Sensorium is an accurate guide to determine the adequacy of fluid resuscitation. The burn injury itself does not affect the sensorium, so the child should be alert and oriented. Any alteration in sensorium should be evaluated further. A neurological assessment would determine the level of sensorium in the child. Options 1, 3, and 4 would not provide an accurate assessment of the adequacy of fluid resuscitation.

The nurse recognizes that which individual or couple would most benefit from obtaining genetic counseling? a) 30-year-old female with a normal alpha-fetoprotein screening b) 23-year-old female, 25-year-old-male, both with family history of sickle cell disorder c) 25-year-old female, 40-year-old male, both with no significant past medical history d) 32-year-old female, 25-year-old male with one pregnancy loss

23-year-old female, 25-year-old-male, both with family history of sickle cell disorder Correct Explanation: A family history of sickle cell disorder increases the risk of passing the disorder to offspring; genetic counseling would benefit this couple most. The usual standard for counseling for pregnancy loss is two or more, not a single loss. A normal alpha-fetoprotein screening is not a criterion for genetic counseling. All ages listed here do not exceed the criterion for advanced maternal or paternal age.

While caring for a 9-year-old female in Buck's traction, which of the following actions by the nurse is correct? 1) The nurse encourages the child's 3 year-old sibling to sit on the bed and visit with the child. 2) The nurse helps the child learn how to raise and lower the head of her bed so she can complete her homework. 3) The nurse checks the capillary refill on the child's extremities every 4 hours. 4) The nurse teaches the child's mother to place the weights on the bedside table before the child uses the bedpan.

3) Extra visitors should not be invited on the bed- especially a toddler who may think the weights at the end of the bed are toys. The head of the bed should only be raised or lowered with physician's orders, and this should be done minimally. The weights should ALWAYS be hanging freely.

The school nurse has provided an instructional session about impetigo to parents of the children attending the school. Which statement, if made by a parent, indicates a need for further instruction? 1."It is extremely contagious." 2."It is most common in humid weather." 3."Lesions most often are located on the arms and chest." 4."It might show up in an area of broken skin, such as an insect bite."

3. Lesions most often are located on the arms and chest." Impetigo is a contagious bacterial infection of the skin caused by b-hemolytic streptococci or staphylococci, or both. Impetigo is most common during hot, humid summer months. Impetigo may begin in an area of broken skin, such as an insect bite or atopic dermatitis. Impetigo is extremely contagious. Lesions usually are located around the mouth and nose, but may be present on the hands and extremities.

Which diagnostic exam does the nurse know will best aid in the diagnosis of Duchenne muscular dystrophy? 1) EEG 2) CT Scan 3) MRI 4) EMG

4)

The day shift pediatric nurse receives report on the following 4 patients. After receiving report, which patient should the nurse assess first? 1) The 4-year-old female with cerebral palsy who admits with difficulty swallowing and a fever of 101.9. 2) The 9-month-old male with myelomeningocele who has absent deep tendon reflexes in bilateral lower extremities. 3) The 6-year-old male with Duchenne muscular dystrophy who appears sad and withdrawn. 4) The 7-year-old female with a spinal cord injury who reports numbness and tingling in her feet.

4) Spinal cord injury is a medical emergency that requires immediate assessment. A fever and difficulty swallowing in a child with cerebral palsy may indicate aspiration pneumonia, so this child should be seen next. Absent deep tendon reflexes are expected in a child with myelomeningocele, related to the paralysis seen below the sac. A child with Duchenne muscular dystrophy may be sad or withdrawn due to corticosteroid medication side effects or chronicity of the disease.

A concerned mother calls the and tells you the following pieces of information about her 2-year-old son. Which statement by the mother most concerns you? 1) "I noticed that when my son is standing, his knees touch but his feet seem really far apart." 2) "My son's feet are so flat, even though he's been waking for 9 months now." 3) "My baby really hates it when I try to feed him broccoli and keeps spitting it out onto his plate." 4) "This morning when I was trying to dress him, my son cried nonstop when I tried to put his shirt on."

4) This could indicate a sprain or fracture, and nursemaid's elbow is common in toddlers and preschoolers. This statement should be further investigated since it indicates pain/discomfort. It is not uncommon for infants to have flat feet, although the arch of the foot should begin to form after walking begins. However, some infants never develop an arch and have flat feet as adults. Genu valgum, or "knock knees", are commonly seen around the ages 2-3, and this will often resolve by ages 7-8. It is not uncommon for a toddler to dislike broccoli. Who does like broccoli.

The mother of a 3 month old infant calls the clinic and says "why can't my baby hold his head up? Is he developmentally delayed?" Which response by the nurse is best? 1) "Your child is unlikely to be developmentally delayed, but we would like you to come to the clinic for a well-baby check-up." 2) "Infants do not typically hold their heads up on their own before age 6 months, so you have nothing to worry about." 3) "You sound very concerned. This may indicate a developmental delay, but we would like you to come to the clinic for a well-baby check-up and testing." 4) "Your infant may not hold his head up for a couple more months. This is nothing to be concerned about."

4) This isn't an ideal answer, and could be worded more therapeutically, but out of the four options, this is the BEST answer. Infants lose head lag around 4-5 months.

A nurse analyzes the lab values of a child with leukemia who is receiving chemotherapy. The nurse notices that the platelet count is 19,500 cell/mm3. Based on this lab value which intervention would the nurse document in her plan of care. " "1. Monitor closely for signs of infection. 2. Temp every four hours. 3. Isolation precautions 4. Use a small toothbrush for mouth care"

4. **Correct... Rationale: Leukemia is a malignant increase in the number of leukocytes, usually at an immature stage, in the bone marrow. It affects the bone marrow, causing from decreased erythrocytes, infection from neutropenia, and bleeding from decreased platelet production. If the platelet count is les than 20,000 than bleeding precautions need to be taken.

A topical corticosteroid is prescribed by a health care provider for a child with atopic dermatitis (eczema). Which instruction should the nurse give the parent about applying the cream? 1.Apply the cream over the entire body. 2.Apply a thick layer of cream to affected areas only. 3.Avoid cleansing the area before application of the cream. 4.Apply a thin layer of cream and rub it into the area thoroughly.

4. Apply a thin layer of cream and rub it into the area thoroughly. Atopic dermatitis is a superficial inflammatory process involving primarily the epidermis. A topical corticosteroid may be prescribed and should be applied sparingly (thin layer) and rubbed into the area thoroughly. The affected area should be cleaned gently before application. A topical corticosteroid should not be applied over extensive areas. Systemic absorption is more likely to occur with extensive application.

Permethrin (Elimite) is prescribed for a child with a diagnosis of scabies. The nurse should give which instruction to the parents regarding the use of this treatment? 1.Apply the lotion to areas of the rash only. 2.Apply the lotion and leave it on for 6 hours. 3.Avoid putting clothes on the child over the lotion. 4.Apply the lotion to cool, dry skin at least 30 minutes after bathing.

4. Apply the lotion to cool, dry skin at least 30 minutes after bathing. Permethrin is massaged thoroughly and gently into all skin surfaces (not just the areas that have the rash) from the head to the soles of the feet. Care should be taken to avoid contact with the eyes. The lotion should not be applied until at least 30 minutes after bathing and should be applied only to cool, dry skin. The lotion should be kept on for 8 to 14 hours, and then the child should be given a bath. The child should be clothed during the 8 to 14 hours of treatment contact time.

The school nurse is conducting pediculosis capitis (head lice) assessments. Which finding indicates a child has a "positive" head check? 1.Maculopapular lesions behind the ears 2.Lesions in the scalp that extend to the hairline or neck 3.White flaky particles throughout the entire scalp region 4.White sacs attached to the hair shafts in the occipital area

4. White sacs attached to the hair shafts in the occipital area Pediculosis capitis is an infestation of the hair and scalp with lice. The nits are visible and attached firmly to the hair shaft near the scalp. The occiput is an area in which nits can be seen. Maculopapular lesions behind the ears or lesions that extend to the hairline or neck are indicative of an infectious process, not pediculosis. White flaky particles are indicative of dandruff.

A nurse is counseling a couple who have a 5-year-old daughter with Down syndrome. The nurse recognizes that their daughter's genome is represented by which of the following? a) 47XY21+ b) 46XX5p- c) 46XX d) 47XX21+

47XX21+ Correct Explanation: In Down syndrome, the person has an extra chromosome 21, so this is abbreviated as 47XX21+ (for a female) or 47XY21+ (for a male). 46XX is a normal genome for a female. The abbreviation 46XX5p- is the abbreviation for a female with 46 total chromosomes but with the short arm of chromosome 5 missing (Cri-du-chat syndrome).

A 25-year-old woman who recently underwent genetic testing has just learned that she is heterozygous dominant for Huntington disease. Her husband, however, who also underwent the testing, is free from the trait. What are the odds that the couple will have a child who will inherit the disorder? a) 25% b) 50% c) 75% d) 100%

50% Correct Explanation: If a person who is heterozygous or has a dominant illness gene opposing a recessive healthy gene mates with a person who is free of the trait, the chances are even (50%) a child born to the couple would have the disorder or would be disease and carrier free (that is, carrying no affected gene for the disorder).

The eyes of a 9-year-old who suffered a head injury are crossed. Besides checking ICP, which of the following interventions would be most important for the nurse to perform? a) Assess the child's level of consciousness. b) Help the child cope with an altered appearance. c) Monitor core body temperature. d) Pull up the side rails on the bed

A Decreased level of consciousness is frequently the first sign of major neurologic problems after head trauma. While body temperature is an important indicator of infection, it is not a priority here. Preventing harm by setting the side rails is more important for a seizure patient. The child's eyes will correct themselves when ICP is reduced.

A pregnant client asks if there is any danger to the development of her fetus in the first few weeks of her pregnancy. How should the nurse respond? a) "During the first 3 to 4 weeks of pregnancy brain and spinal cord development occur and are affected by nutrition, drugs, infection, or trauma." b) "As long as you were taking good care of your health before becoming pregnant, your fetus should be fine during the first few weeks of pregnancy." c) "The respiratory system matures during this time so good prenatal care during the first weeks of pregnancy is very important." d) "Bones begin to harden in the first 5 to 6 weeks of pregnancy so vitamin D consumption is particularly important."

A Brain and spinal cord development occur during the first 3 to 4 weeks of gestation. Infection, trauma, teratogens (any environmental substance that can cause physical defects in the developing embryo and fetus), and malnutrition during this period can result in malformations in brain and spinal cord development and may affect normal central nervous system (CNS) development. Good health before becoming pregnant is important but must continue into the pregnancy. Hardening of bones occurs during 13 to 16 weeks gestation, and the respiratory system begins maturing around 23 weeks' gestation.

The nurse is observing an infant who may have acute bacterial meningitis. Which finding might the nurse look for? a) Irritability, fever, and vomiting b) Jaundice, drowsiness, and refusal to eat c) Negative Kernig's sign d) Flat fontanel

A Findings associated with acute bacterial meningitis may include irritability, fever, and vomiting along with seizure activity. Fontanels would be bulging as intracranial pressure rises, and Kernig's sign would be present due to meningeal irritation. Jaundice, drowsiness, and refusal to eat indicate a GI disturbance rather than meningitis

In caring for a child with a seizure disorder, the highest priority goal is which of the following? a) The child will be free from injury during a seizure. b) The child will have an understanding of the disorder. c) The family will understand seizure precautions. d) The family caregivers anxiety will be reduced.

A Keeping the child free from injury is the highest priority goal. The other choices are important, but keeping the child safe is higher than the anxiety or knowledge deficit concerns. The physical always is a priority over the psychological.

A mother has brought her 5-month-old son to the clinic because he has been drowsy and unresponsive. The child has hydrocephalus and had a shunt placed about a month previously. Which of the following symptoms indicate that the shunt is infected? a) The child is not responding or eating well. b) The fontanels are bulging or tense. c) The child's pupil reaction time is rapid and uneven. d) The child has a high-pitched cry.

A Poor feeding and decreased responsiveness are signs of an infection. The nurse might also observe localized inflammation along the shunt tract. Bulging or tense fontanels suggest a shunt malfunction that is causing increased intracranial pressure. A high-pitched cry suggests increased intracranial pressure due to a shunt malfunction. Decreased and uneven pupil reaction times are symptoms of a shunt malfunction that is causing increased intracranial pressure.

A 6-month-old infant is admitted with a diagnosis of bacterial meningitis. The nurse would place the infant in which room? a) A private room near the nurses' station b) A two-bed room in the middle of the hall c) A room with a 12-month-old infant with a urinary tract infection d) A room with an 8-month-old infant with failure to thrive

A A child who has the diagnosis of bacterial meningitis will need to be placed in a private room until that child has received I.V. antibiotics for 24 hours because the child is considered contagious. Additionally, bacterial meningitis can be quite serious; therefore, the child should be placed near the nurses' station for close monitoring and easier access in case of a crisis.

The nurse knows that the heads of infants and toddlers are large in proportion to their bodies, placing them at risk for which of the following? a) Head trauma b) Positional plagiocephaly c) Congenital hydrocephalus d) Intracranial hemorrhaging

A A larger head size in relation to the rest of their body size gives young children a higher center of gravity, which causes them to hit their head more readily, thus placing them at risk for head trauma. Fragile capillaries in the periventricular area of the brain put preterm infants at risk for intracranial hemorrhage. Congenital hydrocephalus may be caused by abnormal intrauterine development or infection. Positional plagiocephaly is caused by an infant's head remaining in the same position for too long.

The nurse caring for a patient with a cranial injury knows that broad-spectrum antibiotics are used to reduce cerebral edema. a) False b) True

A Antibiotics or antivirals are used to treat infectious disease processes. Glucocorticoids and diuretics are used to reduce cerebral edema.

An otherwise healthy 18-month-old child with a history of febrile seizures is in the wellchild clinic. Which statement by the father would indicate to the nurse that additional teaching should be done? a) "I always keep phenobarbital with me in case of a fever." b) "The most likely time for a seizure is when the fever is rising." c) "I have ibuprofen available in case it's needed." d) "My child will likely outgrow these seizures by age 5."

A Anticonvulsants, such as phenobarbital, are administered to children with prolonged seizures or neurologic abnormalities. Ibuprofen, not phenobarbital, is given for fever. Febrile seizures usually occur after age 6 months and are unusual after age 5. Treatment is to decrease the temperature because seizures occur as the temperature rises.

Which of the following is most correct regarding the nervous system of the child? a) As the child grows, the gross and fine motor skills increase. b) The child has underdeveloped gross motor skills and well-developed fine motor skills. c) The child's nervous system is fully developed at birth. d) The child has underdeveloped fine motor skills and well-developed gross motor skills.

A As the child grows, the quality of the nerve impulses sent through the nervous system develops and matures. As these nerve impulses become more mature, the child's gross and fine motor skills increase in complexity. The child becomes more coordinated and able to develop motor skills.

A child is diagnosed with aseptic meningitis. The child's mother states, "I don't know where she would have picked this up." The nurse prepares to respond to the mother, based on the understanding that this disorder is most likely caused by which of the following? a) Enterovirus b) Escherichia coli c) Streptococcus group B d) Haemophilus influenza type B

A Aseptic meningitis is the most common type of meningitis, and if a causative organism can be identified, it is usually a virus such as enterovirus. E. coli is a cause of bacterial meningitis. H. influenza type B is a cause of bacterial meningitis. Streptococcus group B is a cause of bacterial meningitis.

In caring for a child with a seizure disorder, the highest priority goal is which of the following? a) The child will be free from injury during a seizure. b) The family caregivers anxiety will be reduced. c) The family will understand seizure precautions. d) The child will have an understanding of the disorder.

A Keeping the child free from injury is the highest priority goal. The other choices are important, but keeping the child safe is higher than the anxiety or knowledge deficit concerns. The physical always is a priority over the psychological.

A child is home with the caregivers following a treatment for a head injury. If the child makes which of the following statements, the caregiver should contact the care provider. a) "You look funny. Well, both of you do. I see two of you." b) "It will be nice when you will let me take a long nap. I am sleepy." c) "My stomach is upset. I feel like I might throw up." d) "I am glad that my headache is getting better."

A The caregiver should notify the health-care provider immediately if the child vomits more than three times, has pupillary changes, has double or blurred vision, has a change in level of consciousness, acts strange or confused, has trouble walking, or has a headache that becomes more severe or wakes him or her from sleep. These instructions should be provided in written form to the caregiver. Just feeling naueauted is not a reason to notify the provider.

In understanding the nervous system, the nurse recognizes that the central nervous system is made up of which of the following? a) The brain and spinal cord b) Fluid that flows through the brain c) Nerves throughout the upper body d) A protective cushion for nerve cells

A The central nervous system is made up of the brain and spinal cord. The peripheral nervous system is made up of the nerves throughout the body. A fluid known as cerebrospinal fluid (CSF) flows through the chambers of the brain and through the spinal cord, serving as a cushion and protective mechanism for nerve cells.

To detect complications as early as possible in a child with meningitis who's receiving I.V. fluids, monitoring for which condition should be the nurse's priority? a) Cerebral edema b) Cardiogenic shock c) Renal failure d) Left-sided heart failure

A The child with meningitis is already at increased risk for cerebral edema and increased intracranial pressure due to inflammation of the meningeal membranes; therefore, the nurse should carefully monitor fluid intake and output to avoid fluid volume overload. Renal failure and cardiogenic shock aren't complications of I.V. therapy. The child with a healthy heart wouldn't be expected to develop left-sided heart failure.

The nurse is reinforcing teaching with the caregivers of a child who has had a head injury and is being discharged. Which of the following statements made by the caregiver indicate an accurate understanding of the follow-up care for their child? a) "Even if the flashlight bothers him, we will check his eyes." b) "If he vomits again, we will bring him back immediately." c) "We can give him Tylenol for a headache, but no aspirin." d) "If he falls asleep, we will wake him up every 15 minutes."A

A The child's pupils are checked for reaction to light every four hours for 48 hours. If the child falls asleep, he or she should be awakened every one to two hours to determine that the level of consciousness has not changed. No analgesics or sedatives should be administered during this period of observation. The caregiver should observe the child for at least six hours for vomiting or a change in the child's level of consciousness. The caregiver should notify the health-care provider immediately if the child vomits more than three times, but if the child vomits once, returning to the care provider immediately is not needed.

Which of the following age groups of children have the highest actual rate of death from drowning? a) Toddlers b) School-age children c) Preschool children d) Infants

A Toddlers and older adolescents have the highest actual rate of death from drowning.

A group of nursing students are reviewing cerebral vascular disorders and risk factors in children. The students demonstrate understanding of the material when they identify which of the following as a risk factor for hemorrhagic stroke? a) Arteriovenous malformations (AVMs) b) Sickle cell disease c) Meningitis d) Congenital heart defect

A Vascular malformations such as intracranial AVMs are a risk factor for hemorrhagic stroke. Sickle cell disease is a risk factor for ischemic stroke. Congenital heart defects are risk factors for ischemic stroke. Meningitis or other infection is a risk factor for ischemic stroke.

A client diagnosed with chronic cirrhosis who has ascites and pitting peripheral edema also has hepatic encephalopathy. Which of the following nursing interventions are appropriate to prevent skin breakdown? Select all that apply. A. Alternating air pressure mattress B. Turn and reposition every 2 hours C. Abdominal and foot massages every 2 hours D. Sit in chair for 30 minutes each shift E. Range of motion every 4 hours

A Alternating air pressure mattress B Turn and reposition every 2 hours

When caring for a child who has a history of seizures, which of the following nursing interventions would be appropriate? (Select all that apply) a) The nurse has oxygen available to use during a seizure. b) The nurse positions the child on the side during a seizure. c) The nurse places a washcloth in the mouth to prevent injury during seizure. d) The nurse goes for help as soon as a seizure begins. e) The nurse pads the crib or side rails before a seizure. f) The nurse teaches the caregivers regarding seizure precautions.

A B E F Pad the crib sides and keep sharp or hard items out of the crib. Position the child to one side to prevent aspiration of saliva or vomitus. Have oxygen and suction equipment readily available for emergency use. Teach family caregivers seizure precautions so they can handle a seizure that occurs at home. Do not put anything in the child's mouth; doing so could cause injury to the child or to you. Stay with the patient

Choice Multiple question - Select all answer choices that apply. A nurse is providing information to the parents of a child diagnosed with absence seizures. Which of the following would the nurse expect to include when describing this type of seizure? Select all that apply. a) This type of seizure is usually short, lasting usually for no more than 30 seconds. b) The child will commonly report a strange odor or sensation before the seizure. c) You might see a blank facial expression after a sudden stoppage of speech. d) Your child will probably sleep deeply for ½ to 2 hours after the seizure. e) You might have mistaken this type of seizure for lack of attention. f) This type of seizure is more common in girls than it is in boys.

A C E F Absence seizures are more common in girls than boys and are characterized by a sudden cessation of motor activity or speech with a blank facial expression or rhythmic twitching of the mouth or blinking of the eyelids. This type of seizure lasts less than 30 seconds and may have been mistaken for inattentiveness because of the subtle changes. Absence seizures are not associated with a postictal state.

The nurse and an adolescent are reviewing the adolescent's record of her headaches and activities surrounding them. Which of the following would the nurse identify as a possible trigger? a) Drinking three cans of diet cola b) Swimming twice a week c) 11 p.m. bedtime; 6:30 a.m. wake-up d) Use of nonscented soap

A Cola contains caffeine, which is an associated trigger. Intense activity, not regular exercise, may be a trigger. Odors, such as strong perfumes, may be a trigger. Changes in sleeping patterns may be a trigger.

Choice Multiple question - Select all answer choices that apply. The nurse determines that a child is experiencing late signs of increased intracranial pressure based on assessment of which of the following? Select all that apply. a) Fixed dilated pupils b) Increased blood pressure c) Irregular respirations d) Sunset eyes e) Bradycardia

A E C Late signs of increased intracranial pressure include bradycardia, fixed and dilated pupils, and irregular respirations. Increased blood pressure and sunset eyes are early signs of increased intracranial pressure.

A 9-year-old girl who is suspected of having an infection of the central nervous system is undergoing a lumbar puncture to withdraw cerebrospinal fluid for analysis. The nurse knows that the needle will be introduced into the subarachnoid space at the level of which of the following vertebrae? a) L4 or L5 b) L1 or L2 c) T3 or T4 d) C1 or C2

A Lumbar puncture, the introduction of a needle into the subarachnoid space (under the arachnoid membrane) at the level of L4 or L5 to withdraw CSF for analysis, is used most frequently with children to diagnose hemorrhage or infection in the CNS or to diagnose an obstruction of CSF flow.

The nurse suspects that which client has the highest risk for breast cancer? A. Older adult woman with high breast density B. Nullipara older adult woman C. Obese older adult male with gynecomastia D. Middle-aged woman with high breast density

A Older adult woman with high breast density. People at high increased risk for breast cancer include women aged 65 years and older with high breast density.

A child is home with the caregivers following a treatment for a head injury. If the child makes which of the following statements, the caregiver should contact the care provider. a) "You look funny. Well, both of you do. I see two of you." b) "I am glad that my headache is getting better." c) "It will be nice when you will let me take a long nap. I am sleepy." d) "My stomach is upset. I feel like I might throw up."

A The caregiver should notify the health-care provider immediately if the child vomits more than three times, has pupillary changes, has double or blurred vision, has a change in level of consciousness, acts strange or confused, has trouble walking, or has a headache that becomes more severe or wakes him or her from sleep. These instructions should be provided in written form to the caregiver. Just feeling naueauted is not a reason to notify the provider.

A child has been diagnosed with a basilar skull fracture. The nurse identifies ecchymosis behind the patient's ear. This would be documented as which of the following? a) Battle sign b) Rhinorrhea c) Otorrhea d) Raccoon eyes

A Two signs of basilar skull fracture include Battle sign (bruising or ecchymosis behind the ear) and "raccoon eyes" (blood leaking into the frontal sinuses causing an edematous and bruised periorbital area). Rhinorrhea is CSF leakage from the nose. Otorrhea is CSF leaking from the ear.

You prepare a couple to have a karyotype performed. Which of the following describes a karyotype? a) The gene carried on the X or Y chromosome b) A visual presentation of the chromosome pattern of an individual c) The dominant gene that will exert influence over a correspondingly located recessive gene d) A blood test that will reveal an individual's homozygous tendencies

A visual presentation of the chromosome pattern of an individual Correct Explanation: A karyotype is a photograph of a person's chromosomes aligned in order.

An 8-week-old infant is taken to the ER not breathing; which is a finding of Sudden Infant Death Syndrome (SIDS) is made to the parents. Which intervention should the nurse take first? A) Allow the parents to see their infant B) Gather the infant's belongings and give them to the parents C) Explain the etiology of SIDS D) Call their spiritual advisor

A) Allow the parents to see their infant

Which cell is the site of origin for Non-Hodgkin Lymphoma? A) B or T lymphocyte B) Erythrocyte C) Epithelial cell D) Monocyte

A) B or T lymphocyte

For immunocompromised children with Herpes Zoster (Shingles), acyclovir is given B) Never, due to Reyes Syndrome C) Intramuscular D) Sublingual

A) Intravenously

SELECT ALL THAT APPLY What relieves itching for a child who has Herpes zoster (Shingles)? A) Oatmeal Baths B) Antihistamines C) Aspirin D) Epsin Salt Baths

A) Oatmeal Baths B) Antihistamines

The client with prostate cancer asks the nurse for more information and counseling. Which resources will the nurse suggest? Select all that apply. A. American Cancer Society's Man to Man program. B. Us TOO International. C. American Prostate Cancer Society. D. National Prostate Cancer Coalition. E. The client's church, synagogue, or place of worship.

A,B,D,E: American Cancer Society's Man to Man program helps the client and partner cope with prostate cancer. This program provides one-on-one education, personal visits, educations presentations, and the opportunity to engage in open and candid discussions. Us TOO International is a prostate cancer support group that is sponsored by the Prostate Cancer Education and Support Network. The National Prostate Cancer Coalition provides prostate cancer information.The client's church, synagogue or place of worship is a community support service that may be important for many clients.

The nurse includes which of the following in teaching regarding the warning signs of cancer? Select all that apply. A. Persistent constipation B. Scab present for 6 months C. Curdlike vaginal discharge D. Axillary swelling E. Headache

A,B,D: Change in bowel habit, A sore that does not heal, A lump or thickening in the breast or elsewhere is a warning signal of cancer.

The nurse has received in report that the client receiving chemotherapy has severe neutropenia. Which of the following does the nurse plan to implement? Select all that apply. A. Assess for fever. B. Observe for bleeding. C. Administer pegfilgrastim (Neulasta). D. Do not permit fresh flowers or plants in the room. E. Do not allow his 16-year-old son to visit. F. Teach the client to omit raw fruits and vegetables from his diet.

A,C,D,F: Any temperature elevation in a client with neutropenia is considered a sign of infection and should be reported immediately. Administration of biological response modifiers, such as filgrastim (Neupogen) and pegfilgrastim (Neulasta), is indicated in neutropenia to prevent infection and sepsis. All fruits and vegetables should be cooked well; raw fruits and vegetables may harbor organisms, as well as Flowers and plants. Thrombocytopenia cause bleeding, not low neutrophils.The client is at risk for infection, not the visitors, if they are well. However, very small children, who may get frequent colds and viral infections, may pose a risk.

Which signs or symptoms should the nurse report immediately because they indicate thrombocytopenia secondary to cancer chemotherapy? Select all that apply. A. Bruises B. Fever C. Petechiae D. Epistaxis E. Pallor

A,C,D: Fever is a sign of infection secondary to neutropenia.Pallor is a sign of anemia.

The client with benign prostatic hyperplasia (BPH) is being discharged with alpha-adrenergic blockers. Which information is important for the nurse to include when teaching the client about this type of pharmacologic management? Select all that apply. A. Avoid drugs used to treat erection problems. B. Be careful when changing positions. C. Keep all appointments for follow-up laboratory testing. D. Hearing tests will need to be conducted periodically. E. Take the medication in the afternoon.

A-C: Drugs used to treat erectile dysfunction can worsen side effects, such as hypotension. Alpha-adrenergic blockers may cause orthostatic hypotension, can cause liver damage, do not affect hearing and should be taken in the evening to decrease the risk of problems related to hypotension.

You are going over insulin administration education with a patient's mother. Which statement by her raises concern? A. "When she is sick I will hold her insulin." B."I always carry sugary items in case she has a hypoglemic attack." C. "I will bring her in every 3 months for a glycosylate hemoglobin blood drawn." D. "I ordered her a Medic-Alert bracelet yesterday."

A. When a diabetic is sick, they should never hold their insulin. This is because when the body is stressed or has an infection they are at a very high risk for hyperglycemia so it is essential they monitor their blood glucose closely and administered insulin as needed. All the other options are correct.

As the nurse, taking care of the patient who has been hospitalized for 3 days with dehydration, what abnormal finding would you report to the MD? A. Weight change of 100 lbs to 92 lbs and urinary output of less than 1 ml/kg/hr B. 1-3 second skin turgor C. Weight change of 90 lbs to 93 lbs and dry mucous membranes D. Options A & C

A. The only correct option is: Weight change of 100 lbs to 92 lbs and urinary output of less than 1 ml/kg/hr. All the other answers are normal findings that do not cause concern.

The parents of a pediatric client who has sickle cell anemia ask about the cause of the disorder. Which response by the nurse would best describe the cause? A. "It is caused by a recessive trait the primarily affects African-Americans." B. "It is a rare, malignant disorder of the lymphatic system." C. "It is an inherited disorder caused by the abnormal hemoglobin synthesis." D. "It is caused by an increased demand for iron in the blood stream."

A. "It is caused by a recessive trait the primarily affects African-Americans."

A client that has just been diagnosed with Hodgkin's disease asks the nurse, "How did this happen to me?" What is the correct response by the nurse? A. "The cause of Hodgkin's disease is unknown, but it could be a possibility of having a viral illness, such as mono." B. "That doesn't matter." C. "Hodgkin's disease is caused by trauma to the body." D. "This disease is caused by immunizations."

A. "The cause of Hodgkin's disease is unknown, but it could be a possibility of having a viral illness, such as mono."

Previous administrations of chemotherapy agents to a cancer patient have resulted in diarrhea. Which of the following dietary modifications should the nurse recommend? A. A bland, low-fiber diet B. A high-protein, high-calorie diet C. A diet high in fresh fruits and vegetables D. A diet emphasizing whole and organic foods

A. A bland, low fiber diet Patients experiencing diarrhea secondary to chemotherapy and/or radiation therapy often benefit from a diet low in seasonings and roughage. Fresh fruits and vegetables are high in fiber and should be minimized during treatment. Whole and organic foods do not prevent diarrhea.

A nursing student studies infectious mononucleosis. When the student assesses a patient who has this illness he/she remembers mononucleosis is usually caused by: A. A herpesvirus infection B. A bacterial infection C. A retrovirus infection D. A fungal infection

A. A herpesvirus infection

What is the best way for the nurse to detect fluid retention in a child with nephrotic syndrome who has not yet been toilet-trained? A. Weigh the child daily. B. Check the urine for blood. C. Measure the abdominal girth weekly. D. Count the number of wet diapers.

A. Weigh the child daily.

The nurse is teaching the 47-year-old female client about recommended screening practices for breast cancer. Which statement by the client indicates understanding of the nurse's instructions? A. "My mother and grandmother had breast cancer, so I am at risk." B. "I get a mammography every 2 years since I turned 30." C. "A clinical breast examination is performed every month since I turned 40." D. "A CT scan will be done every year after I turn 50."

A. A strong family history of breast cancer indicates a risk for breast cancer. Annual screening may be indicated for a strong family history. The client may perform a self-breast examination monthly; a clinical examination by a health care provider is indicated annually. An annual mammography is performed after age 40 or in younger clients with a strong family history.

Substance known to disrupt normal hormone-related growth and development of children include all the following except: A. Aloin B. Dioxins C. Oral contraceptives D. Coumestans derived from alfalfa

A. Aloin Aloin is the one of the active compounds found in aloe vera gel, and has not been found to affect childhood growth and development. Dioxins are toxic byproducts of industrial processes. Oral contraceptives contain female sex hormones that can alter normal sexual development in children based on their time of exposure. Coumestans, while natural compounds found in some plants, are considered phytoestrogens, which have been shown to affect endocrine function.

The client has undergone transurethral resection of the prostate (TURP). Which intervention will the nurse incorporate in this client's postoperative care? A. Administer antispasmodic medications. B. Encourage the client to urinate around the catheter if pressure is felt. C. Perform intermittent urinary catheterization every 4 to 6 hours. D. Place the client in a supine position, with his knees flexed.

A. Antispasmodic drugs can be administered to decrease the bladder spasms that may occur after TURP.

Which or the following nursing interventions should be implemented to manage a client with appendicitis? B. Assessing for pain C. Encourage oral intake of clear fluids D. Providing discharge teaching

A. Assessing for symptoms of peritonitis

Sudden Infant Death Syndrome (SIDS) is confirmed by which procedures? A. Autopsy B. Laboratory analysis C. Chest X-Ray D. MRI

A. Autopsy

The client is struggling with body image after breast cancer surgery. Which behavior indicates to the nurse that the client is maladaptive? A. Avoiding eye contact with staff B. Saying, "I feel like less of a woman" C. Requesting a temporary prosthesis immediately D. Saying, "This is the ugliest scar ever"

A. Avoiding eye contact may be an indication of decreased self-image.

A nurse is caring for a child who has cerebral palsy. Which of the following medications should the nurse expect to administer to treat painful muscle spasms? Select all that apply. A. Baclofen (Lioresal) B. Diazepam (Valium) C. Oxybutynin chloride (Ditropan) D. Methotrexate (Rheumatrex) E. Prednisone (Deltasone)

A. Baclofen (Lioresal) B. Diazepam (Valium)

The registered nurse is teaching a nursing student about the importance of observing for bone marrow suppression during chemotherapy. Select the person who displays bone marrow suppression. A. Client with hemoglobin of 7.4 and hematocrit of 21.8 B. Client with diarrhea and potassium level of 2.9 mEq/L C. Client with 250,000 platelets D. Client with 5000 white blood cells/mm3

A. Bone marrow suppression causes anemia, leukopenia, and thrombocytopenia; this client has anemia demonstrated by low hemoglobin and hematocrit.

When caring for a client with cachexia, the nurse expects to note which symptom? A. Weight loss B. Anemia C. Bleeding tendencies D. Motor deficits

A. Cachexia results in extreme body wasting and malnutrition. Severe weight loss is expected.

HF is due to : A. Congenital cardiac defect B. Abnormal mechanical contractions C. Endocarditis D. Volume overload

A. Congenital cardiac defect C. Endocarditis D. Volume overload

A diagnostic workup is being performed on a 1-year-old child with suspected neuroblastoma. The nurse reviews the results of the diagnostic tests and understands that which of the following findings is most specifically related to this type of tumor? A. Elevated vanillylmandelic acid (VMA) urinary levels B. Presence of blast cells in the bone marrow C. Projectile vomiting, usually in the morning D. Postive Babinski's sign"

A. Elevated vanillylmandelic acid (VMA) urinary levels

A client newly diagnosed with acute leukemia asks why he is at such extreme risk for infection when his white blood cell count is so high. What is the nurse's best response? A. "Even though you have many white blood cells, they are too immature to fight infection." B. "For now, your risk is low; however, when the chemotherapy begins, your risk for infection will be high." C. "These white blood cells are cancerous and live longer than normal white blood cells, so they are too old to fight infection." D. "It is not the white blood cells that provide protection; it is the red blood cells, which are very low in your blood right now."

A. For clients who understand that white blood cells are a great protection against infection, being at great risk for infection even when WBC counts are sometimes ten times normal is confusing. These are leukemic cells that overgrow at a very immature level. Therefore even though there can be huge numbers of circulating WBCs, these cells are so immature that they are nonfunctional. In addition, the heavy production of immature leukemic cells prevents normal WBCs, RBCs, and platelets from forming and maturing into functional cells.

The nurse explains to the client that which risk factor most likely contributed to his primary liver carcinoma? A. Infection with hepatitis B virus B. Consuming a diet high in animal fat C. Exposure to radon D. Familial polyposis

A. Hepatitis B and C are risk factors for primary liver cancer. Consuming a diet high in animal fat may predispose to colon or breast cancer. Exposure to radon is a risk factor for lung cancer. Familial polyposis is a risk factor for colorectal cancer.

In addition to presenting symptoms, which laboratory finding indicates nephrosis? A. Hypoalbuminemia B. Low specific gravity C. Decreased hemoglobin level D. Decreased hematocrit

A. Hypoalbuminemia

The client is receiving chemotherapy treatment for breast cancer and asks for additional support for managing the associated nausea and vomiting. Which complementary therapy will the nurse suggest? A. Ginger B. Journaling C. Meditation D. Yoga

A. It has long been believed that ginger helps alleviate nausea and vomiting. Current studies are being done on the effect of ginger on chemotherapy-induced nausea.

The client who has had a mastectomy asks the nurse about breast reconstructive surgery. Which statement by the nurse about breast reconstruction is true? A. "Many women want breast reconstruction using their own tissue immediately after mastectomy." B. "Placement of saline- or gel-filled prostheses is not recommended because of the nature of the surgery." C. "Reconstruction of the nipple-areola complex is the first stage in the reconstruction of the breast." D. "The surgeon should offer the option of breast reconstruction surgery once healing has occurred after the mastectomy."

A. Many women want autogenous reconstruction after mastectomy.

The RN working on an oncology unit has just received report on these clients. Which client should be assessed first? A. A client with chemotherapy-induced neutropenia who has just been admitted with an elevated temperature B. A client with lymphoma who will need administration of an antiemetic before receiving chemotherapy C. A client with metastatic breast cancer who is scheduled for external beam radiation in 1 hour D. A client with xerostomia associated with laryngeal cancer who needs oral care before breakfast

A. Neutropenia poses high risk for life-threatening sepsis and septic shock, which develop and progress rapidly in immune suppressed people; the nurse should see this client first.

A RN is reinforcing home instructions to parents of a child diagnose with infectious mononucleosis. The RN would expect the plan to include: A. Notify the physician if the child develops abdominal pain or left shoulder pain B. Keeping the child supine for a week C. Notify the physician if the child develops a fever of more than 99.9 degrees D. That the child is cleared to return to school immediately

A. Notify the physician if the child develops abdominal pain or left shoulder pain

Which of the following are psychosocial outcomes associated with JIA? (Select all that apply) A. Positive body image B. Weight gain C. Freedom from pain D. Maintenance of joint mobility

A. Positive body image

What can a nurse do for a child with HF: A. Provide skin care B. Position the HOB to 90 degrees C. Keep a cool environment D. Document the urine every 3 hours

A. Provide skin care

Which of the following is the most priority nursing diagnosis in a patient with seizure disorders? A. Risk for injury related to seizure activity B. Fear related to the possibility of seizures C. Ineffective individual coping related to stresses imposed by epilepsy D. Deficient knowledge related to epilepsy and its control

A. Risk for injury related to seizure activity

3) When newborns have been on formula for 36-48 hours, they should have a: A. Screening for PKU B. Vitamin K injection C. Test for necrotizing enterocolitis D. Heel stick for blood glucose level

A. Screening for PKU

Which assessment is most important for the nurse to perform for the client receiving one unit of packed red blood cells from an autologous donation? A. Temperature B. Blood pressure C. Oxygen saturation D. IV site for hives

A. Temperature In an autologous blood transfusion, the client receives his or her own blood components. Therefore the chances for an incompatibility type reaction do not exist. The main problems that can come from autologous transfusion are fluid overload and infection from blood contamination during the collection, storage, or infusion processes. Fluid overload is very unlikely when only one unit is being transfused. Contamination and infection are just as likely with an autologous transfusion as they are with a transfusion of donated blood products. The most important assessment is for signs of infection, including temperature.

The nurse manager in a long-term care facility is developing a plan for primary and secondary prevention of colorectal cancer. Which tasks associated with the screening plan will be delegated to nursing assistants within the facility? A. Testing of stool specimens for occult blood B. Teaching about the importance of dietary fiber C. Referring clients for colonoscopy procedures D. Giving vitamin and mineral supplements

A. Testing of stool specimens for occult blood is done according to a standardized protocol and can be delegated to nursing assistants.

A client who has just been notified that the breast biopsy indicates a malignancy tells the nurse, "I just don't know how this could have happened to me." Which of these responses by the nurse will be most appropriate? A. "Tell me what you mean when you say you don't know how this could have happened to you." B. "Do you have a family history that might make you more likely to develop breast cancer?" C. "Would you like me to help you find more information about how breast cancer develops?" D. "Many risk factors for breast cancer have been identified, so it is difficult to determine what might have caused it."

A. The client's statement may indicate shock and denial or a request for more information. To provide appropriate care, further assessment is needed about the client's psychosocial status. The first action by the nurse in this situation is to obtain more data by asking open-ended questions.

The nurse is teaching postmastectomy exercises to the client. Which statement made by the client indicates that teaching has been effective? A. "For the pulley exercise, I'll drape a 6-foot-long rope over a shower curtain rod or over the top of a door." B. "In rope turning, I'll hold the rope with my arms flexed." C. "In rope turning, I'll start by making large circles." D. "With hand wall climbing, I'll walk my hands up the wall and back down until they are at waist level."

A. This is a correct description of how to perform the pulley exercise properly.

Why is treatment of Kawasaki Disease so imperative within the first 10 days? A. To prevent acquired heart disease B. To prevent further damage to the liver C. To prevent signs and symptoms from progressing D. To prevent the occurrence of Reyes syndrome

A. To prevent acquired heart disease

Which information must the organ transplant nurse emphasize before each client is discharged? A. Taking immune suppressant medications increases your risk for cancer and the need for screenings. B. You are at increased risk for cancer when you reach 60 years of age. C. Immunosuppressant medications will decrease your risk for developing cancers. D. After 6 months, you may stop immune suppressant medications, and your risk for cancer will be the same as that of the general population.

A. Use of immune suppressant medications to prevent organ rejection increases the risk for cancer. Immune suppressant medications must be taken for the life of the organ; the risk for developing cancer remains.

Which of the following laboratory values could indicate that a child has leukemia? A. WBCs 32,000/mm3 B. Platelets 300,000/mm3 C. Hemoglobin 15g/dL D. Blood pH of 7.35

A. WBCs 32,000/mm3

The parents of a child with ASD (autism spectrum disorder) often express concerns that their child: A. does not seek physical comfort or cuddling B. wants to constantly be cuddled C. is "clingy" with friends and siblings D. is always asking for hugs

A. does not seek physical comfort or cuddling

The nurse knows that diabetic teaching has been effective when parents of a newly diagnosed child state they will, during an illness, prove the child with: A. more insulin B. more calories C. less insulin D. less protein

A. more insulin Illness causes greater insulin resistance, so more is needed to achieve normal blood glucose levels. Increased calorie intake will be ineffective without more insulin to assist the body with metabolizing those calories. Restricting protein is not recommended, but during illness, fluids and light carbohydrates are usually tolerated best.

Which of the following symptoms should an infant exhibit with HF: A. tachypnea B.decreased appeitite C.nocturia D.Back pain

A. tachypnea B.decreased appeitite

Meperidine (Demerol) is not recommended for children in sickle cell crisis because it: a. may induce seizures. b. is easily addictive. c. is not adequate for pain relief. d. is given by intramuscular injection.

ANS: A A metabolite of meperidine, normeperidine, is a central nervous system stimulant that produces anxiety, tremors, myoclonus, and generalized seizures when it accumulates with repetitive dosing. Patients with sickle cell disease are particularly at risk for normeperidine-induced seizures. Meperidine is no more addictive than other narcotic agents. Meperidine is adequate for pain relief. It is available for IV infusion.

A young child with leukemia has anorexia and severe stomatitis. The nurse should suggest that the parents try which intervention? a. Relax any eating pressures. b. Firmly insist that child eat normally. c. Begin gavage feedings to supplement diet. d. Serve foods that are either hot or cold.

ANS: A A multifaceted approach is necessary for children with severe stomatitis and anorexia. First, the parents should relax eating pressures. The nurse should suggest that the parents try soft, bland foods; normal saline or bicarbonate mouthwashes; and local anesthetics. The stomatitis is a temporary condition. The child can resume good food habits as soon as the condition resolves.

In which of the conditions are all the formed elements of the blood simultaneously depressed? a. Aplastic anemia b. Sickle cell anemia c. Thalassemia major d. Iron deficiency anemia

ANS: A Aplastic anemia refers to a bone marrowfailure condition in which the formed elements of the blood are simultaneously depressed. Sickle cell anemia is a hemoglobinopathy in which normal adult hemoglobin is partly or completely replaced by abnormal sickle hemoglobin. Thalassemia major is a group of blood disorders characterized by deficiency in the production rate of specific hemoglobin globin chains. Iron deficiency anemia results in a decreased amount of circulating red cells.

The nurse is planning care for an adolescent with AIDS. Which is the priority nursing goal? a. Preventing infection b. Preventing secondary cancers c. Restoring immunologic defenses d. Identifying source of infection

ANS: A Because the child is immunocompromised in association with HIV infection, the prevention of infection is paramount. Although certain precautions are justified in limiting exposure to infection, these must be balanced with the concern for the childs normal developmental needs. Preventing secondary cancers is not currently possible. Current drug therapy is affecting the disease progression; although not a cure, these drugs can suppress viral replication, preventing further deterioration. Case finding is not a priority nursing goal.

The nurse is teaching parents about the importance of iron in a toddlers diet. Which explains why iron deficiency anemia is common during toddlerhood? a. Milk is a poor source of iron. b. Iron cannot be stored during fetal development. c. Fetal iron stores are depleted by age 1 month. d. Dietary iron cannot be started until age 12 months.

ANS: A Children between the ages of 12 and 36 months are at risk for anemia because cows milk is a major component of their diet and it is a poor source of iron. Iron is stored during fetal development, but the amount stored depends on maternal iron stores. Fetal iron stores are usually depleted by age 5 to 6 months. Dietary iron can be introduced by breastfeeding, iron-fortified formula, and cereals during the first 12 months of life.

A possible cause of acquired aplastic anemia in children is: a. drugs. b. injury. c. deficient diet. d. congenital defect.

ANS: A Drugs, such as chemotherapeutic agents and several antibiotics (e.g., chloramphenicol), can cause aplastic anemia. Injury, deficient diet, and congenital defect are not causative agents in acquired aplastic anemia.

A boy with leukemia screams whenever he needs to be turned or moved. Which is the most probable cause of this pain? a. Edema b. Bone involvement c. Petechial hemorrhages d. Changes within the muscles

ANS: B The invasion of the bone marrow with leukemic cells gradually causes a weakening of the bone and a tendency toward fractures. As leukemic cells invade the periosteum, increasing pressure causes severe pain. Edema, petechial hemorrhages, and changes within the muscles would not cause severe pain.

The nurse is administering an IV chemotherapeutic agent to a child with leukemia. The child suddenly begins to wheeze and have severe urticaria. Which is the most appropriate nursing action? a. Stop drug infusion immediately. b. Recheck rate of drug infusion. c. Observe child closely for next 10 minutes. d. Explain to child that this is an expected side effect.

ANS: A If an allergic reaction is suspected, the drug should be immediately discontinued. Any drug in the line should be withdrawn, and a normal saline infusion begun to keep the line open. Rechecking the rate of drug infusion, observing the child closely for next 10 minutes, and explaining to the child that this is an expected side effect can all be done after the drug infusion is stopped and the child is evaluated.

The nurse is conducting a staff in-service on inherited childhood blood disorders. Which statement describes severe combined immunodeficiency syndrome (SCIDS)? a. There is a deficit in both the humoral and cellular immunity with this disease. b. Production of red blood cells is affected with this disease. c. Adult hemoglobin is replaced by abnormal hemoglobin in this disease. d. There is a deficiency of T and B lymphocyte production with this disease.

ANS: A Severe combined immunodeficiency syndrome (SCIDS) is a genetic disorder that results in deficits of both humoral and cellular immunity. Wiskott-Aldrich is an X-linked recessive disorder with selected deficiencies of T and B lymphocytes. Fanconi syndrome is a hereditary disorder of red cell production. Sickle cell disease is characterized by the replacement of adult hemoglobin with an abnormal hemoglobin S.

When both parents have sickle cell trait, which is the chance their children will have sickle cell anemia? a. 25% b. 50% c. 75% d. 100%

ANS: A Sickle cell anemia is inherited in an autosomal recessive pattern. If both parents have sickle cell trait (one copy of the sickle cell gene), then for each pregnancy, a 25% chance exists that their child will be affected with sickle cell disease. With each pregnancy, a 50% chance exists that the child will have sickle cell trait. Percentages of 75% and 100% are too high for the children of parents who have sickle cell trait.

Parents of a child with sickle cell anemia ask the nurse, What happens to the hemoglobin in sickle cell anemia? Which statement by the nurse explains the disease process? a. Normal adult hemoglobin is replaced by abnormal hemoglobin. b. There is a lack of cellular hemoglobin being produced. c. There is a deficiency in the production of globulin chains. d. The size and depth of the hemoglobin are affected.

ANS: A Sickle cell anemia is one of a group of diseases collectively called hemoglobinopathies, in which normal adult hemoglobin is replaced by abnormal hemoglobin. Aplastic anemia is a lack of cellular elements being produced. Thalassemia major refers to a variety of inherited disorders characterized by deficiencies in production of certain globulin chains. Iron deficiency anemia affects the size and depth of the color.

The school nurse is informed that a child with human immunodeficiency virus (HIV) will be attending school soon. Which is an important nursing intervention? a. Carefully follow universal precautions. b. Determine how the child became infected. c. Inform the parents of the other children. d. Reassure other children that they will not become infected.

ANS: A Universal precautions are necessary to prevent further transmission of the disease. It is not the role of the nurse to determine how the child became infected. Informing the parents of other children and reassuring children that they will not become infected is a violation of the childs right to privacy.

The nurse is preparing to give oral care to a school-age child with mucositis secondary to chemotherapy administered to treat leukemia. Which preparations should the nurse use for oral care on this child? (Select all that apply.) a. Chlorhexidine gluconate (Peridex) b. Lemon glycerin swabs c. Antifungal troches (lozenges) d. Lip balm (Aquaphor) e. Hydrogen peroxide

ANS: A, C, D Preparations that may be used to prevent or treat mucositis include chlorhexidine gluconate (Peridex) because of its dual effectiveness against candidal and bacterial infections, antifungal troches (lozenges) or mouthwash, and lip balm (e.g., Aquaphor) to keep the lips moist. Agents that should not be used include lemon glycerin swabs (irritate eroded tissue and can decay teeth), hydrogen peroxide (delays healing by breaking down protein), and milk of magnesia (dries mucosa).

Chelation therapy is begun on a child with b-thalassemia major. The purpose of this therapy is to: a. treat the disease. b. eliminate excess iron. c. decrease risk of hypoxia. d. manage nausea and vomiting.

ANS: B A complication of the frequent blood transfusions in thalassemia is iron overload. Chelation therapy with deferoxamine (an iron-chelating agent) is given with oral supplements of vitamin C to increase iron excretion. Chelation therapy treats the side effect of the disease management. Decreasing the risk of hypoxia and managing nausea and vomiting are not the purposes of chelation therapy.

A young boy will receive a bone marrow transplant (BMT). This is possible because one of his older siblings is a histocompatible donor. Which is this type of BMT called? a. Syngeneic b. Allogeneic c. Monoclonal d. Autologous

ANS: B Allogeneic transplants are from another individual. Because he and his sibling are histocompatible, the BMT can be done. Syngeneic marrow is from an identical twin. There is no such thing as a monoclonal BMT. Autologous refers to the individuals own marrow.

A young child with human immunodeficiency virus (HIV) is receiving several antiretroviral drugs. The purpose of these drugs is to: a. cure the disease. b. delay disease progression. c. prevent spread of disease. d. treat Pneumocystis carinii pneumonia.

ANS: B Although not a cure, these antiviral drugs can suppress viral replication, preventing further deterioration of the immune system and delaying disease progression. At this time, cure is not possible. These drugs do not prevent the spread of the disease. P. carinii prophylaxis is accomplished with antibiotics.

Which is often administered to prevent or control hemorrhage in a child with cancer? a. Nitrosoureas b. Platelets c. Whole blood d. Corticosteroids

ANS: B Most bleeding episodes can be prevented or controlled with the administration of platelet concentrate or platelet-rich plasma. Nitrosoureas, whole blood, and corticosteroids would not prevent or control hemorrhage.

Which immunization should be given with caution to children infected with human immunodeficiency virus (HIV)? a. Influenza b. Varicella c. Pneumococcal d. Inactivated poliovirus (IPV)

ANS: B The children should be carefully evaluated before being given live viral vaccines such as varicella, measles, mumps, and rubella. The child must be immunocompetent and not have contact with other severely immunocompromised individuals. Influenza, pneumococcal, and inactivated poliovirus (IPV) are not live vaccines.

A school-age child is admitted in vasoocclusive sickle cell crisis. The childs care should include: a. correction of acidosis. b. adequate hydration and pain management. c. pain management and administration of heparin. d. adequate oxygenation and replacement of factor VIII.

ANS: B The management of crises includes adequate hydration, minimization of energy expenditures, pain management, electrolyte replacement, and blood component therapy if indicated. Hydration and pain control are two of the major goals of therapy. The acidosis will be corrected as the crisis is treated. Heparin and factor VIII is not indicated in the treatment of vasoocclusive sickle cell crisis. Oxygen may prevent further sickling, but it is not effective in reversing sickling because it cannot reach the clogged blood vessels.

The parents of a child hospitalized with sickle cell anemia tell the nurse that they are concerned about narcotic analgesics causing addiction. Which is appropriate for the nurse to explain about narcotic analgesics? a. Are often ordered but not usually needed b. Rarely cause addiction because they are medically indicated c. Are given as a last resort because of the threat of addiction d. Are used only if other measures, such as ice packs, are ineffective

ANS: B The pain of sickle cell anemia is best treated by a multidisciplinary approach. Mild to moderate pain can be controlled by ibuprofen and acetaminophen. When narcotics are indicated, they are titrated to effect and are given around the clock. Patient-controlled analgesia reinforces the patients role and responsibility in managing the pain and provides flexibility in dealing with pain. Few, if any, patients who receive opioids for severe pain become behaviorally addicted to the drug. Narcotics are often used because of the severe nature of the pain of vasoocclusive crisis. Ice is contraindicated because of its vasoconstrictive effects.

The nurse is planning care for a school-age child admitted to the hospital with hemophilia. Which interventions should the nurse plan to implement for this child? (Select all that apply.) a. Finger sticks for blood work instead of venipunctures b. Avoidance of IM injections c. Acetaminophen (Tylenol) for mild pain control d. Soft tooth brush for dental hygiene e. Administration of packed red blood cells

ANS: B, C, D Nurses should take special precautions when caring for a child with hemophilia to prevent the use of procedures that may cause bleeding, such as IM injections. The subcutaneous route is substituted for IM injections whenever possible. Venipunctures for blood samples are usually preferred for these children. There is usually less bleeding after the venipuncture than after finger or heel punctures. Neither aspirin nor any aspirin-containing compound should be used. Acetaminophen is a suitable aspirin substitute, especially for controlling mild pain. A soft toothbrush is recommended for dental hygiene to prevent bleeding from the gums. Packed red blood cells are not administered. The primary therapy for hemophilia is replacement of the missing clotting factor. The products available are factor VIII concentrates.

Which should the nurse teach about prevention of sickle cell crises to parents of a preschool child with sickle cell disease? (Select all that apply.) a. Limit fluids at bedtime. b. Notify the health care provider if a fever of 38.5 C (101.3 F) or greater occurs. c. Give penicillin as prescribed. d. Use ice packs to decrease the discomfort of vasoocclusive pain in the legs. e. Notify the health care provider if your child begins to develop symptoms of a cold.

ANS: B, C, E The most important issues to teach the family of a child with sickle cell anemia are to (1) seek early intervention for problems, such as a fever of 38.5 C (101.3 F) or greater; (2) give penicillin as ordered; (3) recognize signs and symptoms of splenic sequestration, as well as respiratory problems that can lead to hypoxia; and (4) treat the child normally. The nurse emphasizes the importance of adequate hydration to prevent sickling and to delay the adhesionstasisthrombosisischemia cycle. It is not sufficient to advise parents to force fluids or encourage drinking. They need specific instructions on how many daily glasses or bottles of fluid are required. Many foods are also a source of fluid, particularly soups, flavored ice pops, ice cream, sherbet, gelatin, and puddings. Increased fluids combined with impaired kidney function result in the problem of enuresis. Parents who are unaware of this fact frequently use the usual measures to discourage bedwetting, such as limiting fluids at night. Enuresis is treated as a complication of the disease, such as joint pain or some other symptom, to alleviate parental pressure on the child. Ice should not be used during a vasoocclusive pain crisis because it vasoconstricts and impairs circulation even more.

Parents of a school-age child with hemophilia ask the nurse, Which sports are recommended for children with hemophilia? Which sports should the nurse recommend? (Select all that apply.) a. Soccer b. Swimming c. Basketball d. Golf e. Bowling

ANS: B, D, E Because almost all persons with hemophilia are boys, the physical limitations in regard to active sports may be a difficult adjustment, and activity restrictions must be tempered with sensitivity to the childs emotional and physical needs. Use of protective equipment, such as padding and helmets, is particularly important, and noncontact sports, especially swimming, walking, jogging, tennis, golf, fishing, and bowling, are encouraged. Contact sport such as soccer and basketball are not recommended.

Which is caused by a virus that primarily infects a specific subset of T lymphocytes, the CD4+ T cells? a. Wiskott-Aldrich syndrome b. Idiopathic thrombocytopenic purpura c. Acquired immunodeficiency syndrome (AIDS) d. Severe combined immunodeficiency disease

ANS: C AIDS is caused by the human immunodeficiency virus (HIV), which primarily attacks the CD4+ T cells. Wiskott-Aldrich syndrome, idiopathic thrombocytopenic purpura, and severe combined immunodeficiency disease are not viral illnesses.

Which child should the nurse document as being anemic? a. 7-year-old child with a hemoglobin of 11.5 g/dl b. 3-year-old child with a hemoglobin of 12 g/dl c. 14-year-old child with a hemoglobin of 10 g/dl d. 1-year-old child with a hemoglobin of 13 g/dl

ANS: C Anemia is a condition in which the number of red blood cells, or hemoglobin concentration, is reduced below the normal values for age. Anemia is defined as a hemoglobin level below 10 or 11 g/dl. The child with a hemoglobin of 10 g/dl would be considered anemic. The normal hemoglobin for a child after 2 years of age is 11.5 to 15.5 g/dl.

The nurse is planning activity for a 4-year-old child with anemia. Which activity should the nurse plan for this child? a. Game of hide and seek in the childrens outdoor play area b. Participation in dance activities in the playroom c. Puppet play in the childs room d. A walk down to the hospital lobby

ANS: C Because the basic pathologic process in anemia is a decrease in oxygen-carrying capacity, an important nursing responsibility is to assess the childs energy level and minimize excess demands. The childs level of tolerance for activities of daily living and play is assessed, and adjustments are made to allow as much self-care as possible without undue exertion. Puppet play in the childs room would not be overly tiring. Hide and seek, dancing, and walking to the lobby would not conserve the anemic childs energy.

An adolescent will receive a bone marrow transplant (BMT). The nurse should explain that the bone marrow will be administered by which route? a. Bone grafting b. Bone marrow injection c. IV infusion d. Intra-abdominal infusion

ANS: C Bone marrow from a donor is infused intravenously, and the transfused stem cells will repopulate the marrow. Because the stem cells migrate to the recipients marrow when given intravenously, this is the method of administration.

The nurse is teaching parents of an infant about the causes of iron deficiency anemia. Which statement best describes iron deficiency anemia in infants? a. It is caused by depression of the hematopoietic system. b. It is easily diagnosed because of an infants emaciated appearance. c. Clinical manifestations are similar regardless of the cause of the anemia. d. Clinical manifestations result from a decreased intake of milk and the premature addition of solid foods.

ANS: C In iron deficiency anemia, the childs clinical appearance is a result of the anemia, not the underlying cause. Usually the hematopoietic system is not depressed in iron deficiency anemia. The bone marrow produces red cells that are smaller and contain less hemoglobin than normal red cells. Children who are iron deficient from drinking excessive quantities of milk are usually pale and overweight. They are receiving sufficient calories, but are deficient in essential nutrients. The clinical manifestations result from decreased intake of iron-fortified solid foods and an excessive intake of milk.

Which statement best describes b-thalassemia major (Cooley anemia)? a. All formed elements of the blood are depressed. b. Inadequate numbers of red blood cells are present. c. Increased incidence occurs in families of Mediterranean extraction. d. Increased incidence occurs in persons of West African descent.

ANS: C Individuals who live near the Mediterranean Sea and their descendants have the highest incidence of thalassemia. An overproduction of red cells occurs. Although numerous, the red cells are relatively unstable. Sickle cell disease is common in persons of West African descent.

Iron dextran is ordered for a young child with severe iron deficiency anemia. Nursing considerations include to: a. administer with meals. b. administer between meals. c. inject deeply into a large muscle. d. massage injection site for 5 minutes after administration of drug.

ANS: C Iron dextran is a parenteral form of iron. When administered intramuscularly, it must be injected into a large muscle. Iron dextran is for intramuscular or intravenous (IV) administration. The site should not be massaged to prevent leakage, potential irritation, and staining of the skin.

Which is most descriptive of the pathophysiology of leukemia? a. Increased blood viscosity occurs. b. Thrombocytopenia (excessive destruction of platelets) occurs. c. Unrestricted proliferation of immature white blood cells (WBCs) occurs. d. First stage of coagulation process is abnormally stimulated.

ANS: C Leukemia is a group of malignant disorders of the bone marrow and lymphatic system. It is defined as an unrestricted proliferation of immature WBCs in the blood-forming tissues of the body. Increased blood viscosity may occur secondary to the increased number of WBCs. Thrombocytopenia may occur secondary to the overproduction of WBCs in the bone marrow. The coagulation process is unaffected by leukemia.

Several blood tests are ordered for a preschool child with severe anemia. The child is crying and upset because of memories of the venipuncture done at the clinic 2 days ago. The nurse should explain: a. venipuncture discomfort is very brief. b. only one venipuncture will be needed. c. topical application of local anesthetic can eliminate venipuncture pain. d. most blood tests on children require only a finger puncture because a small amount of blood is needed.

ANS: C Preschool children are concerned with both pain and the loss of blood. When preparing the child for venipuncture, the nurse will use a topical anesthetic to eliminate any pain. This is a traumatic experience for preschool children. They are concerned about their bodily integrity. A local anesthetic should be used, and a bandage should be applied to maintain bodily integrity. The nurse should not promise one attempt in case multiple attempts are required. Both finger punctures and venipunctures are traumatic for children. Both require preparation.

The nurse is conducting a staff in-service on sickle cell anemia. Which describes the pathologic changes of sickle cell anemia? a. Sickle-shaped cells carry excess oxygen. b. Sickle-shaped cells decrease blood viscosity. c. Increased red blood cell destruction occurs. d. Decreased adhesion of sickle-shaped cells occurs.

ANS: C The clinical features of sickle cell anemia are primarily the result of increased red blood cell destruction and obstruction caused by the sickle-shaped red blood cells. Sickled red cells have decreased oxygen-carrying capacity and transform into the sickle shape in conditions of low oxygen tension. When the sickle cells change shape, they increase the viscosity in the area where they are involved in the microcirculation. Increased adhesion and entanglement of cells occurs.

Parents of a hemophiliac child ask the nurse, Can you describe hemophilia to us? Which response by the nurse is descriptive of most cases of hemophilia? a. Autosomal dominant disorder causing deficiency in a factor involved in the blood-clotting reaction b. X-linked recessive inherited disorder causing deficiency of platelets and prolonged bleeding c. X-linked recessive inherited disorder in which a blood-clotting factor is deficient d. Y-linked recessive inherited disorder in which the red blood cells become moon-shaped

ANS: C The inheritance pattern in 80% of all of the cases of hemophilia is X-linked recessive. The two most common forms of the disorder are factor VIII deficiency, hemophilia A or classic hemophilia; and factor IX deficiency, hemophilia B or Christmas disease. The inheritance pattern is X-linked recessive. The disorder involves coagulation factors, not platelets, and does not involve red cells or the Y chromosomes.

A school-age child with leukemia experienced severe nausea and vomiting when receiving chemotherapy for the first time. Which is the most appropriate nursing action to prevent or minimize these reactions with subsequent treatments? a. Encourage drinking large amounts of favorite fluids. b. Encourage child to take nothing by mouth (remain NPO) until nausea and vomiting subside. c. Administer an antiemetic before chemotherapy begins. d. Administer an antiemetic as soon as child has nausea.

ANS: C The most beneficial regimen to minimize nausea and vomiting associated with chemotherapy is to administer the antiemetic before the chemotherapy is begun. The goal is to prevent anticipatory symptoms. Drinking fluids will add to the discomfort of the nausea and vomiting. Remaining until nausea and vomiting subside will help with this episode, but the child will have the discomfort and be at risk for dehydration. Administering an antiemetic as soon as the child has nausea does not prevent anticipatory nausea.

Which should the nurse include when teaching the mother of a 9-month-old infant about administering liquid iron preparations? a. They should be given with meals. b. They should be stopped immediately if nausea and vomiting occur. c. Adequate dosage will turn the stools a tarry green color. d. Allow preparation to mix with saliva and bathe the teeth before swallowing.

ANS: C The nurse should prepare the mother for the anticipated change in the childs stools. If the iron dose is adequate, the stools will become a tarry green color. The lack of the color change may indicate insufficient iron. The iron should be given in two divided doses between meals when the presence of free hydrochloric acid is greatest. Iron is absorbed best in an acidic environment. Vomiting and diarrhea may occur with iron administration. If these occur, the iron should be given with meals, and the dosage reduced, then gradually increased as the child develops tolerance. Liquid preparations of iron stain the teeth. They should be administered through a straw and the mouth rinsed after administration.

Which clinical manifestation should the nurse expect when a child with sickle cell anemia experiences an acute vasoocclusive crisis? a. Circulatory collapse b. Cardiomegaly, systolic murmurs c. Hepatomegaly, intrahepatic cholestasis d. Painful swelling of hands and feet; painful joints

ANS: D A vasoocclusive crisis is characterized by severe pain in the area of involvement. If in the extremities, painful swelling of the hands and feet is seen; if in the abdomen, severe pain resembles that of acute surgical abdomen; and if in the head, stroke and visual disturbances occur. Circulatory collapse results from sequestration crises. Cardiomegaly, systolic murmurs, hepatomegaly, and intrahepatic cholestasis result from chronic vasoocclusive phenomena.

The nurse is preparing a child for possible alopecia from chemotherapy. Which should be included? a. Explain to child that hair usually regrows in 1 year. b. Advise child to expose head to sunlight to minimize alopecia. c. Explain to child that wearing a hat or scarf is preferable to wearing a wig. d. Explain to child that when hair regrows, it may have a slightly different color or texture.

ANS: D Alopecia is a side effect of certain chemotherapeutic agents. When the hair regrows, it may be a different color or texture. The hair usually grows back within 3 to 6 months after cessation of treatment. The head should be protected from sunlight to avoid sunburn. Children should choose the head covering they prefer.

Which is a common clinical manifestation of Hodgkin disease? a. Petechiae b. Bone and joint pain c. Painful, enlarged lymph nodes d. Enlarged, firm, nontender lymph nodes

ANS: D Asymptomatic, enlarged, cervical or supraclavicular lymphadenopathy is the most common presentation of Hodgkin disease. Petechiae are usually associated with leukemia. Bone and joint pain are not likely in Hodgkin disease. The enlarged nodes are rarely painful.

The nurse is recommending how to prevent iron deficiency anemia in a healthy, term, breast-fed infant. Which should be suggested? a. Iron (ferrous sulfate) drops after age 1 month b. Iron-fortified commercial formula by age 4 to 6 months c. Iron-fortified infant cereal by age 2 months d. Iron-fortified infant cereal by age 4 to 6 months

ANS: D Breast milk supplies inadequate iron for growth and development after age 5 months. Supplementation is necessary at this time. The mother can supplement the breastfeeding with iron-fortified infant cereal. Iron supplementation or the introduction of solid foods in a breast-fed baby is not indicated. Providing iron-fortified commercial formula by age 4 to 6 months should be done only if the mother is choosing to discontinue breastfeeding.

Which is the most effective pain-management approach for a child who is having a bone marrow aspiration? a. Relaxation techniques b. Administration of an opioid c. EMLA cream applied over site d. Conscious or unconscious sedation

ANS: D Effective pharmacologic and nonpharmacologic measures should be used to minimize pain associated with procedures. For bone marrow aspiration, conscious or unconscious sedation should be used. Relaxation, opioids, and EMLA can be used to augment the conscious or unconscious sedation.

A child with leukemia is receiving triple intrathecal chemotherapy consisting of methotrexate, cytarabine, and hydrocortisone. The purpose of this is to prevent: a. infection. b. brain tumor. c. drug side effects. d. central nervous system (CNS) disease.

ANS: D For certain children, CNS prophylactic therapy is indicated. This drug regimen is used to prevent CNS leukemia and will not prevent infection or drug side effects. If the child has a brain tumor in addition to leukemia, additional therapy would be indicated.

The nurse is conducting a staff in-service on childhood blood disorders. Which describes the pathology of idiopathic thrombocytopenic purpura? a. Bone marrow failure in which all elements are suppressed b. Deficiency in the production rate of globin chains c. Diffuse fibrin deposition in the microvasculature d. An excessive destruction of platelets

ANS: D Idiopathic thrombocytopenic purpura is an acquired hemorrhagic disorder characterized by an excessive destruction of platelets, discolorations caused by petechiae beneath the skin, and a normal bone marrow. Aplastic anemia refers to a bone marrowfailure condition in which the formed elements of the blood are simultaneously depressed. Thalassemia major is a group of blood disorders characterized by deficiency in the production rate of specific hemoglobin globin chains. Disseminated intravascular coagulation is characterized by diffuse fibrin deposition in the microvasculature, consumption of coagulation factors, and endogenous generation of thrombin and plasma.

The nurse is reviewing first aid with a group of school nurses. Which statement made by a participant indicates a correct understanding of the information? a. If a child loses a tooth due to injury, I should place the tooth in warm milk. b. If a child has recurrent abdominal pain, I should send him or her back to class until the end of the day. c. If a child has a chemical burn to the eye, I should irrigate the eye with normal saline. d. If a child has a nosebleed, I should have the child sit up and lean forward.

ANS: D If a child has a nosebleed, the child should lean forward, not lie down. A tooth should be placed in cold milk or saliva for transporting to a dentist. Recurrent abdominal pain is a physiologic problem and requires further evaluation. If a chemical burn occurs in the eye, the eye should be irrigated with water for 20 minutes.

Several complications can occur when a child receives a blood transfusion. Which is an immediate sign or symptom of an air embolus? a. Chills and shaking b. Nausea and vomiting c. Irregular heart rate d. Sudden difficulty in breathing

ANS: D Signs of air embolism are sudden difficulty breathing, sharp pain in the chest, and apprehension. Air emboli should be avoided by carefully flushing all tubing of air before connecting to patient. Chills, shaking, nausea, and vomiting are associated with hemolytic reactions. Irregular heart rate is associated with electrolyte disturbances and hypothermia.

Myelosuppression, associated with chemotherapeutic agents or some malignancies such as leukemia, can cause bleeding tendencies because of a(n): a. decrease in leukocytes. b. increase in lymphocytes. c. vitamin C deficiency. d. decrease in blood platelets.

ANS: D The decrease in blood platelets secondary to the myelosuppression of chemotherapy can cause an increase in bleeding. The child and family should be alerted to avoid risk of injury. Decrease in leukocytes, increase in lymphocytes, and vitamin C deficiency would not affect bleeding tendencies.

An 8-year-old girl is receiving a blood transfusion when the nurse notes that she has developed precordial pain, dyspnea, distended neck veins, slight cyanosis, and a dry cough. These manifestations are most suggestive of: a. air emboli. b. allergic reaction. c. hemolytic reaction. d. circulatory overload.

ANS: D The signs of circulatory overload include distended neck veins, hypertension, crackles, dry cough, cyanosis, and precordial pain. Signs of air embolism are sudden difficulty breathing, sharp pain in the chest, and apprehension. Allergic reactions are manifested by urticaria, pruritus, flushing, asthmatic wheezing, and laryngeal edema. Hemolytic reactions are characterized by chills, shaking, fever, pain at infusion site, nausea, vomiting, tightness in chest, flank pain, red or black urine, and progressive signs of shock and renal failure.

Which immunization should not be given to a child receiving chemotherapy for cancer? a. Tetanus vaccine b. Inactivated poliovirus vaccine c. Diphtheria, pertussis, tetanus (DPT) d. Measles, rubella, mumps

ANS: D The vaccine used for measles, mumps, and rubella is a live virus and can result in an overwhelming infection. Tetanus vaccine, inactivated poliovirus vaccine, and diphtheria, pertussis, tetanus (DPT) are not live virus vaccines.

A diagnostic workup is being performed on a 1-year-old child with suspected neuroblastoma. The nurse reviews the results of the diagnostic tests and understands that which of the following findings is most specifically related to this type of tumor? "1. Elevated vanillylmandelic acid (VMA) urinary levels 2. Presence of blast cells in the bone marrow 3. Projectile vomiting, usually in the morning 4. Postive Babinski's sign"

ANSWER: 1 Rationale: Neuroblastoma is a solid tumor found only in children. It arises from neural crest cells that develop into the sympathetic nervous system and the adrenal medulla. Typically, the tumor compresses adjacent normal tissue and organs. Neuroblastoma cells may excrete catecholamines and their metabolites. Urine samples will indicate elevated VMA levels. The presence of blast cells in the bone marrow occurs in leukemia. Projectile vomiting occurring most often in the morning and a positive Babinski's sign are clinical manifestations of a brain tumor.

The nurse is caring for a school age child with ringworm the nurse should expect that therapeutic management for this child includes

Administering oral griseofulvin

A pregnant client has heard about Down syndrome and wants to know about the risk factors associated with it. Which of the following would the nurse include as a risk factor? a) Advanced maternal age b) Recurrent miscarriages c) Advanced paternal age d) Family history of condition

Advanced maternal age Correct Explanation: Advanced maternal age is one the most important factors that increases the risk of an infant being born with Down syndrome. Down syndrome is not associated with advanced paternal age, recurrent miscarriages, or family history of Down syndrome.

When the nurse is counseling a 60-year-old African-American male client with all of these risk factors for lung cancer, teaching should focus most on which risk factor? A. Tobacco use B. Ethnicity C. Gender D. Increased age

Although all of these are risk factors for lung cancer, the client's tobacco use is the only factor that he can change.

A woman in her third trimester has just learned that her fetus has been diagnosed with cri-du-chat syndrome. The nurse recognizes that this child will likely have which of the following characteristics? a) Small and nonfunctional ovaries b) Cleft lip and palate c) Rounded soles of the feet (rocker-bottom) d) An abnormal, cat-like cry

An abnormal, cat-like cry Correct Explanation: Cri-du-chat syndrome is the result of a missing portion of chromosome 5. In addition to an abnormal cry, which sounds much more like the sound of a cat than a human infant's cry, children with cri-du-chat syndrome tend to have a small head, wide-set eyes, a downward slant to the palpebral fissure of the eye, and a recessed mandible. They are severely cognitively challenged. Rounded soles of the feet are characteristic of trisomy 18 syndrome. Cleft lip and palate are characteristic of trisomy 13 syndrome. Small and nonfunctional ovaries are characteristic of Turner syndrome.

The pediatric nurse understands that the most common cancer found in children is: A. Non-hodgkin's lymphoma B. Acute lymphocytic leukemia C. Chronic lymphocytic leukemia D. Ewing's sarcoma

B. Acute lymphocytic leukemia

A child with leukemia is being discharged after beginning chemotherapy. Which of the following instructions will the nurse include when teaching the parents of this child? "A. provide a diet low in protein and high in carboydrates B. avoid fresh vegetables that are not cooked or peeled C. notify the doctor if the child's temp exceeds 101 degrees F D. increase the use of humidifiers throughout the house"

Answer B - fresh vegetables harbor microorganisms, which can cause infections in immune-compromised children, fruit or vegetables should be either peeled or cooked. The physician should be notified of a temp above 100 degrees F. A diet low in protein is not indicated. Humidifiers harbor fungi in the water containers.

"A 4-year-old has a right nephrectomy to remove a Wilms tumor. The nurse knows that it is essential to: "A. Request a low-salt diet B. Restrict fluids C. Educate the family regarding renal transplants D. Prevent urinary tract infections"

Answer D is correct. Because the child has only one remaining kidney, it is important to prevent urinary tract infections. Answers A, B, and C are not necessary, so they are incorrect.

When caring for an 11-month-old infant with dehydration and metabolic acidosis, the nurse expects to see which of the following? "a. A reduced white blood cell count b. A decreased platelet count c. Shallow respirations d. Tachypnea"

Answer D. The body compensates for metabolic acidosis via the respiratory system, which tries to eliminate the buffered acids by increasing alveolar ventilation through deep, rapid respirations, altered white blood cell or platelet counts are not specific signs of metabolic imbalance.

"Which diagnostic test should be performed annually after age 50 to screen for colon cancer? "a. Abdominal computed tomography (CT) b. Abdominal X-ray c. Colonoscopy d. Fecal occult blood test"

Answer d: Surface blood vessels of polyps and cancers are fragile and often bleed with the passage of stools, so a fecal occult blood test and CT scan can help establish tumor size and metastasis. A colonoscopy can help to locate a tumor as well as polyps, but is only recommended every 10 years.

Which of the following does the nurse understand places the child with myelomeningocele at high risk for infection? A) Neurogenic bladder B) Bowel incontinence C) Latex allergy D) Exposure of sac E) Corticosteroid use

Answer: A and D. Neurogenic bladder is the failure of the bladder to either store urine properly or empty itself of urine. Because of this urinary stasis occurs in the bladder placing the child at risk for infection. In myelomeningocele, the spinal cord is exposed, placing the child at high risk for infection. Immediate surgery is needed to help prevent infection from occurring. Bowel incontinence often occurs in children with myelomeningocele, but does not pose the same health risks as urinary incontinence. Latex allergy, although common, would not promote risk for infection alone. Corticosteroid use is not common in children with myelomeningocele.

The 7 year old patient has had a cast on to heal his fracture of his arm. After the expected time period, the nurse is teaching a child about what to expect when removing his cast. Which of the following teaching points should the nurse include? Select All that Apply: A) "The cast could begin to feel really warm as the striker saw is taking the cast off" B) "The striker saw will be very loud" C) "Look, see, the saw won't be able to cut your skin" D) "Once the cast is removed we will soak your leg in warm water" E) "you will still need to keep your leg very still even after the cast is removed"

Answer: A, B, C, and D. The cast/extremity under the cast could begin to feel warm during the cast removal process. The saw could be loud, but the nurse should demonstrate on him/herself that the saw can't won't cut the child. Once the cast is removed, it will be soaked and washed in warm soapy water, and it should be soaked in warm water daily. All of these need to be communicated to the child on the level of their understanding. The child should be told to start increasing activity the limb to regain strength and range of motion.

Which of the following could the nurse do to assess for hypotonia of the 4 month-old infant? SATA: A) Pick up the child and see if the child feels like it is slipping out of the nurse's grasp B) Assess to see if the child can momentarily support his own weight when placed in a standing position C) Hold the child up and ask them to walk forward for a few steps D) Move the infant from the supine position to the sitting position and see if the child can hold up his own neck E) Move the infants muscles and note any muscle spasms not associated with the muscle movement

Answer: A, B, and D. Hypotonia is often an indicator that the child has some form of neuromuscular disorder. The feeling that the child is slipping out of the examiners hands is often as sign of hypotonia. By four months the child is often able to support their own neck and to stand momentarily when their feet are placed on a flat surface and their body is in the vertical position. The child would not be able to walk on command at this age. Assessing for extraneous muscle spasms would indicate the presence of hypertonia, not hypotonia.

The nurse anticipates which of the following orders for the patient with Duchenne Muscular Dystrophy? Select all that apply: A) Prednisone B) Calcium supplements C) Bedrest D) Botulinum Toxin E) Chest percussion

Answer: A, B, and E. Corticosteroids are thought to help slow the progression of the disease. Calcium supplements are provided to help with the long-term effects of osteoporosis caused by the corticosteroids. Chest percussion can help remove excess secretions from the respiratory tract that the patient may be too weak to expel by themselves. Bedrest or use of botulinum toxin would not be recommended for this patient.

The nurse is caring for the patient with Russel's traction. Which of the following should the nurse include in this patients plan of care? SATA: A) Weight should remain off of the floor at all times B) Place a foot support to prevent foot drop C) Release traction for 5 minutes of every hour to provide skin care D) Ensure heel is resting on bed at all times E) Assess neurovascular status q 4 hours

Answer: A, B, and E. With all traction, the weights should remain off of the floor at all times and should not be released periodically or stopped for any reason unless emergent. A foot support will be needed for this patient because foot drop could develop related to the heel being elevated without support. The heel should be off of the bed at all times, not resting on it. Neurovascular status should be assessed often on this patient (as often as vitals are done).

Which of the following symptoms would the nurse expect to possibly see in the child with Duchenne muscular dystrophy? Select all that apply a) Protuberant belly b) Diminished intelligence c) Walking on the toes or balls of feet d) Gower's sign e) Spinal curvatures

Answer: A, C, D, and E. To protect balance, which is impaired in this disorder, the child may often have their belly sticking out with their shoulders pulled back. They often have a waddling gait and walk on the toes or on the balls of the feet. Gower's sign is the use of a special technique in order to rise off of the floor. Spinal curvatures often occur as the muscles in the body atrophy (including lordosis, kyphosis, and scoliosis). Intelligence is rarely affected by this disorder.

The nurse is providing discharge instructions to the family and the 5 year old child who has received a cast for his broken lower leg. Which statement, if made by the legal guardian would indicate the need for further teaching? A) I can place ice on the affected leg for the first 2 days for an hour at a time B) I should make sure that the leg is elevated C) We can use a blow dryer to help relieve the itching inside the cast D) We should call in if my kid complains of severe, unrelieved pain in his leg

Answer: A. Ice should only be used for periods of 20-30 minutes. All other pieces of information are correct (page 844).

You are the nurse working at a pediatrics clinic in Miami. You are assessing four amazingly awesome patients today. Which assessment finding concerns the nurse the most? A) The patient diagnosed with Duchenne Muscular Dystrophy demonstrating Gower's sign, waddling gait, and tachycardia B) The patient with cerebral palsy with scoliosis who is need of bracing C) The patient with myelomeningocele whose urine is cloudy and smells foul D) The patient with a brachial plexus injury who has an absent Moro's reflex

Answer: A. Tachycardia in the patient with Duchenne Muscular Dystrophy is a sign of heart muscle weakening, a serious complicating of this disorder. Scoliosis often occurs in patients with cerebral palsy. Cloudy, foul-smelling is a sign of a UTI, which is a complication of neurogenic bladder. This is concerning, but not as concerning as the heart trouble with Duchenne. An absent Moro's reflex would be expected in the patient with a brachial plexus injury.

The nurse is assessing a young infant who comes in after being involved in a serious MVA. The nurse's priority would be? a) Assess for the presence of hypotonia or hypertonia of the neck b) Listen to respiratory sounds c) Brace the child's back to immobilize the child's spine d) Obtain a set of vital signs

Answer: C. It would be important to immobilize this child's spine.

The nurse is especially concerned to assess for adequate respiratory function in which of the following disease processes? Select all that apply: a) Spina bifida occulta b) Duchene muscular dystrophy c) Spinal Muscular Atrophy d) Brachial plexus injury e) Cerebral Palsy

Answer: B, C, and E.

The nurse is providing discharge instructions to the family and the 9 year old child who has received a Gore-tex cast for his broken lower leg. The mother states to the nurse, "how do I know if something is going wrong with the cast?" The nurse should teach the mother to call the physician if: A) The cast gets wet B) The toes become cold to the touch C) The child has a fever of 100 degrees for a couple hours D) The "petals" on the edge of the cast fall off E) The child feels extreme itchiness inside the cast

Answer: B, C, and E. Page 844. Because this is a Gore-tex cast, it is allowed to get wet and would not be a cause for concern. Any decrease in the neurovascular status of the limb should be reported including coolness of the toes. A temperature of 101.5 degrees or higher for 24 hours or longer should be reported. Because this is a gore-tex cast, no petals should be placed around the edges of the cast (page 843). Extreme itchiness within the cast would be reason to call.

A nurse is assessing a 3 year old child in the pediatrics wellness clinic, which of the following would be least concerning to the nurse? a) The child is holding his right arm close to his chest and refusing to use it b) When the child stands with his knees together, his ankles are far apart c) The child's pelvis drops slightly whenever he walks d) The child is in the shortest percentile for his age group

Answer: B. This condition, known as genu valgum or knock-knees, a common finding in children aged 2-3 and would not be concerning to the nurse. Refusal to use a limb could be a sign of damage or fracture to that limb. Trendelenberg's sign, or the drop of the pelvis when walking, could be a sign of hip dysplasia. Being in the shortest percentile for his age group could be a sign of a growth delay possibly caused by an underlying condition.

You, the awesome sauce nurse, are working at the new equally amazing pediatric clinic that opened literally five minutes from your house (nice commute!) your first patient is a child with marked pectus excavatum. What is the top nursing diagnosis for this patient? A) Impaired mobility B) Ineffective breathing pattern C) Disturbed body image D) Chronic pain

Answer: B. children with marked pectus excavatum should have a thorough assessment of their cardiac and pulmonary function, because the funnel chest can place pressure on both of these symptoms impairing proper expansion of the lungs. Page 849.

Little baby joe was born with an outer sac on his spine. Which of the following would correlate with a diagnosis of myelomeningocele? SATA: A) Accompanying hydrocephalus B) Leakage of the CSF C) Absence of deep tendon reflexes D) Constant dribbling of urine E) Meninges of the spine in the sac

Answer: C and D. Hydrocephalus could be present with either meningocele or myelomeningocele. Leakage of CSF would indicate a serious complication, not a common finding. The absence of deep tendon reflexes or the constant dribbling of urine indicates some neural involvement, differentiating it from meningocele. Meninges in the sac occurs in both myelomeningocele and meningocele. In myelomeningocele, the cord itself also protrudes into the sac

You are the nurse taking care of the infant just diagnosed with cerebral palsy. The mother of the child asks you, "What does this mean for my child?" What is the best response by the nurse? A) This means that you child will gradually lose more and more muscle mass until eventually they will be unable to sustain their respiratory function B) This is a disorder related to how your child was born. Most likely they sustained injury during the birthing process C) There really is no specific way to tell how this disease will affect your child other than it will affect the muscle tone and control in some way. D) Why are you asking me? I ain't no doctor!

Answer: C. Cerebral Palsy is a term used to describe a range of nonspecific clinical symptoms characterized by abnormal motor pattern and postures caused by nonprogressive abnormal brain function.

The nursing student is helping to take care of the infant with myelomeningocele. Which of the following actions, if made by the student nurse, should the nurse intervene? a) The nursing student uses and overhead radiant heater to warm the baby b) The student keeps the perineum clean and dry c) The student puts an absorbent diaper on the baby to keep it dry d) The nursing student turns props the baby slightly to the side with a towel under the abdomen

Answer: C. Diapering is not done for infants with myelomeningocele in order to prevent putting pressure on the sac. An overhead warmer should be used with this baby for temperature control. It is important to keep the perineum clean and dry. The child can be propped onto the side with a towel slightly to prevent skin breakdown from being in the prone position for long periods of time.

The nurse is caring for the older child who has just undergone surgical correction of pectus excavatum. Which of the following order should the nurse anticipate doing all of the following except: a) Assessing and teaching correct use of PCA pump b) Assessing for symmetry of breath movement c) Keeping the patient on bedrest with bedside commode d) Teaching the child to avoid lying on her side

Answer: C. The child will be on strict bedrest for until the physician clears the child for activity. It is important that during this time the child keep from rolling on either side in order to keep the newly inserted bar in place. A PCA pump may be used in the older child to provide adequate pain relief. Assessing symmetry of breath movements would be important as pneumothorax is a complication of this surgery.

Tommy is a young child who is started walking early in life and usually is very active and happy. His mother tells you of a slow change that has happened to her son, and that he is less active than he has been. He now seems tired a lot and has difficulty doing things he used to do, such as running and playing. Which of the following would the nurse want to assess first? a) Check the child's back for dimpling or a tuft of hair at the base of the spine b) Assess the child's pain level and level of consciousness c) The child's ability to stand up and walk d) The presence of infantile reflexes

Answer: C. This child is presenting signs that most line up with a form of progressive muscular dystrophy, and it would be important for the nurse to follow up on the mother's claims that the child has difficulty ambulating and playing.

Which of the following actions of the pregnant woman would be most likely to affect the neuromuscular development of her unborn child in utero? A) The mother who says, "I drank a few alcoholic beverages in my second trimester, I just couldn't help it!" B) The 22 year old mother who started taking folic acid before she ever got pregnant C) The 27 year old mother who was involved in a car accident which caused her to go into labor D) The 19 year old who says, "I did drugs pretty bad, but I stopped as soon as I knew I was pregnant,"

Answer: D. Early in gestation, during 3-4 weeks of pregnancy, the neural tubes of the embryo begin to develop and differentiate. This is a critical time for the unborn child, and things like drug or alcohol use are most likely to cause developmental disorders during this time period.

A topic dermatitis or eczema in the infant is

Associated with allergy with a hereditary tendency

When assessing children at the pediatric clinic, which of the following would the nurse want to investigate further? A) The two year old who gained control of his head and neck muscles before he was able to walk B) The infant who has very brisk deep tendon reflexes C) The young child who trips and falls while running around D) The infant who demonstrates hypertonia soon after birth.

Answer: D. Hypertonia of muscles or hypotonia (high or low muscle tone) would be considered abnormal findings in children and could be indicative of an underlying problem. Myelination proceeds in a cephalocaudal and proximodistal fashion, so the child would be expected to gain head control before being able to walk. Brisk deep tendon reflexes, while abnormal in older children, is normal in young infants. Tripping and falling could cause injury in the child, but would overall be considered a normal finding as children are still gaining control of their muscle function.

The nurse expects to see all of the following when assessing the newborn infant except: a) Flexible Metatarsus adductus b) Pes Planus c) Internal tibial torsion d) Polydactyly

Answer: D. Polydactyly, or the presence of an extra digit, would be considered abnormal in a newborn child. Flexible metatarsus adductus, also known as in-toeing, would be expected in the newborn infant, and would resolve as the infant matures (page 833). Internal tibial torsion, or the slight bowing of the legs, would also be expected r/t in-utero positioning. Pes Planus, or flat-footedness also occurs in infants and disappears as the child matures.

Which of the following problems is most often associated with myelomeningocele? A. Biliary atresia B. Hydrocephalus C. Craniosynostosis D. Tracheoesophageal fistula

Answer: b. Hydrocephalus is a frequently associated anomaly in 80% to 90% of children.

Which of the following clinical manifestations of developmental dysplasia of the hip would be seen in the newborn? A. Lordosis B. Ortolani sign C. Trendelenburg sign D. Telescoping of the affected limb

Answer: b. In the newborn period, the dysplasia usually appears as hip joint laxity. During the Ortolani test, the examiner places forward pressure and then backward pressure on the trochanter. If the femoral head is felt to slip, dysplasia may be present. This test is most reliable from birth to 2 to 3 months.

A newborn with congenital clubfoot is being treated with successive casts. The parents ask why so many casts are required. The nurse should explain that: A. casts are needed for the traction. B. each cast is good for only 6 weeks. C. surgical intervention will not be necessary. D. They allow for gradual stretching of tight structures.

Answer: d. Serial casting is begun shortly after birth and before discharge from nursery. Successive casts allow for gradual stretching of skin and tight structures on the medial side of the foot. Manipulation and casting of the leg are repeated frequently (every week) to accommodate the rapid growth of early infancy.

While assessing the newborn infant, the nurse notices a dimple with a tuft of hair at the bottom of the spine. What is the nurse's priority action? a) Place the child in the prone position to protect the area b) Soak a sterile dressing and place it over the area c) Notify the physician d) Educate the family on what this means for the child

Answer: d. it would be important to educate the family on the presence of spina bifida occulta, a neural tube defect that typically is benign and is often considered a normal defect. It would be important to differentiate this form of spina bifida from the highly stigmatized forms of spina bifida such as myelomingocele.

Cystic fibrosis is an example of which type of inheritance? a) Multifactorial b) Autosomal dominant c) X-linked recessive d) Autosomal recessive

Autosomal recessive Correct Explanation: Cystic fibrosis is an autosomal recessive inherited condition. Huntington disease would be an example of an autosomal dominant inherited condition. Hemophilia is an X-linked recessive inherited condition. Cleft lip is a multifactorial inherited condition.

The nurse is caring for a 6-year-old child who has a history of febrile seizures and is admitted with a temperature of 102.2° F(39° C). The highest priority nursing intervention for this child would be which of the following? a) The nurse will encourage the child to do his or her own self-care. b) The nurse will institute safety precautions. c) The nurse will offer age appropriate activities. d) The nurse will provide family teaching related to the child's history.

B A child with an elevated temperature is at high risk for having seizures and therefore actions by the nurse include keeping the child in a safe situation to prevent any injury if the child should have a seizure.

The nurse is caring for a client who is receiving rituximab (Rituxan) for treatment of lymphoma. It is essential for the nurse to observe for which side effect? A. Alopecia B. Allergy C. Fever D. Chills

B. Allergy is the most common side effect. Although fever & chills are side effect of monoclonal antibody therapy, they would not take priority over an allergic response that could potentially involve the airway.

The nurse is collecting data from the caregivers of a child admitted with seizures. Which of the following statements indicates the child most likely had an absence seizure? a) "His arms had jerking movements in his legs and face." b) "He was just staring into space and was totally unaware." c) "He kept smacking his lips and rubbing his hands." d) "He usually is very coordinated, but he couldn't even walk without falling."

B Absence seizures rarely last longer than 20 seconds. The child loses awareness and stares straight ahead but does not fall. Absence seizures rarely last longer than 20 seconds. The child loses awareness and stares straight ahead but does not fall. Myoclonic seizures are characterized by a sudden jerking of a muscle or group of muscles, often in the arms or legs, without loss of consciousness. Complex partial seizures cause nonpurposeful movements, such as hand rubbing and lip smacking. During the prodromal period of the tonic-clonic seizure, the child might have a lack of coordination.

Absence seizures are marked by which of the following clinical manifestations? a) Sudden, brief jerks of a muscle group b) Loss of motor activity accompanied by a blank stare c) Loss of muscle tone and loss of consciousness d) Brief, sudden onset of increased tone of the extensor muscle

B An absence seizure consists of a sudden, brief arrest of the child's motor activity accompanied by a blank stare and loss of awareness. A tonic seizure consists of a brief onset of increased tone or muscle. A myoclonic seizure is characterized by sudden, brief jerks of muscle groups. An atonic seizure involves a sudden loss of muscle tone and loss of consciousness.

Which statement about cerebral palsy would be accurate? a) "Cerebral palsy occurs because of too much oxygen to the brain." b) "Cerebral palsy is a condition that doesn't get worse." c) "Cerebral palsy means there will be many disabilities." d) "Cerebral palsy is a condition that runs in families."

B By definition, cerebral palsy is a nonprogressive neuromuscular disorder. It can be mild or quite severe and is believed to be the result of a hypoxic event during pregnancy or the birth process and doesn't run in families

Which of the following is consistent with increased ICP in the child? a) Increased appetite b) Bulging fontanel c) Emotional lability d) Narcolepsy

B Children with increased ICP exhibit bulging fontanels. They typically have a decreased appetite, are restless, and have trouble sleeping.

The treatment for children with seizures disorders is most often which of the following? a) Restricted fat diet b) Use of anticonvulsant medications c) Strict exercise regimen d) Surgical intervention

B Complete control of seizures can be achieved for most people through the use of anticonvulsant drug therapy. A few children may be candidates for surgical intervention but, in most cases, surgery is not the treatment. Ketogenic diets (high in fat and low in carbohydrates and protein) cause the child to have high levels of ketones, which help to reduce seizure activity. Exercise is not a treatment for seizure disorders.

A child is diagnosed with bacterial meningitis. The nurse would suspect which abnormality of cerebrospinal fluid (CSF)? a) Decreased pressure b) Cloudy appearance c) Elevated sugar d) Decreased leukocytes

B In the CSF of clients diagnosed with bacterial meningitis, the pressure is elevated, the appearance is cloudy, and the leukocytes are elevated. A decreased sugar content is noted.

The nurse is preparing a toddler for a lumbar puncture. For this procedure, the nurse should place the child in which position? a) Lying prone, with the feet higher than the head b) Lying on one side, with the back curved c) Lying prone, with the neck flexed d) Sitting up, with the back straight

B Lumbar puncture involves placing a needle between the lumbar vertebrae into the subarachnoid space. For this procedure, the nurse should position the client on one side with the back curved because curving the back maximizes the space between the lumbar vertebrae, facilitating needle insertion. Prone and seated positions don't achieve maximum separation of the vertebrae.

Which nursing action should be included in the care plan to promote comfort in a 4-year-old child hospitalized with meningitis? a) Rock the child frequently b) Avoid making noise when in the child's room c) Have the child's 2-year-old brother stay in the room d) Keep the lights on brightly so that he can see his mother

B Meningeal irritation may cause seizures and heightens a child's sensitivity to all stimuli, including noise, lights, movement, and touch. Frequent rocking, presence of a younger sibling, and bright lights would increase stimulation.

The mother of a 12-year-old with Reye syndrome approaches the nurse wanting to know how this happened to her child, saying, "I never give my kids aspirin!" What could the nurse say to begin educating the woman? a) "Don't worry; you're in good hands. We have it under control now." b) "Sometimes it's hard to tell what products may contain aspirin." c) "Do you think that maybe your child took aspirin on his own?" d) "Aspirin in combination with the virus will make the brain swell and the liver fail."

B Salicylates are in a wide variety of products, so consumers must read the small print very carefully or they will miss the warning. The parent needs to be receptive to further education, and raising the possibility the child was responsible does not accomplish that goal. Don't state the obvious, but also don't minimize the situation. Encouraging the mother to ask for information and offering explanations in terms she will understand are important, but this response does not address the mother's assertion.

Any individual taking phenobarbital for a seizure disorder should be taught a) to brush his or her teeth four times a day. b) never to discontinue the drug abruptly. c) never to go swimming. d) to avoid foods containing caffeine.

B Phenobarbital should always be tapered, not stopped abruptly, or seizures from the child's dependency on the drug can result.

The nurse is educating the family of a 7-year-old epilepsy patient about care and safety for this child. Which of the following comments will be most valuable in helping the parent and the child cope? a) "If he is out of bed, the helmet's on the head." b) "Use this information to teach family and friends." c) "You'll always need a monitor in his room." d) "Bike riding and swimming are just too dangerous."

B Families need and want information they can share with relatives, childcare providers, and teachers. Wearing a helmet and having a monitor in the room are precautions that may need to be modified as the child matures. The boy may be able to bike ride and swim with proper precautions.

The nurse and an adolescent are reviewing the adolescent's record of her headaches and activities surrounding them. Which of the following would the nurse identify as a possible trigger? a) Use of nonscented soap b) Drinking three cans of diet cola c) Swimming twice a week d) 11 p.m. bedtime; 6:30 a.m. wake-up

B Cola contains caffeine, which is an associated trigger. Intense activity, not regular exercise, may be a trigger. Odors, such as strong perfumes, may be a trigger. Changes in sleeping patterns may be a trigger.

The nurse is caring for a 6-year-old child who has a history of febrile seizures and is admitted with a temperature of 102.2° F(39° C). The highest priority nursing intervention for this child would be which of the following? a) The nurse will encourage the child to do his or her own self-care. b) The nurse will institute safety precautions. c) The nurse will offer age appropriate activities. d) The nurse will provide family teaching related to the child's history

B A child with an elevated temperature is at high risk for having seizures and therefore actions by the nurse include keeping the child in a safe situation to prevent any injury if the child should have a seizure.

A school-aged girl with seizures is prescribed phenytoin sodium, 75 mg four times a day. An instruction you would want to give her parents regarding this is a) watching television while taking the drug may cause seizures. b) their child will have to practice good tooth brushing. c) even small doses may cause noticeable dizziness. d) numbness of the fingers is common while taking this drug.

B A side effect of phenytoin sodium is hypertrophy of the gumline. Good tooth brushing helps prevent inflammation under the hypertrophied tissue.

Dexamethasone is often prescribed for the child who has sustained a severe head injury. Dexamethasone is a(n) a) antihistamine. b) steroid. c) anticonvulsant. d) diuretic.

B A steroid may be prescribed to reduce inflammation and pressure on vital centers.

Absence seizures are marked by which of the following clinical manifestations? a) Brief, sudden onset of increased tone of the extensor muscle b) Loss of motor activity accompanied by a blank stare c) Sudden, brief jerks of a muscle group d) Loss of muscle tone and loss of consciousness

B An absence seizure consists of a sudden, brief arrest of the child's motor activity accompanied by a blank stare and loss of awareness. A tonic seizure consists of a brief onset of increased tone or muscle. A myoclonic seizure is characterized by sudden, brief jerks of muscle groups. An atonic seizure involves a sudden loss of muscle tone and loss of consciousness.

The mother of a newborn with a caput succedaneum asks the nurse how this happened to her baby. Which response by the nurse would be most appropriate? a) "The forceps used during delivery caused this to happen." b) "During delivery, your vaginal wall put pressure on the baby's head." c) "It's normal for this to happen, but they don't really know why." d) "Your baby's head became blocked inside your vagina while you were pushing."

B Caput succedaneum results from pressure from the uterus or vaginal wall during a head-first delivery The use of forceps is associated with a cephalohematoma. Caput succedaneum is not due to the baby's head becoming blocked inside the vagina. The cause of caput succedaneum is known; it is caused by pressure from the uterus or vaginal wall during a head-first delivery.

The eyes of a 9-year-old who suffered a head injury are crossed. Besides checking ICP, which of the following interventions would be most important for the nurse to perform? a) Help the child cope with an altered appearance. b) Assess the child's level of consciousness. c) Monitor core body temperature. d) Pull up the side rails on the bed.

B Decreased level of consciousness is frequently the first sign of major neurologic problems after head trauma. While body temperature is an important indicator of infection, it is not a priority here. Preventing harm by setting the side rails is more important for a seizure patient. The child's eyes will correct themselves when ICP is reduced.

The nurse is discussing with a parent the difference between a breath-holding spell and a seizure. The nurse would be correct in telling the parent which of the following with regards to seizures? a) The patient is bradycardiac. b) Convulsive activity occurs. c) Cyanosis occurs at the onset of the seizure. d) The EEG is normal.

B During seizures convulsive activity is typically noted. During a breath-holding spell, the child is bradycardiac, cyanosis occurs at the onset, and the EEG is normal.

A child is diagnosed with bacterial meningitis. The nurse would suspect which abnormality of cerebrospinal fluid (CSF)? a) Elevated sugar b) Cloudy appearance c) Decreased leukocytes d) Decreased pressure

B In the CSF of clients diagnosed with bacterial meningitis, the pressure is elevated, the appearance is cloudy, and the leukocytes are elevated. A decreased sugar content is noted

A 1-year-old has just undergone surgery to correct craniosynostosis. Which of the following comments is the best psychosocial intervention for the parents? a) "I told you yesterday there would be facial swelling." b) "The surgery was successful. Do you have any questions?" c) "I'll be watching hemoglobin and hematocrit closely." d) "This only happens in 1 out of 2,000 births."

B Often what parents need most is someone to listen to their concerns. Although this is a good time for education, let the parents adjust to their baby's appearance and adapt your teaching to their questions, comments, and knowledge level.

The nurse is educating parents of a male infant with Chiari type II malformation. Which of the following statements about their child's condition is most accurate? a) "Lay him down after feeding." b) "Take your time feeding your baby." c) "You won't need to change diapers often." d) "You'll see a big difference after the surgery."

B One of the problems associated with Chiari type II malformation is poor gag and swallowing reflexes, so the infant must be fed slowly. There is a great risk of aspiration, requiring that the child be placed in an upright position after feeding. The goal of surgery is to prevent further symptoms, rather than to relieve existing ones. Infrequent urination is a problem associated with type I malformations.

The nurse is caring for an 8-year-old girl who was in a car accident. Which symptom suggests the child has a cerebral contusion? a) Bleeding from the ear b) Trouble focusing when reading c) Vomiting d) Difficulty concentrating

B Signs and symptoms for cerebral contusions include disturbances to vision, strength, and sensation. A child suffering a concussion will be distracted and unable to concentrate. Vomiting is a sign of a subdural hematoma. Bleeding from the ear is a sign of a basilar skull fracture.

The nurse is interviewing the caregivers of a child brought to the emergency unit. The caregiver states, "She has a history of seizures but this time it lasted more than 30 minutes and she just keeps having them." The most accurate description of this child's condition would be which of the following? a) The child's history indicates she has infantile seizures. b) The child is in status epilepticus. c) The child is having generalized seizures. d) The child may begin to have absence seizures every day.

B Status epilepticus is the term used to describe a seizure that lasts longer than 30 minutes or a series of seizures in which the child does not return to his or her previous normal level of consciousness. The child likely is having generalized seizures, but the most accurate description of what is happening is status epilepticus. With infantile spasms, muscle contractions are sudden, brief, symmetrical, and accompanied by rolling eyes. With absence seizures the child loses awareness and stares straight ahead but does not fall.

When assisting a child while she is having a tonic-clonic seizure, it would be important to a) turn the child onto her back and observe her. b) protect the child from hitting her arms against furniture. c) place a tongue blade between the child's teeth. d) restrain the child from all movement.

B protect the child from hitting her arms against furniture.

A nurse is assessing a 3-year-old child for possible bacterial meningitis. Which sign would indicate irritation of the meninges? a) Negative Kernig's sign b) Positive Kernig's sign c) Positive Homans' sign d) Negative Brudzinski's sign

B A positive Kernig's sign can indicate irritation of the meninges. A positive Brudzinski's sign also is indicative of the condition. A positive Homans' sign may indicate venous inflammation of the lower leg.

When assessing a neonate for seizures, which of the following would the nurse expect to find? Select all that apply. a) Tonic-clonic contractions b) Elevated blood pressure c) Ocular deviation d) Jitteriness e) Tachycardia

B C D E Neonatal seizures may be difficult to recognize but may be manifested by tremors, jitteriness, tachycardia and elevated blood pressure, and ocular deviation. Tonic-clonic contractions typically are more common in older children.

The mother of a 12-year-old with Reye syndrome approaches the nurse wanting to know how this happened to her child, saying, "I never give my kids aspirin!" What could the nurse say to begin educating the woman? a) "Aspirin in combination with the virus will make the brain swell and the liver fail." b) "Sometimes it's hard to tell what products may contain aspirin." c) "Do you think that maybe your child took aspirin on his own?" d) "Don't worry; you're in good hands. We have it under control now."

B Salicylates are in a wide variety of products, so consumers must read the small print very carefully or they will miss the warning. The parent needs to be receptive to further education, and raising the possibility the child was responsible does not accomplish that goal. Don't state the obvious, but also don't minimize the situation. Encouraging the mother to ask for information and offering explanations in terms she will understand are important, but this response does not address the mother's assertion.

The best way to evaluate a child's level of consciousness is through conversation. a) False b) True

B The best way to evaluate a child's level of consciousness is through conversation. Note any drowsiness or lethargy. Allow the child to answer questions without prompting, and listen carefully to be certain the answer is appropriate to the question.

What is the primary treatment for children with Non-Hodgkin Lymphoma? A) Radiation B) Multiagent Chemotherapy C) Stem Cell Transplant D) Hyperthermia therapy Correct Answer: B

B) Multiagent Chemotherapy

The nurse is teaching a group of clients about cancers related to tobacco or tobacco smoke. Identify the common cancers related to tobacco use. Select all that apply. A. Cardiac cancer B. Lung cancer C. Cancer of the tongue D. Skin cancer E. Cancer of the larynx

B,C,E: The heart does not contain cells that divide; therefore cardiac cancer is unlikely.

When caring for a client who has had a colostomy created as part of a regimen to treat colon cancer, which activities would help to support the client in accepting changes in appearance or function? Select all that apply. A. Explain to the client that the colostomy is only temporary. B. Encourage the client to participate in changing the ostomy. C. Obtain a psychiatric consultation. D. Offer to have a person who is coping with a colostomy visit. E. Encourage the client and family members to express their feelings and concerns

B,D,E

The nurse is instructing a client on how to perform breast self-examination (BSE). Which techniques will the nurse include in teaching the client about BSE? Select all that apply. A. Instruct the client to keep her arm by her side while performing the examination. B. Ensure that the setting in which BSE is demonstrated is private and comfortable. C. Ask the client to remove her shirt. The bra may be left in place. D. Ask the client to demonstrate her own method of BSE. E. Use the fingertips, which are more sensitive than the finger pads, to palpate the breasts.

B,D: The setting should be private and comfortable to promote an environment conducive to learning and to prevent potential client embarrassment. Before teaching breast palpation, ask the client to demonstrate her own method, so that the nurse can assess the client's understanding of BSE. For better visualization, the arm should be placed over the head.The client should undress from the waist up. The finger pads, which are more sensitive than the fingertips, are used when palpating the breasts.

The nurse is conducting a community health education class on diet and cancer risk reduction. What should be included in the discussion? Select all the apply. A. Limit sodium intake. B. Avoid beef and processed meats. C. Increase consumption of whole grains. D. Eat "colorful fruits and vegetables," including greens. E. Avoid gas-producing vegetables such as cabbage.

B-D. Eating cruciferous vegetables such as broccoli, cauliflower, brussels sprouts, and cabbage may reduce cancer risk.

A 5 year old a has temperature of 103.6 'F and is brought into the emergency room by his mother. Which statement by the mother causes concern? A. "I've tried to encourage fluid intake every hour." B. "I administered Aspirin to help with the fever a few hours ago." C. "I re-took his temperature 30 minutes after I gave the medication and it was still high." D. "I gave him a sponge bath to help with the fever."

B. A child should never have aspirin, especially for a fever due to Reyes Syndrome.

A 7 year old has been having vomiting with diarrhea for 3 days. How do you expect the child to present clinically? A. Sunken eyeballs and bradycardia B. None of the options are correct. C. Bradycardia, dry mucous membranes, absence of tears D. Tachycardia, dry mucous membranes, weight loss

B. The child should be experiencing dehydration and would present clinically with: Tachycardia, dry mucous membranes, weight loss, sunken eyes and/or fontanles, decreased urinary output

A nurse is caring for a toddler who has rhinitis, cough, and diarrhea for 2 days. Upon assessment it is noted that the tympanic membrane has an orange discoloration and decreased movement. Which of the following is an appropriate statement for the nurse to make? A. "your child has an ear infection that requires antibiotics" B. "your child could experience transient hearing loss" C. "your child will need to be on a decongestant until this clears up" D. "your child will need to have a myringotomy"

B. "your child could experience transient hearing loss"

The client with prostate cancer asks why he must have surgery instead of radiation, even if it is the least invasive type. What is the nurse's best response? A."It is because your cancer growth is large." B. "Surgery is the most common intervention to cure the disease." C. "Surgery slows the spread of cancer." D. "The surgery is to promote urination."

B. Because some localized prostate cancers are resistant to radiation, surgery is the most common intervention for a cure.

When teaching women about the risk of breast cancer, which risk factor does the nurse know is the most common for the development of the disease? A. Having an aunt with breast cancer B. Being an older adult C. Being a Euro-American D. Consuming a low-fat diet

B. Being an older woman or man is the primary risk factor, although some people are at higher risk than others. There is no single-known cause for breast cancer. Having a first-degree relative (mother, sister, or daughter) with breast cancer can increase the risk; an aunt is not considered a first-degree relative. Although Euro-American women older than 40 years are at a more increased risk than other racial/ethnic groups, the greater risk is being an older adult. Consuming a high-fat diet is considered a risk factor.

2) An infant's PKU test is positive. The nurse's priority is to: A. Provide PKU education to the family members B. Change the infant's formula to Lofenlac C. Obtain an order for packed red cells D. Monitor vital signs every 30 minutes

B. Change the infant's formula to Lofenlac

The nurse is assessing a client with lung cancer. Which symptom does the nurse anticipate finding? A. Easy bruising B. Dyspnea C. Night sweats D. Chest wound

B. Dyspnea is a sign of lung cancer, as are cough, hoarseness, shortness of breath (SOB), bloody sputum, arm or chest pain, and dysphagia.Night sweats is a symptom of the lymphomas.

Which manifestation of an oncologic emergency requires the nurse to contact the health care provider immediately? A. New onset of fatigue B. Edema of arms and hands C. Dry cough D. Weight gain

B. Edema of the arms and hands indicates worsening compression of the superior vena cava consistent with superior vena cava syndrome. The compression must be relieved immediately, often with radiation therapy, because death can result without timely intervention.

Myoclonic seizures are characterized by A. Focal electricqal hyperactivity in the brain B. Generalized EEG abnormalities with gerks C. A form of partial seizures D. Secondary generalized seizures

B. Generalized EEG

A client with advanced cirrhosis has been diagnosed with hepatic encephalopathy. The nurse expects to assess for: A. Malaise B. Hand tremors C. Weight loss D. Stomatitis

B. Hand tremors

Spironolacctone (Aldactone) is prescribed for a client with chronic cirrhosis and ascites. The nurse should monitor the client for which of the following medication-related side effects? A. Tachycardia B. Hyperkalemia C. Constipation D. Jaundice

B. Hyperkalemia

A school nurse identifies head lice in an 8-year-old student. Which concept should be included when teaching the family about treating this condition? A. Daily washing of hair will prevent the recurrence of the lice. B. If the nit cannot be removed from the hair shaft, the hair shaft should be cut. C. Clothing and towels should be washed in cold water. D. Conditioner should be applied prior to lice treatment.

B. If the nit cannot be removed from the hair shaft, the hair shaft should be cut.

The client is treated for head lice with lindane (Kwell). Following treatment, the nurse reinforces instructions to: A. Remain isolated for 48 hours. B. Inspect hair shafts, checking for nits daily, for one week following treatment. C. Shampoo with Kwell three times per week. D. Wash linens with cold water and bleach.

B. Inspect hair shafts, checking for nits daily, for one week following treatment.

What is the spot called, located between the right anterior superior iliac crest and the umbilicus where pain is felt most when a child has appendicitis? A. Kehrs Point B. McBurney Point C. McMurray's Point D. Ilial Umbilical Point

B. McBurney Point

You palpate a large left intra-abdominal mass in an infant. The most likely diagnosis is: A. Wilm's tumor B. Neuroblastoma C. Autosomal recessive polycystic kidney disease (ARPKD)

B. Neuroblastoma

The client has been diagnosed with breast cancer. Which treatment option chosen by the client requires the nurse to discuss with the client the necessity of considering additional therapy? A. Chemotherapy B. Complementary and alternative therapy (CAM) C. Hormonal therapy D. Neoadjuvant therapy

B. No proven benefit has been found with using complementary and alternative therapy alone as a cure for breast cancer. The nurse must ensure that the client's choices can be safely integrated with conventional treatment for breast cancer.

Which medication does the nurse plan to administer to a client before chemotherapy to decrease the incidence of nausea? A. Morphine B. Ondansetron (Zofran) C. Naloxone (Narcan) D. Diazepam (Valium)

B. Ondansetron is a 5-HT3 receptor blocker that blocks serotonin to prevent nausea and vomiting. Diazepam is a benzodiazepine, which is an antianxiety medication only. Lorazepam, a benzodiazepine, may be used for nausea.

Which precaution is most important for the nurse to teach a client with leukemia to prevent an infection by autocontamination? A. Take antibiotics exactly as prescribed. B. Perform mouth care three times daily. C. Avoid the use of pepper and raw foods. D. Report any burning on urination immediately.

B. Perform mouth care 3 times a day Autocontamination is the overgrowth of the client's own normal flora or the translocation of his or her normal flora from its normal location to a different one. Performing frequent mouth care can reduce the number of normal flora organisms in the mouth and decrease the risk for developing an infection from autocontamination. Taking antibiotics does not prevent autocontamination, nor does reporting symptoms of an infection. Avoiding exposure to environmental organisms does not prevent autocontamination.

Which activity performed by the community health nurse best reflects primary prevention of cancer? A. Assisting women to obtain free mammograms B. Teaching a class on cancer prevention C. Encouraging long-term smokers to get a chest x-ray D. Encouraging sexually active women to get annual (Pap) smears

B. Primary prevention involves avoiding exposure to known causes of cancer; education assists clients with this strategy. all the other options are secondary levels of prevention.

The nurse is caring for a 4 year old with cerebral palsy. Which nursing intervention will help ready the child for rehabilitative services? A. Patching one of the eyes to strengthen the muscles B. Providing suckers and pinwheels to strengthen tongue movement C. Providing musical tapes to provide auditory training D. Encouraging paly with a video game to improve muscle coordination

B. Providing suckers and pinwheels to strengthen tongue movement

Nursing care during a seizure include all, except A. Ease the patient to the floor if possible B. Restrain the patient during the seizure to prevent from falling C. Protect the head with a pad to prevent injury D. Loosen constrictive clothing E. If patient is in bed, remove pillows and raise the side rails

B. Restrain the patient during the seizure to prevent from falling

The nurse reviews the chart of the client admitted with a diagnosis of glioblastoma with a T1NXM0 classification. Which explanation does the nurse offer when the client asks what the terminology means? A. "Two lymph nodes are involved in this tumor of the glial cells, and another tumor is present." B. "The brain tumor measures about 1 to 2 cm and shows no regional lymph nodes and no distant metastasis." C. "This type of tumor in the brain is small with some lymph node involvement; another tumor is present somewhere else in your body." D. "Glioma means this tumor is benign, so I will have to ask your health care provider the reason for the chemotherapy and radiation."

B. T1 means that the tumor is increasing in size to about 2 cm, and that no regional lymph nodes are present in the brain. M0 means that no distant metastasis has occurred.

The nurse is caring for a patient receiving an initial dose of chemotherapy to treat a rapidly growing metastatic colon cancer. The nurse is aware that this patient is at risk for tumor lysis syndrome (TLS) and will monitor the patient closely for which of the following abnormalities associated with this oncologic emergency? A. Hypokalemia B. Hypocalcemia C. Hypouricemia D. Hypophosphatemia

B. TLS is a metabolic complication characterized by rapid release of intracellular components in response to chemotherapy. This can rapidly lead to acute renal failure. The hallmark signs of TLS are hyperuricemia, hyperphosphatemia, hyperkalemia, and hypocalcemia.

The client who has recently had breast cancer surgery requests a volunteer to visit her home to help with recovery. Which community resource will the nurse recommend? A. National Breast Cancer Coalition B. Reach for Recovery C. Susan G. Komen for the Cure D. Young Survival Coalition

B. The American Cancer Society's program "Reach for Recovery" provides volunteers who visit clients in the hospital or at home. They bring personal messages of hope, informational materials on breast cancer recovery, and a soft, temporary breast form.

Which of these does the nurse recognize as the goal of palliative surgery for the client with cancer? A. Cure of the cancer B. Relief of symptoms or improved quality of life C. Allowing other therapies to be more effective D. Prolonging the client's survival time

B. The focus of palliative surgery is to improve quality of life during the survival time.

Which client problem does the nurse set as the priority for the client experiencing chemotherapy-induced peripheral neuropathy? A. Potential for lack of understanding related to side effects of chemotherapy B. Risk for Injury related to sensory and motor deficits C. Potential for ineffective coping strategies related to loss of motor control D. Altered sexual function related to erectile dysfunction

B. The highest priority is safety.

The registered nurse is teaching a group of nursing students about malignant transformation. Which statement about the process of malignant transformation is true? A. Mutation of genes is an irreversible event that always leads to cancer development in the initiation phase. B. Insulin and estrogen enhance the division of an initiated cell during the promotion phase. C. Tumors form when carcinogens invade the gene structure of the cell in the latency phase. D. Nutrition of cancer cells is provided by tumor angiogenesis factor (TAF) in the promotion stage.

B. These promoters increase cell division. If cell division is halted, this does not lead to cancer development in the initiation phase.In the initiation phase, carcinogens invade the DNA of the nucleus of a single cell. A 1-cm tumor consists of 1 billion cells. The latent phase occurs between initiation and tumor formation. promotion phase consists of progression when the blood supply changes from diffusion to TAF.

A child returns to school following a 3- week absence due to mononucleosis. The school nurse knows it will be important for the client: A. To drink additional fluids throughout the day B. To avoid contact sports for 1-2 months C. To have a snack twice a day to prevent hypoglycemia D. To continue antibiotic therapy for 5 months

B. To avoid contact sports for 1-2 months

The nurse anticipates administering which medication to treat hyperuricemia associated with tumor lysis syndrome (TLS)? A.Recombinant erythropoietin (Procrit) B. Allopurinol (Zyloprim) C. Potassium chloride D. Radioactive iodine 131

B. Tumor lysis syndrome results in hyperuricemia, Allopurinol decreases uric acid production and is indicated in TLS.

The nurse suspects metastasis from left breast cancer to the thoracic spine when the client has which symptom? A. Vomiting B. Back pain C. Frequent urination D. Cyanosis of the toes

B. Typical sites of breast cancer metastasis include bone, manifested by back pain, lung, liver, and brain. Signs of metastasis to the spine may include numbness, pain, paresthesias and tingling, and loss of bowel and bladder control.

Which action can the same-day surgery charge nurse delegate to an experienced unlicensed assistive personnel (UAP) who is helping with the care of a client who is having a breast biopsy? A. Assess anxiety level about the surgery. B. Monitor the vital signs after surgery. C. Obtain data about breast cancer risk factors. D. Teach about postoperative routine care.

B. Vital sign assessment is included in nursing assistant education and usually is part of the job description for UAP working in a hospital setting.

Which action is most important for the nurse to implement to prevent nausea and vomiting in a client who is prescribed to receive the first round of IV chemotherapy? A. Keep the client NPO during the time chemotherapy is infusing. B. Administer antiemetic drugs before administering chemotherapy. C. Ensure that the chemotherapy is infused over a 4- to 6-hour period. D. Assess the client for manifestations of dehydration hourly during the infusion period.

B. When emetogenic chemotherapy drugs are prescribed, the client should receive antiemetic drugs before the chemotherapy drugs are administered. This allows time for prevention of chemotherapy-associated nausea and vomiting; however, the antiemetic therapy cannot stop until all risks for nausea and vomiting have passed. Clients become nauseated and vomit even if they are NPO.

A student with type I diabetes mellitus complains of feeling lightheaded. Her blood sugar is 60 mg/dL. Using the 15/15 rule, the nurse should: A. give 15 mL of juice, and repeat does in 15 minutes B. give 15 grams of carbohydrates and retest blood sugar in 15 minutes C. Give 15 grams of carbohydrates and 15 g of protein D. Give 15 ounces of juice and retest blood sugar in 15 minutes

B. give 15 grams of carbohydrates and retest blood sugar in 15 minutes 15/15 rule states to give 15 g of carbohydrates (approx 60 calories; roughly 4 oz of juice or a tablespoon of honey or sugar). Protein by itself will treat hypoglycemia. Only 15 mL of juice does not contain enough calories to increase the blood sugar. Fifteen oz of juice nearly 4 times the recommended amount

After a client is admitted to the pediatric unit with a diagnosis of acute lymphocytic leukemia, the laboratory test indicates that the client is neutropenic. The nurse should perform which of the following?" A. advise the client to rest and avoid exertion B. prevent client exposure ot infections C. monitor the blood pressure frequently D. observe for increased bruising

B. prevent client exposure ot infections

When caring for a male client diagnosed with a brain tumor of the parietal lobe, the nurse expects to assess: A. short-term memory impairment. B. tactile agnosia. C. seizures. D. contralateral homonymous hemianopia.

B. tactile agnosia

During the physical assessment of a 2½-month-old infant, the nurse suspects the child may have hydrocephalus. Which of the following signs or symptoms was observed? a) Pupil of one eye dilated and reactive b) Vertical nystagmus c) Dramatic increase in head circumference d) Posterior fontanel is closed

C A dramatic increase in head circumference is a symptom of hydrocephalus, suggesting that there is a build-up of fluid in the brain. Only one pupil that is dilated and reactive is a sign of an intracranial mass. Vertical nystagmus indicates brain stem dysfunction. A closed posterior fontanel would be frequently seen by this age.

The nurse is caring for a child who had a seizure, fell to the ground, and hit and injured his face, head, and shoulders. This information indicates the child likely had which of the following types of seizures? a) Absence b) Myoclonic c) Atonic d) Infantile

C Atonic or akinetic seizures cause a sudden momentary loss of consciousness, muscle tone, and postural control and can cause the child to fall. They can result in serious facial, head, or shoulder injuries. In absence seizures the child loses awareness and stares straight ahead but does not fall. Myoclonic seizures are characterized by a sudden jerking of a muscle or group of muscles, often in the arms or legs, without loss of consciousness. With infantile spasms, muscle contractions are sudden, brief, symmetrical, and accompanied by rolling eyes.

A 6-month-old infant is admitted with suspected bacterial meningitis. She is crying, irritable, and lying in the opisthotonic position. Which of the following interventions should the nurse take initially? a) Encourage the mother to hold and comfort the infant. b) Educate the family about preventing bacterial meningitis. c) Institute droplet precautions in addition to standard precautions. d) Palpate the child's fontanels.

C Bacterial meningitis is a medical emergency. The child must be placed on droplet precautions until 24 hours of antibiotics have been given. Encouraging the mother to hold and comfort the child is an intervention but not the priority one: the focus is to get the infant the appropriate medications to fight the infection and to prevent its spread. Educating the family about preventing bacterial meningitis would be appropriate later on once the initial infection has been controlled. Palpating the fontanels is used to assess for hydrocephalus.

In caring for the child with meningitis, the nurse recognizes that which of the following nursing diagnoses would be the most important to include in this child's plan of care? a) Delayed growth and development related to physical restrictions b) Risk for acute pain related to surgical procedure c) Risk for injury related to seizure activity d) Ineffective airway clearance related to history of seizures

C Keeping the child free of injury would be an appropriate nursing diagnosis. Surgery is not indicated for the child with meningitis, and the history of seizures does not impact the airway clearance. Growth and development issues are a concern but not likely delayed due to this diagnosis.

A 1-year-old has just undergone surgery to correct craniosynostosis. Which of the following comments is the best psychosocial intervention for the parents? a) "This only happens in 1 out of 2,000 births." b) "I'll be watching hemoglobin and hematocrit closely." c) "The surgery was successful. Do you have any questions?" d) "I told you yesterday there would be facial swelling."

C Often what parents need most is someone to listen to their concerns. Although this is a good time for education, let the parents adjust to their baby's appearance and adapt your teaching to their questions, comments, and knowledge level.

Signs of increased intracranial pressure for which you would assess are a) decreased level of consciousness, increased respiratory rate. b) numbness of fingers, decreased temperature. c) increased temperature, decreased respiratory rate. d) increased pulse rate, decreased blood pressure.

C Pressure on the vital-sign centers causes an elevated temperature and a decreased respiratory rate. Blood pressure increases; pulse decreases.

The parents of a child with a history of seizures who has been taking phenytoin (Dilantin) ask the nurse why it's difficult to maintain therapeutic plasma levels of this medication. Which statement by the nurse would be most accurate? a) "A drop in the plasma drug level will lead to a toxic state." b) "The capacity to metabolize the drug becomes overwhelmed over time." c) "Small increments in dosage lead to sharp increases in plasma drug levels." d) "Large increments in dosage lead to a more rapid stabilizing therapeutic effect."

C Within the therapeutic range for phenytoin, small increments in dosage produce sharp increases in plasma drug levels. The capacity of the liver to metabolize phenytoin is affected by slight changes in the dosage of the drug, not necessarily the length of time the client has been taking the drug. Large increments in dosage will greatly increase plasma levels, leading to drug toxicity.

Which of the following children is not at risk for developing osteosarcoma? A. A child who received radiation therapy for a previous illness. B. A child who underwent a rapid growth spurt. C. A child who has broken the same bone twice. D. A child who has a family member that was diagnosed with osteosarcoma.

C. A child who has broken the same bone twice.

During the physical assessment of a 2½-month-old infant, the nurse suspects the child may have hydrocephalus. Which of the following signs or symptoms was observed? a) Posterior fontanel is closed b) Vertical nystagmus c) Dramatic increase in head circumference d) Pupil of one eye dilated and reactive

C A dramatic increase in head circumference is a symptom of hydrocephalus, suggesting that there is a build-up of fluid in the brain. Only one pupil that is dilated and reactive is a sign of an intracranial mass. Vertical nystagmus indicates brain stem dysfunction. A closed posterior fontanel would be frequently seen by this age

Which nursing assessment data should be given the highest priority for a child with clinical findings related to meningitis? a) Occurrence of urine and fecal contamination b) Degree and extent of nuchal rigidity c) Signs of increased intracranial pressure (ICP) d) Onset and character of fever

C Assessment of fever and evaluation of nuchal rigidity are important aspects of care, but assessment for signs of increasing ICP should be the highest priority due to the life-threatening implications. Urinary and fecal incontinence can occur in a child who's ill from nearly any cause but doesn't pose a great danger to life.

A 6-month-old infant is admitted with suspected bacterial meningitis. She is crying, irritable, and lying in the opisthotonic position. Which of the following interventions should the nurse take initially? a) Educate the family about preventing bacterial meningitis. b) Encourage the mother to hold and comfort the infant. c) Institute droplet precautions in addition to standard precautions. d) Palpate the child's fontanels.

C Bacterial meningitis is a medical emergency. The child must be placed on droplet precautions until 24 hours of antibiotics have been given. Encouraging the mother to hold and comfort the child is an intervention but not the priority one: the focus is to get the infant the appropriate medications to fight the infection and to prevent its spread. Educating the family about preventing bacterial meningitis would be appropriate later on once the initial infection has been controlled. Palpating the fontanels is used to assess for hydrocephalus.

Which of the following is consistent with increased ICP in the child? a) Narcolepsy b) Emotional lability c) Bulging fontanel d) Increased appetite

C Children with increased ICP exhibit bulging fontanels. They typically have a decreased appetite, are restless, and have trouble sleeping

The nurse is caring for a child admitted with complex partial seizures. Which of the following clinical manifestations would likely have been noted in the child with this diagnosis? a) The child was dizzy and had decreased coordination. b) The child had jerking movements and then the extremities stiffened. c) The child was rubbing the hands and smacking the lips. d) The child had shaking movements on one side of the body.

C Complex partial seizures, also called psychomotor seizures, change or alter consciousness. They cause memory loss and staring and nonpurposeful movements, such as hand rubbing, lip smacking, arm dropping, and swallowing. In the tonic phase of tonic-clonic seizures, the child's muscles contract, the child may fall, and the child's extremities may stiffen. During the prodromal period of the tonic-clonic seizure, the child might have a lack of coordination. Simple partial motor seizures cause a localized motor activity such as shaking of an arm, leg, or other part of the body

The nurse is caring for a near-term pregnant woman who has not taken prenatal vitamins or folic acid supplements. Which congenital defect is most likely to occur based on the mother's prenatal history? a) Neonatal conjunctivitis b) Incomplete myelinization c) A neural tube defect d) Facial deformities

C Folic acid supplementation has been found to reduce the incidence of neural tube defects by 50%. The fact that the mother has not used folic acid supplements puts her baby at risk for spina bifida occulta, one type of neural tube defect. Neonatal conjunctivitis can occur in any newborn during birth and is caused by virus, bacteria, or chemicals. Facial deformities are typical of babies of alcoholic mothers. Incomplete myelinization is present in all newborns.

An 8-year-old girl is diagnosed as having tonic-clonic seizures. You would want to teach her parents that a) their daughter should be kept quiet late in the day when she is most likely to have a seizure. b) if their daughter shows symptoms of beginning a seizure, immediately give her medication. c) their daughter should maintain an active lifestyle. d) their daughter should carry a padded tongue blade with her at all times.

C It is important for children with seizures to maintain as near normal a lifestyle as possible to maintain self-esteem and achievement. Most seizure medications must create a therapeutic level before they are effective.

The nurse is preparing a toddler for a lumbar puncture. For this procedure, the nurse should place the child in which position? a) Sitting up, with the back straight b) Lying prone, with the feet higher than the head c) Lying on one side, with the back curved d) Lying prone, with the neck flexed

C Lumbar puncture involves placing a needle between the lumbar vertebrae into the subarachnoid space. For this procedure, the nurse should position the client on one side with the back curved because curving the back maximizes the space between the lumbar vertebrae, facilitating needle insertion. Prone and seated positions don't achieve maximum separation of the vertebrae.

Which nursing action should be included in the care plan to promote comfort in a 4-year-old child hospitalized with meningitis? a) Have the child's 2-year-old brother stay in the room b) Keep the lights on brightly so that he can see his mother c) Avoid making noise when in the child's room d) Rock the child frequently

C Meningeal irritation may cause seizures and heightens a child's sensitivity to all stimuli, including noise, lights, movement, and touch. Frequent rocking, presence of a younger sibling, and bright lights would increase stimulation.

The nurse is discussing with a parent the difference between a breath-holding spell and a seizure. The nurse would be correct in telling the parent which of the following with regards to seizures? a) The patient is bradycardiac. b) Cyanosis occurs at the onset of the seizure. c) Convulsive activity occurs. d) The EEG is normal.

C During seizures convulsive activity is typically noted. During a breath-holding spell, the child is bradycardiac, cyanosis occurs at the onset, and the EEG is normal.

Which teaching is most appropriate for a client with chemotherapy-induced neuropathy? A. Bathe in cold water. B. Wear cotton gloves when cooking. C. Consume a diet high in fiber. D. Make sure shoes are snug.

C. A high-fiber diet will assist with constipation due to neuropathy. Cotton gloves may prevent harm from scratching; protective gloves should be worn for washing dishes and gardening. Wearing cotton gloves while cooking can increase the risk for burns

With which male client will the nurse conduct prostate screening and education? A. Young adult with a history of urinary tract infections. B. Client who has sustained an injury to the external genitalia. C. Adult who is older than 50 years. D. Sexually active client.

C. A man who is 50 years or older is at higher risk for prostate cancer.

Which diagnostic test(s) is/are performed to assist in the diagnosis of Non-Hodgkin Lymphoma? A) Physical exam and angiography B) Laparotomy C) Blood and urine cultures D) EKG

C) Blood and urine cultures

A nurse is caring for a child who has bronchiolitis. Which of the following are appropriate actions for the nurse to take? (Select all that apply). A. Administer oral prednisone B. Initiate chest percussion and postural drainage. C. Administer humidified oxygen D. Suction the nasopharynx as needed E. Administer oral penicillin

C-humidified oxygen provides moisture to the airway and is an appropriate action for the nurse to take D-Suctioning that nasopharynx will assist the client to clear secretions and is an appropriate action for the nurse to take.

A mother brings her child in the office for a follow-up appointment and voices concern that her child has started urinating more than normal and is constantly thirsty & hungry. As the RN, you suspect?* A. Hypoglyemia B. Phenylkentonuria C. Diabetes Mellitus D. Tret's syndrome

C. The symptoms the mother is reports are the classic 3 P's of diabetes: polyuria, polydipsia, polyphagia

The most frequently used diagnostic test for persons with GERD is: A. A barium enema. B. An upper endoscopy. C. A barium swallow D/ Acid perfusion tests

C. A barium swallow.

Which of the following findings would alarm the nurse when caring for a client receiving chemotherapy who has a platelet count of 17,000/mm3? A. Increasing shortness of breath B. Diminished bilateral breath sounds C. Change in mental status D. Weight gain of 4 pounds in 1 day

C. A change in mental status could result from spontaneous bleeding; in this case, a cerebral hemorrhage may have developed. Increasing shortness of breath is typically related to anemia, not to thrombocytopenia.

A newly graduated RN has just finished a 6-week orientation to the oncology unit. Which of these clients would be most appropriate to assign to the new graduate? A. A 30-year-old with acute lymphocytic leukemia who will receive combination chemotherapy today B. A 40-year-old with chemotherapy-induced nausea and vomiting who has had no urine output for 16 hours C. A 45-year-old with pancytopenia who will require IV administration of erythropoietin (Procrit) D. A 72-year-old with tumor lysis syndrome who is receiving normal saline IV at a rate of 250 mL/hr

C. A new nurse after a 6-week oncology orientation possesses the skills to care for clients with pancytopenia and with administration of medications to stimulate the bone marrow, the other options are too complex

When a child with nephrotic syndrome is confined to bed, what is an appropriate nursing intervention? A. Restrain the child as necessary. B. Discourage parents from holding the child. C. Adjust activities to child's tolerance level. D. Perform passive range-of-motion exercises daily.

C. Adjust activities to child's tolerance level.

The nurse is instructing a client with breast cancer who will be undergoing chemotherapy about side effects of doxorubicin (Adriamycin). Which side effect will the nurse instruct the client to report to the physician? A. Diaphoresis B. Dysphagia C. Edema D. Hearing loss

C. Doxorubicin (Adriamycin) is an anthracycline, and clients must be instructed to be aware of and to report cardiotoxic effects, including edema, shortness of breath, chronic cough, and excessive fatigue.

Gastroesophageal reflux disease (GERD) weakens the lower esophageal sphincter, predisposing older persons to risk for impaired swallowing. In managing the symptoms associated with GERD, the nurse should assign the highest priority to which of the following interventions? A. Decrease daily intake of vegetables and water, and ambulate frequently. B. Drink coffee diluted with milk at each meal, and remain in an upright position for 30 minutes. C. Eat small, frequent meals, and remain in an upright position for at least 30 minutes after eating. D. Avoid over-the-counter drugs that have antacids in them.

C. Eat small, frequent meals, and remain in an upright position for at least 30 minutes after eating.

A client receiving high-dose chemotherapy who has bone marrow suppression has been receiving daily injections of epoetin alfa (Procrit). Which assessment finding indicates to the nurse that today's dose should be held and the health care provider notified? A. Hematocrit of 28% B. Total white blood cell count of 6200 cells/mm3 C. Blood pressure change from 130/90 mm Hg to 148/98 mm Hg D. Temperature change from 99° F (37.2 C) to 100 F (37.8 C)

C. Epoetin alfa and other erythropoiesis-stimulating agents (ESAs) such as darbepoetin alfa (Aranesp) and epoetin alfa (Epogen, Procrit) increase the production of many blood cell types, not just erythrocytes, which increases the client's risk for hypertension, blood clots, strokes, and heart attacks, especially among older adults. Dosing is based on individual client hemoglobin and hematocrit levels to ensure that just enough red blood cells are produced to avoid the need for transfusion but not to bring hemoglobin or hematocrit levels up to normal. The increased blood pressure is an indication to stop this therapy immediately.

A 25-year-old client with Grave's disease is admitted to the unit. What would the nurse expect the admitting assessment to reveal? A. Bradycardia B. Decreased appetite C. Exophthalmos D. Weight gain

C. Exophthalmos

When reviewing the management of sickle cell anemia with a parent, the nurse understands further teaching would be needed if the parent said that which of the following circumstances contributed to a crisis? A. Excessive vomiting B. Fever C. Foods that are low in iron D. Emotional stress

C. Foods that are low in iron

A child is recently diagnosed with JIA. The most important priority for the child and family is promotion of: A. Optimum nutrition B. Anxiety reduction C. Growth and development D. Education

C. Growth and development

A premenopausal client diagnosed with breast cancer will be receiving hormonal therapy. The nurse anticipates that the physician will request which medication for this client? A. Anastrazole (Arimdex) B. Fulvestrant (Faslodex) C. Leuprolide (Lupron) D. C. Trastuzumab (Herceptin)

C. Leuprolide (Lupron) is used in premenopausal women whose main estrogen source is the ovaries and who may benefit from luteinizing hormone-releasing hormone (LH-RH) agonists that inhibit estrogen synthesis.

Which intervention will be most helpful for the client with mucositis? A. Administering a biological response modifier B. Encouraging oral care with commercial mouthwash C. Providing oral care with a disposable mouth swab D. Maintaining NPO until the lesions have resolved

C. Mouth swabs are soft and disposable and therefore clean. Commercial mouthwashes should be avoided because they may contain alcohol or other drying agents that may further irritate the mucosa.

A client who is scheduled to undergo radiation for prostate cancer is admitted to the hospital by the registered nurse. Which statement by the client is most important to communicate to the physician? A. "I am allergic to iodine." B. "My urinary stream is very weak." C. "My legs are numb and weak." D. "I am incontinent when I cough."

C. Numbness and weakness should be reported to the physician because paralysis caused by spinal cord compression can occur.

A nurse is caring for a client with neutropenia who has a suspected infection. Which intervention does the nurse implement first? A. Hydrates the client with 1000 mL of IV normal saline B. Initiates the administration of prescribed antibiotics C. Obtains requested cultures D. Places the client on Bleeding Precautions

C. Obtaining cultures to identify the infectious agent correctly is the priority for this client.

The nurse is caring for a child with osteosarcoma. Which symptom would the nurse expect? A. Difficulty controlling fine motor movements of the hand. B. Swelling around the area of the elbow. C. Pain in the femur, especially when bearing weight. D. A stiff neck.

C. Pain in the femur, especially when bearing weight.

John's mother just noticed that John has lumps around his neck while bathing him that she has never seen before. What would the doctor associate with the manifestations of Hodgkin's disease? A. Painful, non-moveable lymph nodes B. Bone pain C. Painless, firm, moveable lymph nodes D. Generalized edema

C. Painless, firm, moveable lymph nodes

Which nursing consideration is important when caring for a child with in Pedigo contagiosum

Carefully wash hands maintain cleanse leanness when caring for an infected child

A nurse is asked to provide education for a 15-year-old who requires surgical treatment for scoliosis. Which should be an appropriate explanation for the adolescent? "The goal of surgery is to": A. Allow you to be taller B. Prevent pain C. Prevent problems with breathing D. Allow clothes to fit you better

C. Prevent problems with breathing

During teaching, the nurse should advise the family of a child newly-diagnosed with Graves' disease to: A. Encourage outdoor activities B. Limit bathing to prevent skin irritation C. Promote interaction with one friend instead of a group D. Set the thermostat higher than normal for comfort

C. Promote interaction with one friend instead of a group Children with Graves' disease (an autoimmune conditions that causes hyperthyriodism) tend to be more emotionally labile, and may have difficulty managing group dynamics. Sweating and feeling too warm are common complaints; showering should be encouraged. Bright light, such as sunshine, may be irritating because of disease-related ophthalmopathy.

A 52-year-old client relates to the nurse that she has never had a mammogram because she is terrified that she will have cancer. Which response by the nurse is therapeutic? A. "Don't worry, most lumps are discovered by women during breast self-examination." B. "Does anyone in your family have breast cancer?" C. "Finding a cancer in the early stages increases the chance for cure." D. "Have you noticed a lump or thickening in your breast?"

C. Providing truthful information addresses the client's concern.

Which precaution is most important for the nurse to teach a client receiving radiation therapy for head and neck cancer? A. Avoid eating red meat during treatment. B. Pace your leisure activities to prevent fatigue. C. See your dentist twice yearly for the rest of your life. D. Avoid using headphones or headsets until your hair grows back.

C. Radiation therapy that is directed in or around the oral cavity has a variety of actions that increase the risk for dental caries (cavities) and tooth decay. The salivary glands are affected, which changes the composition of the person's saliva and often causes "dry mouth." This result allows rapid bacterial overgrowth, which leads to cavity formation. In addition, the radiation damages the integrity of the enamel and also damages some of the living cells in the tooth. All contribute to an increased risk for dental infections and cavities.

A male client with a cerebellar brain tumor is admitted to an acute care facility. The nurse formulates a nursing diagnosis of Risk for injury. Which "related-to" phrase should the nurse add to complete the nursing diagnosis statement? A. Related to visual field deficits B. Related to difficulty swallowing C. Related to impaired balance D. Related to psychomotor seizures

C. Related to impaired balance

Which intervention will be most helpful in preventing disseminated intravascular coagulation (DIC)? A. Monitoring platelets B. Administering packed red blood cells C. Using strict aseptic technique to prevent infection D. Administering low-dose heparin therapy for clients on bedrest

C. Sepsis is a major cause of DIC, especially in the oncology client. Heparin may be administered to clients with DIC who have developed clotting, but this has not been proven to prevent the disorder.

The best way to communicate to a child with ASD about a preocedure they are going to have would be: A. smile to let them know everything is going to be ok. B. tell them they are going to have a shot C. Show them pictures of the procedure that include few simple words D. Explain the procedure in great detail

C. Show them pictures of the procedure that include few simple words

A nurse is caring for a 2 year old child who has had 3 ear infections in the past 5 months. The nurse should know that the child is at risk for developing which of the following as a long-term complication? A. Balance difficulties B. Prolonged hearing loss C. Speech delays D. Mastoiditis

C. Speech delays

The registered nurse would correct the nursing student when caring for a client with neutropenia secondary to chemotherapy in which circumstance? A. Student scrubs the hub of IV tubing before administering an antibiotic. B. Nurse overhears the student explaining to the client the importance of handwashing. C. Student teaches the client that symptoms of neutropenia include fatigue and weakness. D. The nurse observes the student providing oral hygiene and perineal care.

C. Symptoms of neutropenia include low neutrophil count, fever, and signs and symptoms of infection; the student should be corrected.

A client with a high genetic risk for breast cancer asks the nurse about options for prevention and early detection. Which option for prevention and early detection is the option of choice? A. Breast self-examination (BSE) beginning at 20 years of age B. Hormone replacement therapy combining estrogen and progesterone C. Magnetic resonance imaging (MRI) and mammography every year beginning at age 30 D. Prophylactic mastectomy

C. The American Cancer Society recommends that high-risk women (greater than 20% lifetime risk) have an MRI and mammography every year beginning at age 30.

Six weeks after hematopoietic stem cell transplantation for leukemia, the client's white blood cell (WBC) count is 8200/mm3. What is the nurse's best action in view of this laboratory result? A. Notify the health care provider immediately. B. Assess the client for other symptoms of infection. C. Document the laboratory report as the only action. D. Remind the client to avoid crowds and people who are ill.

C. The WBC count is now within the normal range (5000 to 10,000/mm3) and is an indicator of successful engraftment. The client is not at any particular risk for infection at this time, nor is there reason to believe an infection is present. (At any post-transplantation check-up, the client is assessed for infection.)

A school nurse plans to reinforce information about the most effective methods to prevent the spread of head lice in school age children at a teacher's conference. The most appropriate information to give would be that: A. The classroom should be sprayed with an insecticide at the end of each day B. Hand washing should be done before and after each break by each child C. The children are not to share hats and scarves D. The heads of children are to be checked weekly for lice

C. The children are not to share hats and scarves

The nurse is conducting a staff in-service appearance of childhood skin conditions lymphangitis streaking is frequently seen in which condition

Cellulitis

A client being treated for advanced breast cancer with chemotherapy reports that she must be allergic to one of her drugs because her entire face is swollen. What assessment does the nurse perform? A. Asks whether the client has other known allergies B. Checks the capillary refill on fingernails bilaterally C. Examines the client's neck and chest for edema and engorged veins D. Compares blood pressure measured in the right arm with that in the left arm

C. The client's swollen face indicates possible superior vena cava syndrome, which is an oncologic emergency. Manifestations result from the blockage of venous return from the head, neck, and upper trunk. Early manifestations occur when the client arises after a night's sleep and include edema of the face, especially around the eyes, and tightness of the shirt or blouse collar. As the compression worsens, the client develops engorged blood vessels and erythema of the upper body, edema in the arms and hands, dyspnea, and epistaxis. Interventions at this stage are more likely to be successful. Late manifestations include hemorrhage, cyanosis, mental status changes, decreased cardiac output, and hypotension. Death results if compression is not relieved.

The client receiving chemotherapy will experience the lowest level of bone marrow activity and neutropenia during which period? A. Peak B. Trough C. Nadir D. Adjuvant

C. The lowest point of bone marrow function is referred to as the nadir. The peak of bone marrow function occurs when the client's blood levels are at their highest.Trough, which means low, is typically used in reference to drug levels.

The nurse caring for a child with neuroblastoma includes which of the following assessments as a measure of tumor responsiveness? A. Serum copper levels B. Erythrocyte sedimentation rate (ESR) C. Urine catecholamines D. Urinalysis with a culture and sensitivity

C. Urine catecholamines

Indicators for ASD include all the following except A. lack of verbalization B. lack of social ability C. smiling all the time D. lack of response to verbal interactions

C. smiling all the time

The client who has undergone breast surgery is struggling with issues concerning her sexuality. What is the best way for the nurse to address the client's concerns? A. Allow the client to bring up the topic first. B. Remind the client to avoid sexual intercourse for 2 months after the surgery. C. Suggest that the client wear a bra during intercourse. D. Teach the client that birth control is a priority.

C.Suggest that the client wear a bra during intercourse. Clients may prefer to lay a pillow over the surgical site or wear a bra or camisole to prevent contact with the surgical site during intercourse.

A child being treated for Acute Lymphocytic Leukemia (ALL) has a white blood cell (WBC) count of 7,000/mm3. the nursing care plan lists risk for infection as a priority nursing diagnosis, and measures are being taken to reduce the child's exposure to infection. the nurse determines that the plan has been successful when which outcome has been met? "1. child's WBC count goes up. 2. child's WBC count goes down. 3. child's temperature remains within normal range. 4. parents demonstrate good hand washing technique."

CORRECT is #3 - RATIONALE: in leukemia, the WBCs that are present are immature and incapable of fighting infection. increases or decreases in the number of WBCs can be related to the disease process and treatment, and not related to infection. the only value that indicates the child is infection-free is the temperature. the use of proper handwashing technique is a measure or intervention used to meet a goal. but is not a goal itself. STRATEGY: the core issue of the question is knowledge of an indicator of infection in a client who is immunosuppressed from leukemia. recall that temperature and WBC counts are frequently used as indicators of infection. recall that in leukemia the WBCs are abnormal so choose the option related to temperature.

A pregnant woman has a child at home who has been diagnosed with neurofibromatosis She asks the nurse what she should look for in the new baby that would indicate that it also has neurofibromatosis. What sign should the nurse instruct the woman to look for in the new baby? a) Projectile vomiting b) Café-au-lait spots c) Xanthoma d) Increased urination

Café-au-lait spots Correct Explanation: Physical assessment may provide clues that a particular genetic condition is present in a person and family. Family history assessment may offer initial guidance regarding the particular area for physical assessment. For example, a family history of neurofibromatosis type 1, an inherited condition involving tumors of the central nervous system, would prompt the nurse to carry out a detailed assessment of closely related family members. Skin findings such as café-au-lait spots, axillary freckling, or tumors of the skin (neurofibromas) would warrant referral for further evaluation, including genetic evaluation and counseling. A family history of familial hypercholesterolemia would alert the nurse to assess family members for symptoms of hyperlipidemias (xanthomas, corneal arcus, abdominal pain of unexplained origin). As another example, increased urination could indicate type 1 diabetes. Projectile vomiting is indicative of pyloric stensosis.

The nurse is examining 12 month old Amy who is brought to the clinic for diaper rash the nurse finds information with satellite lesions that cost the inguinal fault this is most likely caused by

Candida albicans

Which type of genetic test would be used to detect the possibility of Down syndrome? a) Complete blood count (CBC) b) DNA analysis c) Hemoglobin electrophoresis d) Chromosomal analysis

Chromosomal analysis Correct Explanation: Chromosomal analysis is part of the genetic testing for Down syndrome. DNA analysis may be used in the detection of Huntington disease. Hemoglobin electrophoresis may be used in genetic testing for sickle cell anemia. A complete blood count (CBC) may be used as part of testing for a thalassemia

You care for a child with Down syndrome (trisomy 21). This is an example of which type of inheritance? a) Mendelian dominant b) Phase 2 atrophy c) Chromosome nondisjunction d) Mendelian recessive

Chromosome nondisjunction Correct Explanation: Down syndrome occurs when an ovum or sperm cell does not divide evenly, permitting an extra 21st chromosome to cross to a new cell.

A nursing student is reviewing information about inheritance and genetic disorders. The student demonstrates understanding of the information by identifying which of the following as an example of a disorder involving multifactorial inheritance? a) Cystic fibrosis b) Hypophosphatemic rickets c) Hemophilia d) Cleft palate

Cleft palate Correct Explanation: Cleft palate is attributed to multifactorial inheritance. Hemophilia follows an X-linked recessive inheritance pattern. Hypophosphatemic rickets follows an X-linked dominant inheritance pattern. Cystic fibrosis follows an autosomal recessive inheritance pattern.

For which of the following clients is preimplantation genetic diagnosis (PGD) a viable option? a) Clients carrying cystic fibrosis gene b) Prevention of Pyloric stenosis c) Prevention of DiGeorge syndrome d) Client in the second week of pregnancy

Clients carrying cystic fibrosis gene Correct Explanation: Preimplantation genetic diagnosis (PGD) is a viable option for clients carrying the cystic fibrosis gene. PGD does not help prevent DiGeorge syndrome or pyloric stenosis. PGD is not a viable option for pregnant clients.

Upon assessment, the nurse notices that the infant's ears are low-set. What is the priority action by the nurse? a) Inform the parents that low-set ears are a sign of Down syndrome b) Place the infant on a cardiac monitor c) Continue to assess the infant to look for other abnormalities d) Give a vitamin B12 injection to combat the metabolic disorder

Continue to assess the infant to look for other abnormalities Correct Explanation: Continue to assess for major and minor congenital anomalies because major anomalies may require immediate medical attention. Three or more minor anomalies increase the chance of a major anomaly. Low-set ears can be a symptom of a variety of genetic disorders. Mentioning Down syndrome without further investigation can cause undue stress in parents. The infant may not need cardiac monitoring; further assessment will provide clues. Diagnostic testing is needed to determine whether the child is afflicted with a metabolic disorder.

The management of a child who has just been stung by a bee or wasp should include the application of

Cool compress

A 12 year old boy seen in the clinic, and a diagnosis of Hodgkin's disease is suspected . Which diagnostic test results confirm the diagnosis of Hodgkin's disease? 1 . Elevated vanillylmandelic acid urinary level. 2. The presence of blast cells in the bone marrow 3. The presence of Epsetin-Barr virus in the blood. 4. The presence of Reed-Sternberg cells in the lymph nodes

Correct Answer 4 . Hodgkin's disease is a neoplasm of lymphatic tissue. The presence of gaint multinucleated cells ( Reed- Sternbergs cells) is the hallmark of this disease. The presence of blast cells in the bone marrow indicates leukemia. The Epstein-Barr virus is associated with infectious mononucleosis . Elevated levels of vanillylmandelic acid in the urine may be found in children with neroblastoma.

A child is diagnosed with Wilms' tumor. In planning teaching interventions, what key point should the nurse emphasize to the parents? "1. Do not put pressure on the abdomen. 2. Frequent visits from friends and family will improve morale. 3. Appropriate protective equipment should be worn for contact sports. 4. Encourage the child to remain active."

Correct answer: 1. Do not put pressure on the abdomen. Palpation of Wilms' tumor can cause rupture and spread of cancerous cells. Frequent visitation might allow the child to be exposed to more infections, and activity and sports are discouraged because of the risk of rupture of the encapsulated tumor.

"What is a characteristic manifestation of Hodgkin's Disease? "1.) petechiae 2.) erythematous rash 3.) enlarged lymph nodes 4.) pallor"

Correct: #3 "Knowledge of the usual pattern of spread of this lymphoma, with its orderly progression through lymph node groups and its typical forms of extranodal involvement, facilitates timely diagnosis, staging, and treatment planning".

"A 15-year-old has been admitted to the hospital with the diagnosis of acute lymphocytic leukemia. Which of the following signs and symptoms require the most immediate nursing intervention? (Choices were deleted)

Correct: 2. Fever and petechiae associated with acute lymphocytic leukemia indicate a suppression of normal white blood cells and thrombocytes by the bone marrow and put the client at risk for other infections and bleeding. The nurse should initiate infection control and safety precautions to reduce these risks. Fatigue is a common symptom of leukemia due to red blood cell suppression. Although the client should be told about the need for rest and meal planning, such teaching is not the priority intervention. Swollen glands and lethargy may be uncomfortable but they do not require immediate intervention. An enlarged liver and spleen do require safety precautions that prevent injury to the abdomen; however, these precautions are not the priority.

What assessment finding would the nurse expect to find specifically for a client admitted with Hodgkin's disease? "1. Fatigue 2. weakness. 3. Weight gain 4. Enlarged lymph nodes."

Correct: 4. Enlarged lymph nodes with progression to extralymphatic sites. This is a characteristic specifically to lymphoma, where as fatigue and weakness can occur with other diseases. Weight loss is more likely than weight gain.

Any individual taking phenobarbital for a seizure disorder should be taught a) never to go swimming. b) to avoid foods containing caffeine. c) to brush his or her teeth four times a day. d) never to discontinue the drug abruptly.

D Phenobarbital should always be tapered, not stopped abruptly, or seizures from the child's dependency on the drug can result.

The nurse is collecting data from a child who may have a seizure disorder. Which of the following is a description of an absence seizure? a) Sudden, momentary loss of muscle tone, with a brief loss of consciousness b) Brief, sudden contracture of a muscle or muscle group c) Muscle tone maintained and child frozen in position d) Minimal or no alteration in muscle tone, with a brief loss of consciousness

D A child with an elevated temperature is at high risk for having seizures and therefore actions by the nurse include keeping the child in a safe situation to prevent any injury if the child should have a seizure.

Which of the following is most correct regarding the nervous system of the child? a) The child has underdeveloped fine motor skills and well-developed gross motor skills. b) The child's nervous system is fully developed at birth. c) The child has underdeveloped gross motor skills and well-developed fine motor skills. d) As the child grows, the gross and fine motor skills increase.

D As the child grows, the quality of the nerve impulses sent through the nervous system develops and matures. As these nerve impulses become more mature, the child's gross and fine motor skills increase in complexity. The child becomes more coordinated and able to develop motor skills

The treatment for children with seizures disorders is most often which of the following? a) Strict exercise regimen b) Restricted fat diet c) Surgical intervention d) Use of anticonvulsant medications

D Complete control of seizures can be achieved for most people through the use of anticonvulsant drug therapy. A few children may be candidates for surgical intervention but, in most cases, surgery is not the treatment. Ketogenic diets (high in fat and low in carbohydrates and protein) cause the child to have high levels of ketones, which help to reduce seizure activity. Exercise is not a treatment for seizure disorders.

The nurse has developed a nursing plan for the care of a 6-year-old girl with congenital hydrocephalus whose shunt has become infected. The most important discharge teaching point for this family is: a) maintaining effective cerebral perfusion. b) encouraging development of motor skills. c) establishing seizure precautions for the child. d) ensuring the parents know how to properly give antibiotics.

D ensuring the parents know how to properly give antibiotics. Explanation: Educating parents how to properly give the antibiotics would be the priority intervention because the child's shunt has become infected. Maintaining cerebral perfusion is important for a child with hydrocephalus, but the priority intervention for the parents at this time is in regards to the infection. Establishing seizure precautions is an intervention for a child with a seizure disorder. Encouraging development of motor skills would be appropriate for a microcephalic child.

A patient with a history is diabetes is exhibiting sweating and slurred speech. What do you suspect is the cause? A. hyponaterima B. hypernaterima C.hyperglycemia D. hypoglycemia

D These are the classic symptoms of hypoglycemia.

A preschool-age child has just been admitted to the pediatric unit with a diagnosis of bacterial meningitis. The nurse would include which recommendation in the nursing plan? a) Take vital signs every 4 hours b) Monitor temperature every 4 hours c) Encourage the parents to hold the child d) Decrease environmental stimulation

D A child with the diagnosis of meningitis is much more comfortable with decreased environmental stimuli. Noise and bright lights stimulate the child and can be irritating, causing the child to cry, in turn increasing intracranial pressure. Vital signs would be taken initially every hour and temperature monitored every 2 hours. Children with bacterial meningitis are usually much more comfortable if allowed to lie flat because this position doesn't cause increased meningeal irritation.

The nurse is collecting data from the caregivers of a child admitted with seizures. Which of the following statements indicates the child most likely had an absence seizure? a) "He kept smacking his lips and rubbing his hands." b) "He usually is very coordinated, but he couldn't even walk without falling." c) "His arms had jerking movements in his legs and face." d) "He was just staring into space and was totally unaware."

D Absence seizures rarely last longer than 20 seconds. The child loses awareness and stares straight ahead but does not fall. Absence seizures rarely last longer than 20 seconds. The child loses awareness and stares straight ahead but does not fall. Myoclonic seizures are characterized by a sudden jerking of a muscle or group of muscles, often in the arms or legs, without loss of consciousness. Complex partial seizures cause nonpurposeful movements, such as hand rubbing and lip smacking. During the prodromal period of the tonic-clonic seizure, the child might have a lack of coordination.

Which is one of the first signs of overwhelming sepsis and a child with burn injuries

Disoriented

A mother had just given birth to her first child. The mother shows concern to the nurse about Sudden Infant Death Syndrome (SIDS). The mother asks the nurse how to position her newborn baby when sleeping. The nurse explains to the mother that the newborn should be placed on: A) Back or prone B) Side or prone C) Stomach with the face turned D) Back rather than on the stomach

D) Back rather than on the stomach

The nurse is teaching a client who is receiving an anti-estrogen drug about the side effects she may encounter. Which of these should the nurse include in the discussion? Select all that apply. A. Heavy menses B. Smooth facial skin C. Hyperkalemia D. Breast tenderness E. Weight loss F. Deep vein thrombosis (DVT)

D,F: Breast tenderness and shrinking breast tissue, Venous thromboembolism, Irregular menses or no menstrual period, Acne may develop, Hypercalcemia, not hyperkalemia, is typical and Fluid retention with weight gain may occur.

A child is ordered by the doctor for ketone and glucose urine testing. The patient is to collect it at home. How would you instruct the patient to collect the specimen?* A. Cleanse the area with betadine. B. Encourage the patient to consume at least 24 oz of water prior to the specimen collection. C. Demonstrate a clean catch techinque. D. Use the second voided urine for most accurate results.

D. The patient should use the second voided urine to ensure that the results are accurate. First voided urines tend to be concentrated and could effect results.

A 15 year old, who is type 1 diabetic, reports that she almost "passes out" during gym class. What information would you assess from the teenager?* A. None of the options are correct. B. What type of form she needs to have filled out so she can be excused from gym class. C. How she takes her blood glucose after exercise. D. Her eating habits prior to gym class.

D. It is very important to ask the teen when and what she eats before gym. Type 1 diabetics are encouraged to eat before physical activity to decrease the chances of hypoglycemia (which is what this teen is experiencing). She should take her blood glucose BEFORE exercise not AFTER. There is no need for her to be excused from gym class because exercise is essential for diabetics.

A client is admitted to the unit to determine if his symptoms are being caused by a brain tumor. He is scheduled to have an MRI. Which question by the nurse is most important in preparing the client for the MRI? A. "Have you had anything to eat or drink today?" B. "Are you afraid of the dark?" C. "When was the last time you had a bowel movement?" D. "Do you have a pacemaker?"

D. "Do you have a pacemaker?"

The issue that is often foremost in the minds of men who have been diagnosed with prostate cancer and must be addressed by the nurse is the alteration of which factor? A. Comfort because of surgical pain. B. Mobility because of treatment. C. Nutrition because of radiation treatment. D. Sexual function after treatment.

D. : Altered sexual function is one of the biggest concerns of men after cancer treatment.

Which client being cared for on the medical-surgical unit will be best to assign to a nurse who has floated from the intensive care unit (ICU)? A. Recent radical mastectomy client who requires chemotherapy administration B. Modified radical mastectomy client who needs discharge teaching C. Stage III breast cancer client who is requesting information about radiation and chemotherapy D. A client with a Jackson-Pratt drain in place who has just arrived from the postanesthesia care unit (PACU) after a quadrantectomy

D. A nurse working in the ICU would be familiar with postoperative monitoring and care of clients with Jackson-Pratt drains.

Which of the following items would be most beneficial when providing oral care to a patient with metastatic cancer who is at risk for oral tissue injury secondary to chemotherapy? A. Firm-bristle toothbrush B. Hydrogen peroxide rinse C. Alcohol-based mouthwash D. 1 tsp salt in 1 L water mouth rinse

D. A salt-water mouth rinse will not cause further irritation to oral tissue that is fragile because of mucositis, which is a side effect of chemotherapy.

A nurse is caring for a toddler who has acute otitis media. Which of the following is a priority action for the nurse to take? A. Provide emotional support to the family B. Educate the family on the care of the child C. Prevent clinical complications D. Administer analgesics

D. Administer analgesics

A 72-year-old client recovering from lung cancer surgery asks the nurse to explain how she developed cancer when she has never smoked. Which factor may explain the possible cause? A. A diagnosis of diabetes treated with insulin and diet B. An exercise regimen of jogging 3 miles 4x/wk C. A history of cardiac disease D. Advancing age

D. Advancing age is the single most important risk factor for cancer. As a person ages, immune protection decreases.

A client who is undergoing chemotherapy for breast cancer reports problems with concentration and memory. Which intervention is indicated at this time? A. Explain that this occurs in some clients and is usually permanent. B. Encourage the client that a small glass of wine may help her relax. C. Protect the client from infection. D. Allow the client an opportunity to express her feelings.

D. Although no specific intervention for the side effect is known, therapeutic communication and listening may be helpful to the client.

The client with gastroesophageal reflux disease complains of a chronic cough. The nurse understands that in a client with GERD this symptom may be indicative of which of the following conditions? A. Development of laryngeal cancer. B. Irritation of the esophagus. C. Esophageal scar tissue formation. D. Aspiration of gastric contents

D. Aspiration of gastric contents

The nurse is assigned to care for a client immediately after breast-conserving surgery for cancer. What is the priority for care of the client at this time? A. Teach the client to sleep in the prone position each night. B. Empty wound drains and record the output amount. C. Remind the client how to perform breast self-examination. D. Monitor the incision and flap for adequate tissue perfusion.

D. Assess the incision and flap for duskiness and decreased capillary refill during dressing changes, which are signs of poor tissue perfusion. The client should avoid sleeping in the prone position. Emptying drains, documenting output, and encouraging BSE are important but are not the priority in the immediate postoperative phase.

The large-breasted client reports discomfort, backaches, and fungal infections because of her excessive breast size. The nurse plans to provide information to the client about which breast treatment option? A. Augmentation B. Compression C. Reconstruction D. Reduction mammoplasty

D. Breast reduction mammoplasty surgery removes excess breast tissue and repositions the nipple and remaining skin flaps to produce the best cosmetic effect.

The home health RN is caring for a client who has a history of a kidney transplant and takes cyclosporine (Sandimmune) and prednisone (Deltasone) to prevent rejection. Which assessment data would be most important to communicate to the transplant team? A. The temperature is 96.6° F. B. The client reports joint pain. C. The oral mucosa appears pink and dry. D. A lump is palpable in the client's axilla.

D. Clients taking immune suppressive drugs to prevent rejection are at increased risk for development of cancer; any lump should be reported to the physician.

The nurse teaches the client that intraperitoneal chemotherapy will be delivered where? A. Into the veins of the legs B. Into the lung C. Into the heart D. Into the abdominal cavity

D. Intraperitoneal chemotherapy is placed in the peritoneal cavity or the abdominal cavity.

The nurse is caring for an adult client with Down syndrome who reports fatigue and shortness of breath. Which type of cancer has been identified in clients with Down syndrome? A. Breast cancer B. Colorectal cancer C. Malignant melanoma D. Leukemia

D. Leukemia is associated with Down syndrome and Turner syndrome.

The outpatient client is receiving photodynamic therapy. Which environmental factor is a priority for the client to adjust for protection? A. Storing drugs in dark locations at room temperature B. Wearing soft clothing C. Wearing a hat and sunglasses when going outside D. Reducing all direct and indirect sources of light

D. Lighting of all types must be kept to a minimum. It can lead to burns of the skin and damage to the eyes because they are sensitive to light. Any drug that the client is prescribed should be considered for its photosensitivity properties. Drugs should be stored according to the recommendations, but this is not the primary concern for this client. The client will be homebound for 1 to 3 months after the treatment and should not go outside.

Which assessment finding indicates to the nurse that the client is at high risk for a malignant breast lesion? A. 1-cm freely mobile rubbery mass discovered by the client B. Ill-defined painful rubbery lump in the outer breast quadrant C. Backache and breast fungal infection D. Nipple discharge and dimpling

D. Nipple discharge and dimpling are high-risk assessment findings for a malignant breast lesion.

The nurse is giving a group presentation on cancer prevention and recognition. Which statement by an older adult client indicates understanding of the nurse's instructions? A. "Cigarette smoking always causes lung cancer." B. "Taking multivitamins will prevent me from developing cancer." C. "If I have only one shot of whiskey a day, I probably will not develop cancer." D. "I need to report the pain going down my legs to my health care provider."

D. Pain in the back of the legs could indicate prostate cancer in an older man.

When assessing the development of a 15 month old child with cerebral palsy, which of the following milestones would the nurse expect a toddler of this age to have achieved? A. Walking up steps B. Using a spoon C. Coping a circle D. Putting a block in a cup

D. Putting a block in a cup

A client who has just been discharged from the hospital after a modified radical mastectomy is referred to a home health agency. Which nursing action will be most appropriate to delegate to an experienced home health aide? A. Assessing the safety of the home environment B. Developing a plan to decrease lymphedema risk C. Monitoring pain level and analgesic effectiveness D. Reinforcing the guidelines for hand and arm care

D. Reinforcing the guidelines for hand and arm care Reinforcement of previously taught information about hand and arm care should be done by all caregivers.

Which of the following nursing diagnoses is most appropriate for a patient experiencing myelosuppression secondary to chemotherapy for cancer treatment? A. Acute pain B. Hypothermia C. Powerlessness D. Risk for infection

D. Risk for infection Myelosuppression is accompanied by a high risk of infection and sepsis. Hypothermia, powerlessness, and acute pain are also possible nursing diagnoses for patients undergoing chemotherapy, but the threat of infection is paramount.

The nurse is caring for a child whose parents are both African Americans. The child exhibits swelling of their hands and feet, fever, and signs of blocked capillaries. What is the likely cause? A. Hodgkin's disease B. Diabetic ketoacidosis C. Hyperthyroidism D. Sickle cell anemia

D. Sickle cell anemia

Which of the following terms best describes appendicitis? A. Aching B. Fleeting C. Intermittent D. Steady

D. Steady

The mother of a child with JIA asks the nurse what activities the child can enjoy. Based on knowledge of the physiologic aspects of JIA, which of the following would be the most appropriate? A. Running B. Boxing C. Skiing D. Swimming

D. Swimming

A 2 year old child is diagnosed with bronchiolitis caused by respiratory syncytial virus (RSV). The child's family also includes an 8 year old child. Which statement is correct? A. RSV isn't highly communicable in infants. B. RSV isn't communicable to older children and adults. C. The 2 year old client must be admitted to the hospital for isolation. D. The children should be separated to prevent the spread of infection.

D. The children should be separated to prevent the spread of infection.

A new mother has some questions about (PKU). Which of the following statements made by a nurse is not correct regarding PKU? A. A Guthrie test can check the necessary lab values. B. The urine has a high concentration of phenylpyruvic acid C. Mental deficits are often present with PKU. D. The effects of PKU are reversible.

D. The effects of PKU are reversible.

If the osteosarcoma tumor metastases, which organ is primarily involved? A. The liver B. The brain C. The kidneys D. The lungs

D. The lungs

The nurse is caring for a patient suffering from anorexia secondary to chemotherapy. Which of the following strategies would be most appropriate for the nurse to use to increase the patient's nutritional intake? A. Increase intake of liquids at mealtime to stimulate the appetite. B. Serve three large meals per day plus snacks between each meal. C. Avoid the use of liquid protein supplements to encourage eating at mealtime. D. Add items such as skim milk powder, cheese, honey, or peanut butter to selected foods.

D. The nurse can increase the nutritional density of foods by adding items high in protein and/or calories (such as peanut butter, skim milk powder, cheese, honey, or brown sugar) to foods the patient will eat.

A diagnosis of Hodgkin's disease is suspected in a 13 year old child seen in a clinic. Several diagnostic studies are performed to determine the presence of this disease. Which diagnostic test results confirm the diagnosis of Hodgkin's disease? A. Elevated vanillylmandelic acid urinary levels. B. The presence of blast cells in the bone marrow. C. The presence of Epstein-Barr virus in the blood. D. The presence of Reed-Sternberg cells in the lymph nodes"

D. The presence of Reed-Sternberg cells in the lymph nodes"

A client asks the nurse about early detection of breast masses. Which statement by the nurse about early detection of breast masses is correct? A. "A yearly breast examination by a health care provider can substitute for breast self-examination (BSE)." B. "Detection of breast cancer before axillary node invasion yields the same survival rate." C. "Mammography as a baseline screening is recommended by the American Cancer Society at 30 years of age." D. "The goal of screening for breast cancer is early detection because BSE does not prevent breast cancer."

D. The purpose of screening is early detection. BSE does not prevent breast cancer.

The nurse presents a cancer prevention program to teens. Which of the following will have the greatest impact in cancer prevention? A. Avoid asbestos. B. Wear sunscreen. C. Get the human papilloma virus (HPV) vaccine. D. Do not smoke cigarettes.

D. Tobacco is the single most important source of preventable carcinogenesis.

Which best describes a Full thickness burn

Destruction of all layers of the skin evident with extension into subcutaneous tissue

A client diagnosed with widespread lung cancer asks the nurse why he must be careful to avoid crowds and people who are ill. What is the nurse's best response? A. "With lung cancer, you are more likely to develop pneumonia and could pass this on to other people who are already ill." B. "When lung cancer is in the bones, it becomes a bone marrow malignancy, which stops producing immune system cells." C. "The large amount of mucus produced by the cancer cells is a good breeding ground for bacteria and other microorganisms." D. "When lung cancer is in the bones, it can prevent production of immune system cells, making you less resistant to infection."

D. Tumor cells that enter the bone marrow reduce the production of healthy white blood cells (WBCs), which are needed for normal immune function. Therefore clients who have cancer, especially leukemia, are at an increased risk for infection. Other people are not at risk for becoming infected as a result of contact with a person who has lung cancer. Lung cancer that has spread to the bone is still lung cancer; it is not a bone marrow malignancy. It is true that the person with lung cancer may produce more mucus, which can harbor microorganisms, but this is not the main reason why the client should avoid crowds and people who are ill.

The nurse is discussing treatment options with the client newly diagnosed with breast cancer. Which statement by the client indicates a need for further teaching? A. "Hormonal therapy is only used to prevent the growth of cancer. It won't get rid of it." B. "I might have chemotherapy before surgery." C. "If I get radiation, I am not radioactive to others." D. "Radiation will remove the cancer, so I might not need surgery."

D. Typically, radiation therapy follows surgery to kill residual tumor cells. Radiation therapy plays a critical role in the therapeutic regimen and is effective treatment for almost all sites where breast cancer can metastasize. The purpose of radiation therapy is to reduce the risk for local recurrence of breast cancer.

An adolescent with Addison's disease may need an increased dosage of glucocorticoids to which of the following situations? A. completing spring semester of school B. Gaining 7 pounds C. Death of a family member D. Undergoing a root canal

D. Undergoing a root canal Physical stress, such as infection, surgery, dental work and pregnancy, can lead to adrenal crisis in those with Addison's disease. Psychological stress has less effect than physical stress. Adrenal insufficiency leads to weight loss.

A nurse is caring for a client with neutropenia. Which clinical manifestation indicates that an infection is present or should be ruled out? A. Coughing and deep breathing B. Evidence of pus C. Fever of 102 deg. F or higher D. Wheezes or crackles

D. Wheezes or crackles in the neutropenic client may be the first symptom of infection in the lungs.

A client had a transurethral resection of the prostate (TURP) yesterday. The staff nurse notes that the hemoglobin is 8.2 g/dL. What is the nurse's best action? A. Notify the charge nurse as soon as possible. B. Irrigate the catheter with 30 mL normal saline. C. Document the assessment in the medical record. D. Prepare for a blood transfusion.

D.Prepare for a blood transfusion Blood transfusions are commonly given after a TURP surgery; a blood transfusion is warranted for a hemoglobin reading of 8.2 g/dL. The nurse is capable of managing this situation with the physician, especially since blood transfusions after a TURP are common. Irrigating the catheter is necessary only if the color of the drainage indicates bleeding or there is a presence of clots. Documentation should be done, but it is not the first priority.

The nurse understands that hormone treatment for prostate cancer works by which action? A. Decreases blood flow to the tumor. B. Destroys the tumor. C. Shrinks the tumor. D. Suppresses growth of the tumor.

D.Suppresses growth of the tumor. Hormone therapy, particularly anti-androgen drugs, inhibits tumor progression by blocking the uptake of testicular and adrenal androgens at the prostate tumor site. Anti-androgens may be used alone or in combination with luteinizing hormone-releasing hormone agonists for a total androgen blockade (hormone ablation).

Hydro therapy is required to treat a child with extensive partial thickness burns which is the primary purpose of Hydro therapy

Debride the wounds

Nondisjunction of a chromosome results in which of the following diagnoses? a) Duchenne muscular dystrophy b) Down syndrome c) Marfan syndrome d) Huntingon disease

Down syndrome Correct Explanation: When a pair of chromosomes fails to separate completely (nondisjunction) the resulting sperm or oocyte contains two copies of a particular chromosome. Nondisjunction can result in a fertilized egg having trisomy 21 or Down syndrome. Huntington disease is one example of a germ-line mutation. Duchenne muscular dystrophy, an inherited form of muscular dystrophy, is an example of a genetic disease caused by structural gene mutations. Marfan syndrome is a genetic condition that may occur in a single family member as a result of spontaneous mutation.

A pregnant woman undergoes maternal serum alpha-fetoprotein (MSAFP) testing at 16 to 18 weeks' gestation. Which of the following would the nurse suspect if the woman's level is decreased? a) Cardiac defects b) Open neural tube defect c) Down syndrome d) Sickle-cell anemia

Down syndrome Explanation: Decreased levels might indicate Down syndrome or trisomy 18. Sickle cell anemia may be identified by chorionic villus sampling. MSAFP levels would be increased with cardiac defects, such as tetralogy of Fallot. A triple marker test would be used to determine an open neural tube defect.

A 45-year-old man has just been diagnosed with Huntington disease. He and his wife are concerned about their four children. What will the nurse explain about the children's possibility of inheriting the gene for the disease? a) Each child will have no chance of inheriting the disease b) Each child will have a 50% chance of inheriting the disease c) Each child will have a 25% chance of inheriting the disease d) Each child will have a 75% chance of inheriting the disease

Each child will have a 50% chance of inheriting the disease Correct Explanation: Huntington disease is an autosomal dominant disorder. Autosomal dominant inherited conditions affect female and male family members equally and follow a vertical pattern of inheritance in families. A person who has an autosomal dominant inherited condition carries a gene mutation for that condition on one chromosome pair. Each of that person's offspring has a 50% chance of inheriting the gene mutation for the condition and a 50% chance of inheriting the normal version of the gene. Based on this information, the choices of 25%, 75%, or no chance of inheriting the disease are incorrect.

The nurse is caring for a 1-year-old boy with Down syndrome. Which of the following would the nurse be least likely to include in the child's plan of care? a) Promoting annual vision and hearing tests b) Describing the importance of a high-fiber diet c) Explaining developmental milestones to parents d) Educating parents about how to deal with seizures

Educating parents about how to deal with seizures Correct Explanation: It is unlikely that the parents will need to know how to deal with seizures. It will be helpful to provide parents with growth and developmental milestones that are unique to children with Down syndrome. More than 60% of children with Down syndrome have hearing loss, so promoting annual vision and hearing tests is the priority intervention. Special diets are usually not necessary; however, a balanced, high-fiber diet and exercise are important because constipation is frequently a problem.

When counseling potential parents about genetic disorders, which of the following statements would be appropriate? a) Genetic disorders primarily follow Mendelian laws of inheritance. b) Environmental influences may affect multifactorial inheritance. c) All genetic disorders involve a similar number of abnormal chromosomes. d) The absence of genetic disorders in both families eliminates the possibility of having a child with a genetic disorder.

Environmental influences may affect multifactorial inheritance. Correct Explanation: It is difficult to predict with certainty the incidence of genetic disorders because in some disorders, more than one gene is involved and environmental insults may play a role (cleft palate, for example).

Tretinon Is a typical agent commonly used to treat acne nursing considerations with this drug should include

Explaining that medication should not be applied until at least 20 to 30 minutes after washing

The incidence of Down syndrome is 1:1600 in women older than 40 years of age, compared with 1:100 in women younger than 20 years. a) True b) False

False Correct Explanation: The incidence of Down syndrome is 1:100 in women older than 40 years of age, compared with 1:1600 in women younger than 20 years.

Which is an effective strategy to reduce the stress of bird and dressing procedures

Give child as many choices as possible

The nurses taking care of a two-year-old child with a macule skin lesion which clinical findings should the nurse expect to assess with this type of lesion

Flat brown more less than 1 cm in diameter

Ringworm frequently found in school children is caused by

Fungus

After teaching a class of students about genetics and inheritance, the instructor determines that the teaching was successful when the students identify which of the following as the basic unit of heredity? a) Allele b) Autosome c) Chromosome d) Gene

Gene Correct Explanation: A gene is the basic unit of heredity of all traits. A chromosome is a long, continuous strand of DNA that carries genetic information. An allele refers to one of two or more alternative versions of a gene at a given position on a chromosome that imparts the same characteristic of that gene. An autosome is a non-sex chromosome.

A client who is 37 years of age presents to the health care clinic for her first prenatal check up. Due to her advanced age, the nurse should prepare to talk with the client about her increased risk for what complication? a) Incompetent cervix b) Genetic disorders c) Gestational diabetes d) Preterm labor

Genetic disorders Correct Explanation: Women over the age of 35 are at increased risk of having a fetus with an abnormal karyotype or other genetic disorders. Gestational diabetes, an incompetent cervix, and preterm labor are risks for any pregnant woman.

A toddler sustains a minor burn on the hand from hot coffee which is the first action the nurse should recommend in treating the burn

Hold burn the area under cold running water

When describing genetic disorders to a group of childbearing couples, the nurse would identify which as an example of an autosomal dominant inheritance disorder? a) Phenylketonuria b) Cystic fibrosis c) Huntington's disease d) Sickle cell disease

Huntington's disease Correct Explanation: Huntington's disease is an example of an autosomal dominant inheritance disorder. Sickle cell disease, phenylketonuria, and cystic fibrosis are examples of autosomal recessive inheritance disorders.

An 18-year-old male patient is diagnosed with Klinefelter syndrome. What signs and symptoms are consistent with this diagnosis? a) Hypergonadism and decreased pubic hair b) Hypogonadism and gynecomastia c) Long torso and decreased facial hair d) Enlaged testes and tall stature

Hypogonadism and gynecomastia Correct Explanation: Klinefelter syndrome affects males, causing only testosterone deficiency. Males may develop female-like characteristics such as gynecomastia and may experience hypogonadism. Decreased pubic and facial hair, along with tall stature, are characteristic of the disorder. The corresponding signs and symptoms listed in the other answer selections are not signs and symptoms of the disorder.

The nurse is teaching a class on preventing diaper rash newborns to a group of new parents which statement made by a parent indicates a correct understanding of the teaching

If my infants buttocks become slightly red I will expose the skin to air

The nurses taking care of a seven-year-old child with a skin rash called a papule which clinical findings should the nurse expect to assess with this type of skin rash

Illusion that is elevated, palpable, firm and circumscribed, less than 1 cm in diameter

Which explanation for Adema formation that occurs with burns

Increased capillary permeability

After the acute stage in during the healing process the primary complication from burn injury is

Infection

A child is admitted with extensive burns the nurse notes that there burns on the child's lips and since nasal hairs the nurse should suspect that the child has

Inhalation injury

A couple wants to start a family. They are concerned that their child will be at risk for cystic fibrosis because they each have a cousin with cystic fibrosis. They are seeing a nurse practitioner for preconceptual counseling. What would the nurse practitioner tell them about cystic fibrosis? a) It is an X-linked inherited disorder b) It is an autosomal dominant disorder c) It is an autosomal recessive disorder d) It is passed by mitochondrial inheritance

It is an autosomal recessive disorder Correct Explanation: Cystic fibrosis is autosomal recessive. Nurses also consider other issues when assessing the risk for genetic conditions in couples and families. For example, when obtaining a preconception or prenatal family history, the nurse asks if the prospective parents have common ancestors. This is important to know because people who are related have more genes in common than those who are unrelated, thus increasing their chance for having children with autosomal recessive inherited condition such as cystic fibrosis. Mitochondrial inheritance occurs with defects in energy conversion and affects the nervous system, kidney, muscle, and liver. X-linked inheritance, which has been inherited from a mutant allele of the mother, affects males. Autosomal dominant is an X-linked dominant genetic disease

Which statement about nondisjunction of a chromosome is true? a) Only the X chromosomes are affected. b) Only 4% of Down syndrome cases are attributed to this defect. c) It may result from genomic imprinting. d) It is failure of the chromosomal pair to separate.

It is failure of the chromosomal pair to separate. Correct Explanation: Nondisjunction simply means failure to separate. Nondisjunction can happen at any chromosome and is attributed to 95% of Down syndrome cases. Genomic imprinting is a different genetic disorder that is not related to nondisjunctioning.

Which is usually the only symptom of had lice

Itching

The family of a four-month-old infant will be vacationing at the beach which should the nurse teach the family about exposure of the infant to the sun

Keep the infant in total shade at all times

A mother calls emergency department nurse because her child was stung by a scorpion the nurse should recommend

Keeping the child quiet and coming to the emergency department

The nurse should implement which prescribed treatment for a child with warts

Local distraction

When assessing newborns for chromosomal disorders, which assessment would be most suggestive of a problem? a) Low-set ears b) Bowed legs c) Short neck d) Slanting of the palpebral fissure

Low-set ears Correct Explanation: A number of common chromosomal disorders, such as trisomies, include low-set ears.

An infant with craniosynostosis from Apert syndrome becomes lethargic and starts to vomit. What is the priority nursing intervention? a) Give IV dextrose b) Monitor intake and output c) Notify the doctor and prepare for surgery d) Reassess every hour and document findings

Notify the doctor and prepare for surgery Correct Explanation: The child is exhibiting signs and symptoms of increased intracranial pressure related to premature fusing of the skull joints. Surgery will be needed to relieve the pressure. IV dextrose is contraindicated with increased intracranial pressure. Waiting 1 hour to reassess may lead to brain damage and death. Monitoring intake and output is needed with a hospitalized child but is not the priority intervention based on presentation of symptoms.

The nurse is performing a physical examination on a 1-week-old girl with trisomy 13. Which of the following would the nurse expect to assess? a) Inspection reveals hypoplastic fingernails. b) Observation discloses severe hypotonia. c) Inspection shows a clenched fist with overlapping fingers. d) Observation reveals a microcephalic head.

Observation reveals a microcephalic head. Correct Explanation: Children with trisomy 13 have microcephalic heads with malformed ears and small eyes. Severe hypotonia, hypoplastic fingernails, and clenched fists with index and small fingers overlapping the middle fingers are typical symptoms of trisomy 18.

Biologic dressings are applied to a child with partial thickness burns on both legs which nursing intervention should be implemented

Observing rooms for signs of infection

A nurse teaching a couple says that when X-linked recessive inheritance is present in a family, the genogram will reveal which of the following? a) Only males in the family have the disorder. b) Sons of an affected man are also affected. c) A history of boys dying at birth for unknown reasons often exists. d) The parents of the affected man have the disorder.

Only males in the family have the disorder. Correct Explanation: When X-linked recessive inheritance is in a family, a genogram will reveal only males in the family with the disorder, a history of girls dying at birth for unknown reasons, unaffected sons of affected men, and parents of affected children not having the disorder

A female patient has the Huntington's disease gene. She and her husband want to have a child but are apprehensive about possibly transmitting the disease to their newborn child. They have strong views against abortion. They would also like to have their "own" child and would consider adopting only as a last resort. Which of the following would be most appropriate in this situation? a) Chancing the conception and birth of a child b) Using donor gametes for conception of a child c) Opting for a preimplantation genetic diagnosis d) Undergoing prenatal diagnosis with prenatal choice of continuing pregnancy

Opting for a preimplantation genetic diagnosis Correct Explanation: The most appropriate choice would be opting for a preimplantation genetic diagnosis (PGD). A PGD is a genetic evaluation of the embryo created through IVF which will reveal whether the Huntington's disease gene is present in the embryo. Undergoing prenatal diagnosis with prenatal choice of continuing pregnancy is not an option because the client and her husband are against abortion. Chancing the conception and birth of a child involves the risk of passing the gene to the newborn child. Using donor gametes may reduce the risk, but it is against the client's preferences.

The nurses taking care of a seven-year-old child with herpes simplex virus which prescribe medication should the nurse expect to be included in the treatment plan

Oral antiviral agent

Three sisters decide to have genetic testing done because their mother and their maternal grandmother died of breast cancer. Each of the sisters has the BRCA1 gene mutation. The nurse explains that just because they have the gene does not mean that they will develop breast cancer. What does the nurse explain their chances of developing breast cancer depend on? a) Susceptibility b) Their lifestyles c) Penetrance d) What other gene mutations they have

Penetrance Correct Explanation: A woman who has the BRCA1 hereditary breast cancer gene mutation has a lifetime risk of breast cancer that can be as high as 80%, not 100%. This quality, known as incomplete penetrance, indicates the probability that a given gene will produce disease. The other answers are incorrect because lifestyles, other gene mutations, and susceptibility are not the deciding factor in getting breast cancer if you have the BRCA1 gene mutation.

Which of the following conditions is a part of normal newborn screening? a) Down syndrome b) Cystic fibrosis c) Sickle cell anemia d) Phenylketonuria

Phenylketonuria Correct Explanation: Phenylketonuria is part of normal newborn screening. Prenatal screening includes Down syndrome. Preconception screening includes sickle cell anemia and cystic fibrosis.

A nurse is assessing a child diagnosed with Sturge-Weber syndrome. Which of the following would the nurse expect to find when assessing the skin? a) Pigmented nevi b) Café-au-lait spots c) Port wine stain d) Tumors

Port wine stain Correct Explanation: Facial nevus or port wine stain is most often seen on the forehead and on one side of the face. Café-au-lait spots are commonly associated with neurofibromatosis. Tumors are associated with tuberous sclerosis and neurofibromatosis. Pigmented nevi are associated with neurofibromatosis.

Herpes sister is caused by the Varicella virus and has an affinity for

Posterior root ganglia and posterior horn of the spinal cord

Nursing care of the infant with a topic dermatitis focuses on

Preventing infection

Women having in vitro fertilization (IVF) can have both the egg and sperm examined for genetic disorders of single gene or chromosome concerns before implantation. a) True b) False

True

Nitrous oxide is being administered to a child with extensive burn injuries which is the purpose of this medication

Provide anesthesia

Fentanyl and verse said are given before debridement of a child's birth and moon which is the rationale for administration of these medications

Provide pain relief

The nurses taking care of a child with scabies which primary clinical manifestation should the nurse expect to assess with this disease

Pruritus

A child experiences frostbite of the fingers after prolonged exposure to the cold which intervention should the nurse implement first

Rapid rewarming of the fingers by placing in warm water

A child with extensive burns requires debridement and or shouldn't have to pay which prior goal related to this procedure

Reduce pain

And adolescent girls cooking and a gas stove when her bathrobe catches fire your fathers mothers the flames of the rug because in ambulance she sustained major burns over much of her body which is important in her immediate care

Remove her burned clothes and jewelry

Parents tell the nurse that their child keeps scratching their areas where he is poison ivy the nurses response should be based on which knowledge

Scratching the lesions make laws them to become secondarily infected

Nurse is teaching a parent of an infant about treatment of cradle cap which should the nurse include in the instructions

Shampoo every day with an anti-sybaritic shampoo

The most immediate threat to life in children with thermal injury is is

Shock

The nurse is performing an assessment of a 6-year-old girl with Turner syndrome. Which of the following would the nurse most likely assess? a) Pectus carinatum b) Enlarged thyroid gland c) Short stature and slow growth d) Short, stubby trident hands

Short stature and slow growth Correct Explanation: Short stature and slow growth are frequently the first indication of Turner syndrome. While children with Turner syndrome are more prone to thyroid problems, these problems are not as likely to occur as in other symptoms. Pectus carinatum is typical of children with Marfan syndrome. Short, stubby trident hands are typical of achondroplasia.

An African American couple presents for a genetic counseling appointment. They are pregnant and are concerned about their child. What would a patient of African American heritage have genetic carrier testing for? a) Sickle cell anemia b) Asthma c) Rubella d) Meckel's diverticulum

Sickle cell anemia Correct Explanation: Assessing ancestry and ethnicity is important to help identify individuals and groups who could benefit from genetic testing for carrier identification, such as African Americans routinely offered testing for sickle cell anemia. The other answers are incorrect because they are not identified with the African American race

The child steps on a nail and sustains a puncture wound on the foot which is most appropriate method for cleansing the wound

Soak foot in warm water and soap

Cellulitis is often caused by

Streptococcus or Staphylococcus organisms.

A father calls the clinic nurse because his two-year-old child was bitten by a black widow spider the nurse should advise which to the father

Take the child to the emergency department

Which prescribe treatment should the nurse plan to implement for a child with psoriasis

Tar and exposure to sunlight and ultraviolet light

A Caucasian female client of Jewish ancestry is pregnant. The nurse is aware that the client may be a carrier for which of the following conditions? a) Tay Sachs disease b) Phenylketonuria c) Dupuytrens d) Krabbe disease

Tay Sachs disease Correct Explanation: Because the client is of Jewish ancestry, there is an increased risk of her being a carrier of the Tay Sachs disease gene. Norwegians are at a greater risk for Dupuytrens and Phenylketonuria, while Icelanders have an increased risk for Phenylketonuria.

The nurse is teaching parents of toddlers about animal safety which should be included in the teaching session

Teacher Tyler not to disturb and animal that is eating

When is Accutane indicated for the treatment of acne during adolescence

The acne has not responded to other treatments

A parent of a child with a major burn asked the nurse way high calorie high-protein diet as prescribed which response should the nurse make

The diet will avoid protein breakdown

A baby is born with what the physician believes is a diagnosis of trisomy 21. This means that the infant has three number 21 chromosomes. What factor describes this genetic change? a) The mother also has genetic mutation of chromosome 21 b) The patient has a nondisjunction occurring during meiosis c) The patient will have a single X chromosome and infertility d) During meiosis, a reduction of chromosomes resulted in 23

The patient has a nondisjunction occurring during meiosis Correct Explanation: During meiosis, a pair of chromosomes may fail to separate completely, creating a sperm or oocyte that contains either two copies or no copy of a particular chromosome. This sporadic event, called nondisjunction, can lead to trisomy. Down syndrome is an example of trisomy. The mother does not have a mutation of chromosome 21, which is indicated in the question. Also, Trisomy does not produce a single X chromosome and infertility. Genes are packaged and arranged in a linear order within chromosomes, which are located in the cell nucleus. In humans, 46 chromosomes occur in pairs in all body cells except oocytes and sperm, which contain only 23 chromosomes.

The nurse is caring for a five-year-old child with in Pedigo contagiosum the parents asked the nurse what will happen to their child skin after the infection has subsided and healed would change or should the nurse give

There will be no scarring

A woman with both heart disease and osteoarthritis has come to the genetics clinic for genetic screening. What would the nurse know about these two diseases? a) They are direct result of the patient's lifestyle b) They are multifactorial c) They do not have a genetic basis d) They are caused by a single gene

They are multifactorial Correct Explanation: Genomic or multifactorial influences involve interactions among several genes (gene-gene interactions) and between genes and the environment (gene-environment interactions), as well as the individual's lifestyle.

A two-year-old has been admitted with a diagnosis of Kawasaki disease. Which of the following would be a priority on the plan of care for this child? A. vital signs every 6 hours B. Hourly intake and output records C. Skin care D. Passive range-of-motion exercises

Those with Kawasaki disease are at high-risk for CHF in the initial stages. Vital signs would need to be recorded more often than every 6 hours. Skin care and ROM exercises are important, but not the priority at this time.

Rocky Mountain spotted fever is caused by the bite of a

Tick

What is the main purpose of nurses having basic genetic knowledge? a) To advocate for a cure for genetic disorders b) To provide support and education to families c) To understand all genetic disorders, allowing for improved quality of life d) To ensure proper medical diagnosis

To provide support and education to families Correct Explanation: The purpose of the nurse knowing about basic genetics is that it helps her to provide support and education to families. Nurses can advocate for a cure, but this is not the main purpose of attaining basic knowledge of genetics. Providing a medical diagnosis is beyond the scope of practice for a nurse. It would be impossible for the nurse to understand all genetic disorders; it is more reasonable for the nurse to be familiar with the most common genetic disorders

A number of inherited diseases can be detected in utero by amniocentesis. Which of the following diseases can be detected by this method? a) Diabetes mellitus b) Phenylketonuria c) Trisomy 21 d) Impetigo

Trisomy 21 Correct Explanation: Karyotyping for chromosomal defects can be carried out using amniocentesis.

The nurse is teaching nursing students about childhood skin lesions which an elevated circumscribed skin lesion that is less than 1 cm diameter and filled with serous fluid

Vesicle

The nurse should expect to assess which closet of agent and a child with warts

Virus

Which is an important consideration for the nurse when changing dressings in applying topical medications to a child's abdominal and leg burns

Wash hands and forearms before and after dressing change

Is school-age child falls on the playground and has a small laceration on the forum the school nurse should do which to cleanse the wound

Washroom gently with mild soap and water for several minutes

A young child has sustained a minor burn to the foot which is recommended for treatment of a minor burn

When is the world with a mild soap and tepid water

A 33-years old female is admitted to the hospital with a suspected diagnosis of grave's disease. Which symptom related to the client's menstrual cycle would the client likely report? a) amenorrhea b) metrorrhagia c) menorrhagia d) dysmenorrha

a) amenorrhea

The nurse is talking to the parents of a child with headlights which should the nurse include when explaining how to manage the headlights

You will need to remove minutes with an extra fine tooth comb or tweezers

You are assessing a 5-year-old who has been admitted with an acute respiratory infection. You review the chart and see the child has cystic fibrosis. What is the priority for assessment? a. Opening and maintain a patent airway b. Giving antibiotics to the child c. Checking levels of pancreatic enzymes d. Modifying the child's diet

a. Opening and maintain a patent airway

3. What type of gene is CAH: a) Autosomal recessive gene b) Autosomal Dominate gene

a) Autosomal recessive gene

When caring for a client with suspected SIADH, the nurse reviews the medical record to uncover which signs and symptoms consistent with this syndrome? (select all that apply) A. Hyponatremia B. Mental status changes C. Azotemia D. Bradycardia E. Weakness

a,b,e: ADH is secreted or produced ectopically, resulting in water retention and sodium dilution which causes confusion and changes in mental status and weakness. Tachycardia may result from fluid volume excess.

The client has a diagnosis of lung cancer. To which areas does the nurse anticipate that this client's tumor may metastasize? Select all that apply. A. Brain B. Bone C. Lymph nodes D. Kidneys E. Liver

a-c, e. as well as the pancreas.

A nurse is teaching the parent of a child who has hand, foot, and mouth disease. Which of the following should be included in the teaching? a. "Your child can be contagious when the symptoms are gone." b. "The incubation period is 10 to 21 days." c. "It is transmitted by droplet." d. "Once infected, your child will be a lifetime carrier."

a. "Your child can be contagious when the symptoms are gone."

A nurse is performing a neonatal assessment. Which assessment should the nurse use to assess for developmental dysplasia of the hip? (Select all that apply) a. Barlow Test b. Trendelenburg sign c. Manipulation of foot and ankle d. Ortolani test

a. Barlow Test d. Ortolani test

The nurse is talking with the family of an 18 month-old newly diagnosed with retinoblastoma. A priority in communicating with the parents is. a. Discuss the need for genetic counseling b. Inform them that combined therapy is seldom effective c. Prepare for the child's permanent disfigurement d. Suggest that totally blindness may follow surgery.

a. Discuss the need for genetic counseling

Which of the following are characteristics of Von Willebrand's Disease? Select all that apply. a. Easy bruising occurs b. Gum bleeding occurs c. It is a hereditary bleeding disorder d. Its characterized by extremely high creatinine levels

a. Easy bruising occurs b. Gum bleeding occurs c. It is a hereditary bleeding disorder

Which phase includes red eyes, high fever, strawberry tongue and rash on main trunk of the body? a. First Phase b. Second Phase c. Third Phase

a. First Phase

A 4 year old is brought into the emergency department by his mother. She reports that he has had an uncontrollable fever for five days. You, the nurse, notice that his lips are cracking, swelling of the hands and feet, erythema of the soles and palms, and a generalized rash. What do you expect the diagnosis from the doctor to be? a. Kawasaki Disease b. Meningitis c. Rheumatic Fever d. Shingles

a. Kawasaki Disease

2. What is a characteristic of chickenpox? a. Lesion appear in crops b. Koplik Spots c. Petechiae on soft palate d. Abdominal cramps

a. Lesion appear in crops

Nurse Mariane is caring for an infant with spina bifida. Which technique is most important in recognizing possible hydrocephalus? a. Measuring head circumference b. Obtaining skull X-ray c. Performing a lumbar puncture d. Magnetic resonance imaging (MRI)

a. Measuring head circumference

What is the major source of nutrients in the basic American diet that correlates with lactose intolerance? a. Milk and dairy b. Fiber and sodium c. Potassium and bananas d. Iron and red meats

a. Milk and dairy

The nurse teaches the mother of a young child with Duchenne's muscular dystrophy about the disease and it's management. Which of the following statements by the mother indicates successful teaching? a. My son will probably be unable to walk independently by the time he is 9 to 11 years old. b. Muscle relaxants are effective for some children; I hope they help my son. c. When my son is a little bit older, he can have surgery to improve his ability to walk. d. I need to help my son be as active as possible to prevent progression of the disease.

a. My son will probably be unable to walk independently by the time he is 9 to 11 years old.

A nurse is caring for a child after spinal fusion for scoliosis treatment. The child complains of abdominal discomfort and begins to have episodes of vomiting. On further assessment, the nurse notes abdominal distention. Based on these findings, the nurse should take which action? a. Notify the physician. b. Administer an antiemetic. c. Increase the intravenous fluids. d. Place the child in a Sims' position.

a. Notify the physician.

When planning a screening clinic for scoliosis, the nurse would anticipate targeting which of the following groups? a. Preadolescents at the beginning of a growth spurt. b. Toddlers who have diets low in calcium and vitamin D. c. Preschoolers who are entering kindergarten. d. Infants whose mothers have had no prenatal care.

a. Preadolescents at the beginning of a growth spurt.

A child with Atopic Dermatitis has higher-than-normal colonization of which bacteria? a. Staphylococcus aureus b. Escherichia coli c. streptococci d. Neisseria

a. Staphylococcus aureus

What is one of the major precipitating factors in the development of Irritable Bowel Syndrome? a. Stress b. Ulcers c. GERD d. Stomach Flu

a. Stress

Which of the following interventions is most appropriate for helping parents to cope with a child newly diagnosed with bronchopulmonary dysplasia? a. Teach cardiopulmonary resuscitation b. Refer them to support groups c. Help parents identify necessary lifestyle changes d. Evaluate and assess parents; stress and anxiety levels

a. Teach cardiopulmonary resuscitation

3. Your 8-yr old patient has DI. While educating the family, the mother asks "If our son has dry mouth and seems extra tired we should immediately give him fluids, right?" Is the mother correct or incorrect?

a. The mother is correct

Why is diagnosing Kawasaki disease so difficult? (Select all that apply) a. There is no specific test to diagnose Kawasaki disease b. Diagnosis is primarily ruling out all other diseases that cause similar signs and symptoms c. Treatment needs to occur within a 10 day window

a. There is no specific test to diagnose Kawasaki disease b. Diagnosis is primarily ruling out all other diseases that cause similar signs and symptoms c. Treatment needs to occur within a 10 day window

Which of the following anomalies typically accompanies hypospadias? a. Undescended testicles b. Ambiguous genitalia c. Umbilical hernias d. Inguinal hernias

a. Undescended testicles

\A nurse is caring for an infant. Which of the following are clinical manifestations of coarctation of the aorta? Select all that apply. a. Weak femoral pulses b. Cool skin of the lower extremities c. Severe cyanosis d. Clubbing of the fingers e. Heart failure

a. Weak femoral pulses b. Cool skin of the lower extremities e. Heart failure

What are some signs and symptoms of croup (select all that apply): a. difficulty swallowing b. rapid breaths (80 breaths per minute) c. sucking in of the chest d. non stop drooling e. vision loss

a. difficulty swallowing b. rapid breaths (80 breaths per minute) c. sucking in of the chest d. non stop drooling

The nurse is caring for a neonate born with a myelomeningocele. Surgery to repair the defect is scheduled the next day. The most appropriate way to position and feed this neonate is to place him: a. prone with head turned to side for feeding. b. on side to facilitate feeding. c. supine in infant carrier for feedings. d. supine, with defect supported with rolled blankets, and with nipple-feeding

a. prone with head turned to side for feeding.

\How would you conserve a child's energy that has coarctation of the aorta? Select all that apply. a. providing frequent rest periods b. clustering care c. providing large meals d. bathing every day

a. providing frequent rest periods b. clustering care

What is the most common bacteria that causes osteomyelitis? a. staphylococcus aureus b. kingella kingae c. escherichia coli d. group B streptococci

a. staphylococcus aureus

All of the following are true statements regarding impetigo EXCEPT: a. The child's school should be notified of the diagnosis b. Scarring may occur if the child picks off the scar prematurely c. Most common during the dry winter months d. Commonly diagnosed by the appearance of the pustluar or crusty lesions

c. Most common during the dry winter months

2. What occurs in salt water crisis a) Hypervolemia b) Hypovolemia c) Hyperkalemia d) hypokalemia

b) Hypovolemia

2. A child with hemophilia states that he wants to participate in sports. Which sport should the nurse recommend as most appropriate for the child? a)Running b)Swimming c)Biking d)Baseball

b)Swimming

2. At what age do children normally have corrective surgery? a. Right away b. 3-6 months c. 3-6 years old d. 1-2 month

b. 3-6 months

\When caring for a male client with severe impetigo, the nurse should include which intervention in the plan of care? a. Placing mitts on the client's hands b. Administering systemic antibiotics as prescribed c. Applying topical antibiotics as prescribed d. Continuing to administer antibiotics for 21 days as prescribed

b. Administering systemic antibiotics as prescribed

3. Which of the following is NOT a risk factor for shingles? a. History of chickenpox b. Age over 20 c. Being treated for cancer d. Weakened immune system

b. Age over 20

What kind of isolation would a hospitalized child with hand, foot, and mouth disease have? a. Droplet b. Contact c. Airborne d. No isolation

b. Contact

The nurse is planning a diet for an eight-year old with cystic fibrosis (CF). which of the following dietary requirements should be considered? a. High protein, high fat and high calories. b. High protein, low fat and high calories. c. Low protein, low fat and low carbohydrates. d. High protein, high fat and low carbohydrates.

b. High protein, low fat and high calories.

What is the most common occurrence of croup a. When there is a bacterial infection b. In boys that are between the age of 6 months and 3 years c. In the lower respiratory tract d. When caused by S. Aureous

b. In boys that are between the age of 6 months and 3 years

What are the most common findings of Retinoblastoma? a. Pain and discharge b. Leukocoria and strabismus c. Photosensitivity and cataracts d. Glaucoma and colored blindness

b. Leukocoria and strabismus

A nurse is caring for a child who is in Bryant traction. Which of the following is an appropriate action for the nurse to take? a. Place the client in bed with the head of the bed flat and the legs elevated b. Maintain the hips at a 90* angle to the body c. Place the patient in the prone position d. Place the weights on the floor

b. Maintain the hips at a 90* angle to the body

3. Cleft lip and palate normally occur in what population? Select all the apply a. Mexicans b. Native Americans c. Asians d. African Americans

b. Native Americans c. Asians

The most important nursing intervention when caring for a child with myelomeningocele in the preoperative stage is which of the following? a. Take vital signs every hour. b. Place child on side to decrease pressure on the spinal sac. c. Watch for signs that might indicate developing hydrocephalus. d. Apply a heat lamp to facilitate drying and toughening of the sac.

b. Place child on side to decrease pressure on the spinal sac.

3. Which statemtent(s) about VWD are accurate? Select all that apply. a. VWD is an acquired bleeding disorder. b. Prolonged and excessive bleeding and menorrhagia are signs of VWD. c. The von Willebrand protein carries coagulation factor IX. d. VWD is sometimes treated with desmopressin acetate (DDAVP).

b. Prolonged and excessive bleeding and menorrhagia are signs of VWD. d. VWD is sometimes treated with desmopressin acetate (DDAVP).

Which of the following factors should the nurse recognize as predisposing the urinary tract to infections in males or females? a. Increased fluid intake b. Short urethra in young females c. Ingestion of highly acidic juices d. Frequent emptying od the bladder

b. Short urethra in young females

Which statement is true about Celiac Disease: a. The disease results from the inability to digest lactose b. Siblings and children of affected individuals are at a higher risk for the disease c. The major manifestations of celiac disease include fever and vomiting. d. Celiac disease is temporary if treated promptly.

b. Siblings and children of affected individuals are at a higher risk for the disease

A nurse performs an admission assessment on a female client with a diagnosis of tuberculosis. The nurse reviews the results of which diagnostic test that will confirm this diagnosis? a. Bronchoscopy b. Sputum culture c. Chest x-ray d. Tuberculin skin test

b. Sputum culture

Which of the following symptoms is seen in a child with bronchopulmonary dysplasia? a. Minimal work of breathing b. Tachypnea and dyspnea c. Easily consolable d. Hypotension

b. Tachypnea and dyspnea

What is a primary nursing interventions for the adolescent with IBS? a. Teach them to take laxatives if they feel constipated for more than 24 hours. b. Teach them to maintain well-balanced meals, eat lots of fruits and vegetables, and maintain a exercise regimen. c. Drink coffee to help with symptoms d. Eat 3x the amount of recommended fiber to help with symptoms.

b. Teach them to maintain well-balanced meals, eat lots of fruits and vegetables, and maintain a exercise regimen.

The parent of a child with TB asks a nurse to explain how the TB is transmitted to others. What is the most appropriate response by the nurse: a. Hand and mouth b. The airborne route c. The fecal-oral route d. Blood and body fluids

b. The airborne route

When caring for a patient who has Guillain-Barré syndrome, which assessment data obtained by the nurse will require the most immediate action? a. The patient complains of severe tingling pain in the feet. b. The patient has continuous drooling of saliva. c. The patient's blood pressure (BP) is 106/50 mm Hg. d. The patient's quadriceps and triceps reflexes are absent.

b. The patient has continuous drooling of saliva.

The mother of a child with hemophilia asks the nurse which over-the-counter medication is suitable for her child's joint discomfort. The nurse should tell the mother to purchase: a. Advil (ibuprofen) b. Tylenol (acetaminophen) c. Aspirin (acetylsalicytic acid) d. Naproxen (naprosyn)

b. Tylenol (acetaminophen)

You are the nurse caring for a 3 year old patient with Kawasaki Disease. What medications should you expect to find? a. methotrexate b. aspirin and intravenous immune globulin c. digoxin d. IV tetracycline and furosemide

b. aspirin and intravenous immune globulin

What is a key sign of a child who has coarctation of the aorta? a. Swollen joints, hands , and feet b. high blood pressure in your arms, but low blood pressure in your legs and ankles c. Loud harsh, murmur d. Cyanosis

b. high blood pressure in your arms, but low blood pressure in your legs and ankles

Which diet should the nurse recommend for a child with celiac disease? a. Wheat and oats b. rice and corn c. barley and rye d. both a and c are correct

b. rice and corn

What is the most common type of CAH: a) Low serum levels b) Ambiguous genitalia c) Salt water crisis d) Increased sodium levels

c) Salt water crisis

After the nurse provides dietary restrictions to the parents of a child with celiac disease, which statement by the parents indicates effective teaching? a. "We will follow this instructions until the symptoms disappear." b. "Our child must maintain these dietary restrictions until adulthood." c. "Our child must maintain these dietary restrictions lifelong." d. "We will follow these instructions until our child had completely grown and developed."

c. "Our child must maintain these dietary restrictions lifelong."

A child is admitted to the hospital with suspected Rheumatic Fever. Which of the following does NOT confirm the diagnosis? a. Reddened rash visible over trunk and extremities b. History of sore throat that is self-limiting in the past c. A negative antistreptolysin O titer d. An unexplained fever

c. A negative antistreptolysin O titer

The most important measure to include in the nursing management for a child with cystic fibrosis would be to? a. Promote optimal nutrition with a high-protein, low-fat diet b. Administer only water-soluble vitamins c. Administer pancreatic enzymes before each meal d. Encourage lots of fluids, especially fruit juices

c. Administer pancreatic enzymes before each meal

What is a major sign/symptom of Irritable Bowel Syndrome? a. Vomitting with alternating diarrhea b. Constipation that goes away within 1 week c. Alternating bowel habits such as diarrhea and constipation d. Impaction

c. Alternating bowel habits such as diarrhea and constipation

Antihistamines, such as diphenhydramine (Benadryl) should be administered to a child with Atopic Dermatitis in which way? a. With food b. In the morning c. At Bedtime d. Without food

c. At Bedtime

What food is appropriate for a lactose-free diet? a. Yogurt b. Ice cream c. Broccoli d. Pancakes

c. Broccoli

When is the child with croup most likely to infect others? a. After the incubation period b. Once the cough recedes and the child's temperature is back to normal c. During the first days when the child has a temperature d. Once a cough begins

c. During the first days when the child has a temperature

All of the following are used to diagnosis osteomyelitis, except... a. Radiography b. MRI c. Echocardiogram d. Lab values: elevated ESR

c. Echocardiogram

Which of the following orgnaisms is responsible for the development of Rheumatic Fever? a. Streptococeal pneumonia b. Haemophilus Influenza c. Group A B-Hemolytic Streptococcus d. Staphylococcus aureus

c. Group A B-Hemolytic Streptococcus

Prevention of Rheumatic Fever can best be accomplished by: a. Keeping children with a fever home from school b. Sending children with sore throats home from school c. Having sore throats cultured as soon as possible d. Treating all colds with antibiotics

c. Having sore throats cultured as soon as possible

Infants with bronchopulmonary dysplasia are commonly treated with bronchodilators such as theophylline. Which of the following adverse effects is common with this drug? a. Lethargy b. Decreased calcium level c. Increased heart rate d. Decreased serum potassium level

c. Increased heart rate

What device is used to realign parts of the palate before surgery? a. No device is used b. Surgery is right away c. Latham d. Lip or palate brace

c. Latham

A nurse is caring for a child who has Muscular Dystrophy. For which of the following findings should the nurse asses? (select all that apply) a. Purposeless, involuntary, abnormal movements b. Spinal defect and saclike protrusion c. Muscular weakness in lower extremities d. Unsteady, wide-based gait or waddling gait e. Upward slant to the eyes

c. Muscular weakness in lower extremities d. Unsteady, wide-based gait or waddling gait

In Retinoblastoma, what is Enucleation? a. A tumor in the lungs b. A yellow discharge in the eye c. Removal of the eye d. A form of cancer

c. Removal of the eye

An example of a disorder inherited in an autosomal-inherited pattern is: a. Hemophilia A b. Beta-thalassemia c. VWD

c. VWD

The nursing care plan for a toddler diagnosed with Kawasaki Disease should be based on the high risk for development of which problem? a. hypertension b. seizures c. coronary artery aneurysms d. stroke

c. coronary artery aneurysms

Your 10-yr old patient is diagnosed with Diabetes Insipidus. You are to treat them with synthetic vasopressin. You read the order wrong and administer 6mcg/mL subcutaneously to the child. You will notice the child: a. acting normal and states he feels better than ever b. has increased urine output and sunken eyes c. has decreased urine output, edema, and complaints of a headache d. the child immediately falls asleep

c. has decreased urine output, edema, and complaints of a headache

A client with hemophilia has a very swollen knee after falling from bicycle riding. Which of the following is the first nursing action? a)initiate an IV site to begin administration of cryoprecipitate b) type and cross-match for possible transfusion c) monitor the client's vital signs for the first 5 minutes d) apply ice pack and compression dressings to the knee

d) apply ice pack and compression dressings to the knee

A nurse is caring for a toddler who is diagnosed with hip dysplasia and has been placed in a hip spica cast. The child's mother asks the nurse why a Pavlik harness is not being used. Which of the following responses by the nurse appropriately addresses the mother's question? a. "The Pavlik harness is used for children with scholiosis, not hip dysplasia." b. "The Pavlik harness is used for school-age children." c. "The Pavlik harness cannot be used for your child because her condition is too severe." d. "The Pavlik harness is used for infants less than 6 months of age."

d. "The Pavlik harness is used for infants less than 6 months of age."

When reviewing the results of a clean-voided urine specimen, which of the following results would indicate to the nurse that the child may have a urinary tract infection? a. A specific gravity of 1.02 b. Cloudy color without odor c. A large amount of casts present d. 100,000 bacterial colonies per milliliter

d. 100,000 bacterial colonies per milliliter

Impetigo is caused by all of the following EXCPET: a. S. aureus b. Group A beta-hemolytic streptococci c. Sarcoptes scabiei d. A combinations of S. aureus and group A beta-hemolytic streptococci

d. A combinations of S. aureus and group A beta-hemolytic streptococci

A community health nurse is conducting an educational session with community members regarding tuberculosis. The nurse tells the group that one of the first symptoms associated with tuberculosis is: a. Dyspnea b. Chest pain c. A bloody, productive cough d. A cough with the expectoration of mucoid sputum

d. A cough with the expectoration of mucoid sputum

By which mode(s) of transmission is chickenpox spread? a. Fecal - oral route b. Sexually transmitted c. Bite of infected animal d. Direct contact or airborne

d. Direct contact or airborne

When talking with the parents of a child or adolescent with a history of Cryptorchidism, the nurse will most stress the importance of the child doing which of the following things? a. Wearing a jock strap while participating in sports b. Complying 100% with hormone therapy beginning at age 15 c. Getting a mumps vaccine booster every 5 years d. Doing monthly self-testicular exams beginning at age 15

d. Doing monthly self-testicular exams beginning at age 15

The nurse should include which of the following facts when teaching parents about handling a child with recurrent urinary tract infections? a. Antibiotics should be discontinued 48 hours after symptoms subside. b. Recurrent symptoms should be treated by renewing the antibiotics prescription. c. Complicated urinary tract infections are related to poor perineal hygiene practice. d. Follow-up urine cultures are necessary to detect recurrent infections and antibiotics effectiveness.

d. Follow-up urine cultures are necessary to detect recurrent infections and antibiotics effectiveness.

When trying to stand the child puts hands on knees and moves the hands up the legs until in a standing position. The name for this is? a. limb girdle b. myotonic c. Becker d. Growers maneuver

d. Growers maneuver

A nurse has provided discharge instructions to the mother of a 2-year-old child who had an orchiopexy to correct Cryptorchidism. Which statement made by the mother of the child indicates that further teaching is needed? a. I'll check his temperature b. I'll give him medication so he will be comfortable c. I'll check his voiding to ensure there are no problems d. I'll let him decide when to return to his play activities

d. I'll let him decide when to return to his play activities

A patient who has numbness and weakness of both feet is hospitalized with Guillain-Barré syndrome. The nurse will anticipate that collaborative interventions at this time will include a. intubation and mechanical ventilation. b. insertion of a nasogastric (NG) feeding tube. c. administration of methylprednisolone (Solu-Medrol). d. IV infusion of immunoglobulin (Sandoglobulin).

d. IV infusion of immunoglobulin (Sandoglobulin).

Which nursing diagnosis takes highest priority for a female client with hyperthyroidism? a. Risk for imbalanced nutrition: More than body requirements related to thyroid hormone excess b. Risk for impaired skin integrity related to edema, skin fragility, and poor wound healing c. Body image disturbance related to weight gain and edema d. Imbalanced nutrition: Less than body requirements related to thyroid hormone excess

d. Imbalanced nutrition: Less than body requirements related to thyroid hormone excess

A nurse is teaching a group of parents about communicable diseases. Which of the following is the most appropriate method to prevent a communicable disease? a. Handwashing b. Avoiding persons with active disease c. Covering your cough d. Obtaining immunizations

d. Obtaining immunizations

What special consideration should you teach parents when caring for a child with Atopic Dermatitis? a. Instruct parents to keep the child's skin dry b. Teach parents to keep child in water for a long period of time to promote hydration c. Instruct parents to dress child in heavy clothing to induce sweating d. Teach parents to use mild detergents and rinse the clothing twice after washing.

d. Teach parents to use mild detergents and rinse the clothing twice after washing.

Which is NOT an expected outcome for person with lactose intolerance? a. The child will be free from abdominal pain b. The child will have soft, formed stools c. The child will state foods to be avoided or provided in diet and appropriate lactase products d. The child would contain a diet that includes dairy products

d. The child would contain a diet that includes dairy products

Which child is at the highest risk for osteomyelitis? a. a female 13 year old who likes playing soccer b. a male 10 year old with the flu c. a female 4 month old who was small for gestational age d. a male preschooler who likes climbing trees

d. a male preschooler who likes climbing trees

The physician confirms a diagnosis if DI. As a nurse, you should be alert for: a. dysuria b. decreased thirst and decreased urine output c. hyponatremia d. increased urine output and excessive thirst

d. increased urine output and excessive thirst

A 24-year-old patient is hospitalized with the onset of Guillain-Barré syndrome. During this phase of the patient's illness, the most essential assessment for the nurse to carry out is a. monitoring the cardiac rhythm continuously. b. determining the level of consciousness q2hr. c. evaluating sensation and strength of the extremities. d. performing constant evaluation of respiratory function.

d. performing constant evaluation of respiratory function.

A gene that is expressed when paired with another gene for the same trait is called a) recessive. b) dominant. c) heterozygous. d) homozygous.

dominant. Correct Explanation: A dominant gene is one that will be expressed when paired with a like gene.

When an infant is born with a genetic disorder, it is appropriate to advise the parents that a) experiences the mother had during pregnancy are probably not related. b) not all genetic disorders are inherited. c) the disorder has probably occurred in the family before. d) it is likely that the mother drank alcohol during early cell division.

experiences the mother had during pregnancy are probably not related. Explanation: As genetic disorders occur at the moment of conception, events during pregnancy occur after the problem is already present.

A nursing student correctly identifies that a person's outward appearance or expression of genes is referred to as which of the following? a) allele b) genome c) phenotype d) genotype

phenotype Correct Explanation: Alleles are two like genes. Phenotype refers to a person's outward appearance or the expression of genes. Genotype refers to his or her actual gene composition. Genome is the complete set of genes present in a person.

Girls with Turner Syndrome will usually exhibit a) short stature b) progressive dementia c) chorealike movements d) painful joints

short stature Correct Explanation: Girls with Turner syndrome usually have a single X chromosome, causing them to have short stature and infertility. Persons with sickle cell anemia have painful joints. Color blindness occurs in persons diagnosed with Huntington disease and they may exhibit chorealike movements. Progressive dementia occurs in early-onset familial Alzheimer's disease.

True or False, In a hospital setting, children with Herpes Zoster should have their own private room and remain in isolation until infection is over

true


Kaugnay na mga set ng pag-aaral

GMAT Focus Practice Exam Question Review

View Set

PREPU WK 1 CHP 12, 65, 70, 68, 63

View Set

Communication 457 Quiz 3-Scripts, Episodes, and Nonverbal Communication

View Set

Chapter 6 General Anatomy: Fill in the Blanks

View Set

Political Statement: from the Universal Declaration of Human Rights

View Set

Ch. 26, Bipolar and related disorders

View Set

The 8 Elements of Humanistic Psychology

View Set

az-900 sample questions 11-22-22

View Set